2
0
Fork 0
mirror of https://github.com/MartinThoma/LaTeX-examples.git synced 2025-04-11 15:38:15 +02:00

Remove trailing spaces

The commands

find . -type f -name '*.md' -exec sed --in-place 's/[[:space:]]\+$//' {} \+

and

find . -type f -name '*.tex' -exec sed --in-place 's/[[:space:]]\+$//' {} \+

were used to do so.
This commit is contained in:
Martin Thoma 2015-10-14 14:25:34 +02:00
parent c578b25d2f
commit 7740f0147f
538 changed files with 3496 additions and 3496 deletions

View file

@ -10,6 +10,7 @@
this should include the rendered image. For documents, there should be
some example text explaining what it is about.
3. Use spaces for indenting.
4. Don't have trailing spaces.
## Commit messages

View file

@ -5,7 +5,7 @@ Compiled example
Currently, this one is not working, because of this error:
```
/usr/local/texlive/2013/texmf-dist/asymptote/three.asy:
/usr/local/texlive/2013/texmf-dist/asymptote/three.asy:
2906.13: runtime: to support onscreen rendering, please install glut library, run ./configure, and recompile
```

View file

@ -1,7 +1,7 @@
I took this from a [StackExchange Question](http://tex.stackexchange.com/a/84757/5645)
If you want to change it (I know you can make it better with LaTeX!)
or if you have some important commant, feel free to submit a merge
or if you have some important commant, feel free to submit a merge
request or send me an email (info@martin-thoma.de)

View file

@ -33,7 +33,7 @@
\item Basic path: \myCode{\textbackslash drawing-command [options] path-specification;}
\item Path specification: \myCode{(coordinate) path-component (coordinate);}
\item Path Reusage \myCode{postaction=\{<basic drawing commands> or <decorate>\}} When this option is given to any basic drawing commands below, the path is not immediately discarded and reused after the initial drawing command is finished. \\
\myCode{preaction=\{<basic drawing commands> or <decorate>\}} When this option is given to any basic drawing commands below, the path is used once before the initial drawing command is executed.
\myCode{preaction=\{<basic drawing commands> or <decorate>\}} When this option is given to any basic drawing commands below, the path is used once before the initial drawing command is executed.
\end{itemize}
\section*{Basic Drawing Commands}

View file

@ -8,8 +8,8 @@
\begin{preview}
\begin{circuitikz}
\draw (0,0) node[european and port] (myand){}
(myand.in 1) node[anchor=east]{P}
(myand.in 2) node[anchor=east]{Q}
(myand.in 1) node[anchor=east]{P}
(myand.in 2) node[anchor=east]{Q}
(1.8,0) node[european not port] (mynot){}
(mynot.out) node[anchor=west]{S}
(myand.out) -- (mynot.in);

View file

@ -11,11 +11,11 @@
\title{The Game '2048'}
\author{Martin Thoma}
\hypersetup{
pdfauthor = {Martin Thoma},
pdfkeywords = {Game, combinatorics, 2048},
pdftitle = {The Game '2048'}
}
\hypersetup{
pdfauthor = {Martin Thoma},
pdfkeywords = {Game, combinatorics, 2048},
pdftitle = {The Game '2048'}
}
%%%%%%%%%%%%%%%%%%%%%%%%%%%%%%%%%%%%%%%%%%%%%%%%%%%%%%%%%%%%%%%%%%%%%
% Begin document %
@ -36,7 +36,6 @@
\section{Related}
\begin{itemize}
\item \url{http://math.stackexchange.com/q/716469/6876}
\item
\end{itemize}
\end{document}

View file

@ -33,7 +33,7 @@
\foreach \line [count=\y] in \pixels {
\foreach \pix [count=\x] in \line {
\draw[fill=pixel \pix] (\x,-\y) rectangle +(1,1);
\ifthenelse{\equal{0}{\pix}}
{}
{\node at ($(\x,-\y) + (0.5,0.5)$) {\Huge \pix};}

View file

@ -10,11 +10,11 @@
\semester{Wintersemester 04/05}
\scriptstate{complete}
\hypersetup{
pdfauthor = {Die Mitarbeiter von http://mitschriebwiki.nomeata.de/ und GitHub-Beiträge},
pdfkeywords = {Analysis},
pdftitle = {Analysis I}
}
\hypersetup{
pdfauthor = {Die Mitarbeiter von http://mitschriebwiki.nomeata.de/ und GitHub-Beiträge},
pdfkeywords = {Analysis},
pdftitle = {Analysis I}
}
\author{Die Mitarbeiter von \url{http://mitschriebwiki.nomeata.de/} und GitHub-Beiträge}
\title{Analysis I}
@ -95,7 +95,7 @@ In $\MdR$ sind zwei Verknüpfungen "`+"' und "`$\cdot$"' gegeben, die jedem Paar
\end{liste}
\end{axiom}
Dabei nennt man \textbf{A1} und \textbf{A2} \begriff{Assoziativgesetze}, \textbf{A3} und \textbf{A4} \begriff{Kommutativgesetze} und \textbf{A9} \begriff{Distributivgesetz},
Dabei nennt man \textbf{A1} und \textbf{A2} \begriff{Assoziativgesetze}, \textbf{A3} und \textbf{A4} \begriff{Kommutativgesetze} und \textbf{A9} \begriff{Distributivgesetz},
Alle Regeln der Grundrechenarten lassen sich aus \textbf{(A1)} bis \textbf{(A9)} herleiten. Diese Regeln seien von nun an bekannt.
@ -140,11 +140,11 @@ In \MdR\ ist eine Relation "`$\le$"' gegeben. Es sollen gelten:
\end{schreibweisen}
\begin{definition}[Betrag]
Für $a \in \MdR$ heißt $ |a| :=
Für $a \in \MdR$ heißt $ |a| :=
\begin{cases}
a & \mbox{falls } a \ge 0 \\
-a & \mbox{falls } a < 0
\end{cases} $.
\end{cases} $.
$|a|$ wird der \begriff{Betrag} von a genannt und entspricht dem "`Abstand"' von $a$ und $0$. $|a-b|$ entspricht dem "`Abstand"' von $a$ und $b$.
\end{definition}
@ -275,12 +275,12 @@ Für jedes $n \in \MdN$ sei eine Aussage $A(n)$ gemacht. Es gelte: (I) $A(1)$ is
\item $A(n) := n \ge 1$. $A(n)\ \forall n \in \MdN$. Beweis (induktiv):\\
Induktionsanfang (IA): $1 \ge 1$, also ist $A(1)$ wahr. \\
Induktionsvorausseztung (IV): Sei $n \in \MdN$ und $A(n)$ wahr (also $n \ge 1$) \\
Induktionsschritt (IS, $n \curvearrowright n + 1$): $n+1
Induktionsschritt (IS, $n \curvearrowright n + 1$): $n+1
\stackrel{(IV)}{\ge} 1 + 1 \ge 1$, also $A(n+1)$ wahr.
\item Für $n \in \MdN$ sei $A_n:=(\MdN\ \cap\ [1,n])\ \cup\ [n+1, \infty)$. \\
Behauptung: $\underbrace{A_n \text{ ist eine Induktionsmenge}}_{A(n)} \ \forall n \in \MdN$
\item Sei $n \in \MdN, x \in \MdR$ und $n<x<n+1$. Behauptung: $x \notin \MdN$. Beweis: Annahme: $x \in \MdN$. Sei $A_m$ wie im oberen Beispiel (2) \folgt $A_m \in J \folgt \MdN \subseteq A_m \folgt x \in A_m \folgt x \le m$ oder $x\ge m+1$, Widerspruch!
\item
\item
Behauptung: $\underbrace{1+2+\dots +n = \frac{n(n+1)}{2}}_{A(n)}\ \forall n\in \MdN$\\
\textbf{Beweis:} (induktiv)\\
IA: $\frac{1+1}{2}=1 \folgt A(1)$ ist wahr.\\
@ -294,7 +294,7 @@ $1+2+\dots +n+(n+1)
\folgt \text{A(n+1) ist wahr}$
\end{beispiele}
\indexlabel{Summenzeichen}
\indexlabel{Summenzeichen}
\indexlabel{Produktzeichen}
\begin{definition}[Summen- und Produktzeichen]
\begin{liste}
@ -395,7 +395,7 @@ Also $C:=\{b_1,b_2,\ldots\} \subseteq B$. $\forall x \in B \folgt x \in A \folgt
\begin{liste}
\item Für $a \in\MdR$ und $n\in\MdN$ gilt $a^n := a\cdot a\cdot a\cdot a\cdot \ldots \cdot a$ ($n$ Faktoren) und heißt die \indexlabel{Potenz!natürliche}\textit{$n$-te Potenz} von $a$\\
$a^0:=1$ \\
Für $a\ne 0$ gilt: $a^{-n}=\frac{1}{a^n}$
Für $a\ne 0$ gilt: $a^{-n}=\frac{1}{a^n}$
\item Für $n\in\MdN$ gilt $n! := 1\cdot 2\cdot 3\cdot \ldots \cdot n $ und heißt die \begriff{Fakultät} von $n$, $0! := 1$.
\item Für $n\in\MdN$, $k\in\MdN_{0}$ und $k\le n$ gilt $\binom{n}{k}:=\frac{n!}{k!(n-k)!}$ ("`n über k"')
\end{liste}
@ -554,7 +554,7 @@ $$ \mathop{\inf{a_n}}_{n=1}^{\infty} := \mathop{\inf{a_n}}_{n\in \MdN} := \matho
\begin{definition}[Konvergente Folge]
Sei $(a_n)$ eine Folge. $(a_n)$ heißt \begriff{konvergent} $:\equizu \exists a \in \MdR$, so dass es für \textit{jedes} $\varepsilon > 0$ ein $n_0 = n_0(\varepsilon) \in \MdN$ gibt, so dass $|a_n - a| < \varepsilon \ \forall n \ge n_0$ gilt. In diesem Fall heißt $a$ der \begriff{Grenzwert} (GW) oder \begriff{Limes} von $(a_n)$ und man schreibt: $\lim_{n \to \infty}(a_n) = a$ oder $\lim{a_n} = a$ oder $a_n \to a \ (n \to \infty)$ oder $a_n \to a$. Ist $(a_n)$ nicht konvergent, so heißt $(a_n)$ \begriff{divergent}.
\begin{eqnarray*}
\text{Also: } a_n \to a \ (n \to \infty)
\text{Also: } a_n \to a \ (n \to \infty)
&\equizu& \forall\varepsilon > 0 \ \exists n_0 = n_0(\varepsilon) \in\MdN: |a_n - a| < \varepsilon \ \forall n \ge n_0 \\
&\equizu& \forall\varepsilon > 0 \ \exists n_0 = n_0(\varepsilon) \in\MdN: a_n \in U_{\varepsilon}(a)\ \forall n \ge n_0 \\
&\equizu& \forall\varepsilon > 0 \text{ gilt: } a_n \in U_\varepsilon(a) \text{ \ffa } n \in \MdN.
@ -737,7 +737,7 @@ $a_n := \sqrt[n]{n} -1 \folgt a_n > 0 \ \forall n \in\MdN$. Zu zeigen ist: $a_n
Sei $c>0$. Dann: $\sqrt[n]{c} \to 1 \ (n\to\infty)$.
\end{wichtigesbeispiel}
\begin{beweis}
\begin{beweis}
Fall 1: $c\ge 1 \ \exists m \in\MdN: m \ge c \folgt 1\le c\le n \ \forall n\ge m \folgt \sqrt[n]{n} \le \underbrace{\sqrt[n]{n}}_{\to 1} \folgtnach{7.4} \sqrt[n]{c} \to 1$ \\
Fall 2: $c<1 \folgt \frac{1}{c} > 1 \folgtnach{Fall 1} \underbrace{\sqrt[n]{\frac{1}{c}}}_{=\frac{1}{\sqrt[n]{c}}} \to 1 \folgtnach{6.2(vii)} \sqrt[n]{c} \to 1$
\end{beweis}
@ -789,12 +789,12 @@ $(a_n)$ sei eine Folge und $\alpha \in\MdR$. $\alpha$ heißt ein \begriff{Häufu
\end{definition}
\begin{beispiele}
\item $a_n = (-1)^n$. $a_{2n} = 1, a_{2n-1} = -1.$
Sei $\ep > 0: a_{2n} \in U_{\ep}(1)\ \forall n \in \mathbb{N} \Rightarrow a_n \in U_{\ep}(1)$
für unendlich viele $n \in \mathbb{N} \Rightarrow 1 \in H (a_n)$.
Analog: $a_n \in U_{\ep}(-1)$ für unendlich viele $n \in \mathbb{N} \Rightarrow -1 \in \H(a_n)$.
Sei $\alpha \in \mathbb{R}$ und 1 $\neq \alpha \neq -1$.
Wähle $\ep > 0$ so, dass $1, -1 \not\in U_{\ep}(\alpha) \Rightarrow a_n \not\in U_{\ep}(\alpha)\ \forall n \in \mathbb{N} \Rightarrow \alpha \not\in \H(a_{n})$.
\item $a_n = (-1)^n$. $a_{2n} = 1, a_{2n-1} = -1.$
Sei $\ep > 0: a_{2n} \in U_{\ep}(1)\ \forall n \in \mathbb{N} \Rightarrow a_n \in U_{\ep}(1)$
für unendlich viele $n \in \mathbb{N} \Rightarrow 1 \in H (a_n)$.
Analog: $a_n \in U_{\ep}(-1)$ für unendlich viele $n \in \mathbb{N} \Rightarrow -1 \in \H(a_n)$.
Sei $\alpha \in \mathbb{R}$ und 1 $\neq \alpha \neq -1$.
Wähle $\ep > 0$ so, dass $1, -1 \not\in U_{\ep}(\alpha) \Rightarrow a_n \not\in U_{\ep}(\alpha)\ \forall n \in \mathbb{N} \Rightarrow \alpha \not\in \H(a_{n})$.
Fazit: $\H(a_n) = \{1; -1\}$.
\item $a_n = n$. Sei $\alpha \in \mathbb{R}$ und $\varepsilon > 0$. $\exists n_0 \in \mathbb{N}: n_0 > \alpha + \ep \Rightarrow n > \alpha + \ep\ \forall n \geq n_0 \Rightarrow a_n \not\in U_{\ep}(\alpha)\ \forall n \geq n_0 \Rightarrow a_n \in U_{\ep}(\alpha)$ für höchstens endlich viele $n \in \mathbb{N}$. $\Rightarrow \alpha \not\in \H(a_n)$. Fazit: $\H(a_n) = \emptyset$.
\item $\mathbb{Q}$ ist abzählbar. Also: $\mathbb{Q} = \{a_1, a_2, \ldots\}$.\\
@ -853,7 +853,7 @@ $(a_n)$ sei eine beschränkte Folge. Dann H$(a_n) \ne \emptyset$.
\end{satz}
\begin{beweis}
$\exists c>0: |a_n| \le c \ \forall n\in\MdN$. Hilfssatz $\folgt (a_n)$ enthält eine monotone Teilfolge $(a_{n_k})$. $|a_{n_k}| \le c \ \forall k \in\MdN$. $(a_{n_k})$ ist aber schränkt. 6.3 $\folgt (a_{n_k})$ ist konvergent. $\alpha := \lim_{k\to\infty}a_{n_k}$. 8.1(1) $\folgt \alpha \in \H(a_n)$.
$\exists c>0: |a_n| \le c \ \forall n\in\MdN$. Hilfssatz $\folgt (a_n)$ enthält eine monotone Teilfolge $(a_{n_k})$. $|a_{n_k}| \le c \ \forall k \in\MdN$. $(a_{n_k})$ ist aber schränkt. 6.3 $\folgt (a_{n_k})$ ist konvergent. $\alpha := \lim_{k\to\infty}a_{n_k}$. 8.1(1) $\folgt \alpha \in \H(a_n)$.
\end{beweis}
\chapter{Oberer und unterer Limes}
@ -1075,13 +1075,13 @@ klar.
\end{beweis}
\begin{definition}
Die Reihe $\areihe$ heißt \begriff{absolut konvergent} $:\equizu \reihe{|a_n|}$ ist konvergent.
Die Reihe $\areihe$ heißt \begriff{absolut konvergent} $:\equizu \reihe{|a_n|}$ ist konvergent.
\end{definition}
\begin{satz}[Dreiecksungleichung für Reihen]
Ist $\areihe$ absolut konvergent, so ist $\areihe$ konvergent und
$$|\areihe| \le \reihe{|a_n|}$$
\
\
\end{satz}
\begin{beweis}
@ -1099,7 +1099,7 @@ Die \begriff{alternierende Harmonische Reihe} $\reihe{(-1)^{n+1} \frac{1}n}$.\\
Hier: $a_n = (-1)^{n+1}\frac{1}{n}$. $|a_n| = \frac{1}{n} \folgt \reihe{a_n}$ konvergiert nicht absolut.\\
\textbf{Behauptung:} $\reihe{a_n}$ ist konvergent. (Später: $\reihe{a_n} = \log 2$)\\
\textbf{Beweis:} $s_n = a_1 + a_2 + \ldots + a_n$. \\
$s_{2n+2} = s_{2n} + a_{2n+1} + a_{2n+2} = s_{2n} + \underbrace{\frac{1}{2n+1} - \frac{1}{2n+2}}_{>0} \folgt (s_{2n})$ ist monoton wachsend. Analog: $(s_{2n-1})$ ist monoton fallend.
$s_{2n+2} = s_{2n} + a_{2n+1} + a_{2n+2} = s_{2n} + \underbrace{\frac{1}{2n+1} - \frac{1}{2n+2}}_{>0} \folgt (s_{2n})$ ist monoton wachsend. Analog: $(s_{2n-1})$ ist monoton fallend.
$s_{2n} = s_{2n-1} + a_{2n} = s_{2n-1} - \frac{1}{2n}$ $(*)$\\
Dann gilt $s_2 \le s_4 \le \ldots \le s_{2n} \gleichwegen{(*)} s_{2n-1} - \frac{1}{2n} < s_{2n-1} \le \ldots \le s_3 \le s_1 \folgt (s_{2n})$ und $(s_{2n-1})$ sind beschränkt. 6.3 $\folgt$ $(s_{2n})$ und $(s_{2n-1})$ sind konvergent. Aus $(*)$ folgt dann $\lim s_{2n} = \lim s_{2n-1}$. A16 $\folgt$ $(s_n)$ hat genau einen Häufungswert. 9.3 $\folgt$ $(s_n)$ ist konvergent.
\end{beispiel}
@ -1155,7 +1155,7 @@ Sei $(a_n)$ eine Folge und $\alpha := \limsup \sqrt[n]{|a_n|}$.
\begin{beweise}
\item $\alpha < 1 $. Sei $\ep>0$ so, dass $x:= \alpha+\ep<1$. 9.2 $\folgt \sqrt[n]{|a_n|} < \alpha + \ep = x \ffa n\in\MdN \folgt |a_n| < x^n \ffa n\in\MdN$. $\reihe{x^n}$ ist konvergent $\folgtnach{12.1(1)}$ Behauptung.
\item
\item
\begin{liste}
\item $\alpha>1$, $\alpha<\infty$: Sei $\ep>0$ so, dass $\alpha-\ep>1$. 9.2 $\folgt \sqrt[n]{|a_n|}>\alpha-\ep>1$ für unendlich viele $n\in\MdN$ \folgt $|a_n|>1$ für unendlich viele $n\in\MdN$ \folgt $a_n \to 0 \folgtnach{11.1} \reihe{a_n}$ ist divergent.
\item $\alpha = \infty \folgt \sqrt[n]{|a_n|} > 1$ für unendlich viele \natn \folgtnach{wie eben} $\reihe{a_n}$ ist divergent.
@ -1194,7 +1194,7 @@ Sei $(a_n)$ eine Folge in $\MdR$ und $a_n \ne 0 \ffa \natn$. $\alpha_n := \frac{
O.B.d.A.: $a_n \ne 0 \ \forall\natn$
\begin{liste}
\item Dann: $|a_2|\ge |a_1|>0$, $|a_3|\ge|a_2|\ge|a_1|>0$, \ldots allgemein: $|a_n|\ge|a_1|>0\ \forall\natn \folgt a_n\nrightarrow 0 \folgt$ die Behauptung.
\item
\item
\begin{liste}
\item Sei $\beta >1$, Sei $\ep>0$ so, dass $\beta-\ep>1$. 9.2 $\folgt |\alpha_n|>\beta-\ep>1 \ffa n\in\MdN \folgt$ die Behauptung.
\item Sei $\alpha < 1$. Sei $\ep>0$ so, dass $x:=\alpha+\ep<1$. 9.2 $\folgt |\alpha_n|<\alpha+\ep=x \ffa\natn$. Dann: $|a_2|\le|a_1|x$, $|a_3|\le|a_2|x\le|a_1|x^2$,\ldots allgemein: $|a_n|\le|a_n1|x^{n-1} \ffa\natn$. $\reihe{|a_1|x^{n-1}}$ ist konvergent \folgtnach{12.2} $\reihe{a_n}$ ist absolut konvergent.
@ -1499,7 +1499,7 @@ $\displaystyle{a_n := (-1)^n\cdot\frac{x^{2n}}{(2n)!} \folgt \sqrt[n]{|a_n|} = \
\begin{definition}[Potenzreihe]
Sei $(a_n)_{n=0}^{\infty}$ eine Folge in \MdR. Eine Reihe der Form $\sum_{n=0}^{\infty}\
{a_nx^n} = {\nobreak a_0 + a_1x + a_2x^2 + \ldots}$ heißt eine \begriff{Potenzreihe} (PR). Die Menge
{a_nx^n} = {\nobreak a_0 + a_1x + a_2x^2 + \ldots}$ heißt eine \begriff{Potenzreihe} (PR). Die Menge
$\{x \in\MdR : \reihenull{a_nx^n}$ konvergent$\}$ heißt der \begriff{Konvergenzbereich} (KB) der Potenzreihe. Klar: Die Potenzreihe konvergiert für $x=0$.
\end{definition}
@ -1541,13 +1541,13 @@ $(\sqrt[n]{|a_nx^n|})=(\sqrt[n]{|a_n|}|x|) \folgt\ (\sqrt[n]{|a_nx^n|})$ ist nic
n2^n&\text{n ungerade}\end{cases}$. A16 \folgt $\H(\sqrt[n]{|a_n|})=\{0, 2\}
\folgt \rho=2 \folgt r=\frac{1}{2}$. Die Potenzreihe konvergiert absolut für $|x|<\frac{1}{2}$, sie divergiert für $|x|>\frac{1}{2}$. Sei $|x|=\frac{1}{2}$. $|a_nx^n|=|a_n|\frac{1}{2^n}=n$ falls $n$ ungerade \folgt $a_nx^n \nrightarrow 0 \folgt$ die Potenzreihe divergiert für $|x|=\frac{1}{2}$.
\end{beispiele}
Die folgenden Potenzreihen haben jeweils den Konvergenzradius $r=\infty:$\\
Die folgenden Potenzreihen haben jeweils den Konvergenzradius $r=\infty:$\\
$e^x=\reihenull{\frac{x^n}{n!},\ \sin x=\reihenull{(-1)^n\frac{x^{2n+1}}{(2n+1)!}}},\\ \cos x = \reihenull{(-1)^n\frac{x^{2n}}{(2n)!}},\ f'(x)=\reihenull{a_nnx^{n-1}}$, falls $f(x)=\reihenull{a_nx^n}$ KR $r=\infty$ hat.
\begin{definition}
$\cosh x:=\frac{1}{2}(e^x+e^{-x})\ (x\in\MdR)$ (Cosinus Hyperbolikus)\\
$\sinh x:=\frac{1}{2}(e^x-e^{-x})\ (x\in\MdR)$ (Sinus Hyperbolikus)\\
Nachrechnen: $\cosh x=\reihenull{\frac{x^{2n}}{(2n)!}},
Nachrechnen: $\cosh x=\reihenull{\frac{x^{2n}}{(2n)!}},
\sinh x=\reihenull{\frac{x^{2n+1}}{(2n+1)!}} (x\in\MdR)$
\end{definition}
@ -1571,7 +1571,7 @@ Sei $x \in (-R, R): (\reihenull{a_nx^n})(\reihenull{b_nx^n})\overset{\text{13.4}
\end{beweis}
\begin{bemerkung}
Sei $(a_n)^{\infty}_{n=0}$ eine Folge und $x_0 \in\MdR$. Eine Reihe der Form $(*) \reihenull{a_n(x-x_0)^n}$ heißt ebenfalls eine Potenzreihe ($x_0$ heißt \begriff{Entwicklungspunkt} der Potenzreihe). Substitution $t:=x-x_0$, dann erhält man die Potenzreihe $\reihenull{a_nt^n}$. Sei $r$ der Konvergenzradius dieser Potenzreihe. Dann: ist $r=0$, so konvergiert die Potenzreihe in $(*)$ \emph{nur} in $x=x_0$. Ist $r=\infty$, so konvergiert die Potenzreihe absolut $\forall\ x\in\MdR.$ Ist $0<r<\infty$, so konvergiert die Potenzreihe in $(*)$ absolut für $|x-x_0|<r$, sie divergiert für $|x-x_0|>r.$
Sei $(a_n)^{\infty}_{n=0}$ eine Folge und $x_0 \in\MdR$. Eine Reihe der Form $(*) \reihenull{a_n(x-x_0)^n}$ heißt ebenfalls eine Potenzreihe ($x_0$ heißt \begriff{Entwicklungspunkt} der Potenzreihe). Substitution $t:=x-x_0$, dann erhält man die Potenzreihe $\reihenull{a_nt^n}$. Sei $r$ der Konvergenzradius dieser Potenzreihe. Dann: ist $r=0$, so konvergiert die Potenzreihe in $(*)$ \emph{nur} in $x=x_0$. Ist $r=\infty$, so konvergiert die Potenzreihe absolut $\forall\ x\in\MdR.$ Ist $0<r<\infty$, so konvergiert die Potenzreihe in $(*)$ absolut für $|x-x_0|<r$, sie divergiert für $|x-x_0|>r.$
\end{bemerkung}
\chapter{$g$-adische Entwicklungen}
@ -1619,7 +1619,7 @@ Sei $a=0,z_1z_2z_3\ldots$ ein $g$-adischer Bruch.
\begin{beweise}
\item $0 \le a = \reihe{\frac{z_n}{g^n}} \stackrel{\text{($*$)}, Bem. (2)}{<} \reihe{\frac{g-1}{g^n}} \gleichnach{15.1} 1$.
\item \textbf{Annahme:} $\exists n\in\MdN: z_n \ne w_n$. Sei $m$ der kleinste solche Index, also $z_m \ne w_m$ und $z_j = w_j$ für $j=1,\ldots ,m-1$. Etwa $z_m < w_m \folgt z_m - w_m < 0 \overset{z_m - w_m \in\MdZ}{\folgt} z_m - w_m \le -1$. $\forall n\in\MdN: z_n - w_n \le z_n \le g-1$. $\exists \nu \in\MdN$ mit $\nu \ge m+1$ und $z_\nu - w_\nu < g-1$. (andererenfalls $z_\nu - w_\nu = g-1 \ \forall \nu \ge m+1 \folgt z_\nu = w_\nu + g-1 \ \forall \nu \ge m+1 \folgt w_\nu = 0 \ \forall \nu \ge m+1 \folgt z_\nu = g-1 \ \forall \nu \ge m+1$. Widerspruch zu $(*)$). Dann:
\item \textbf{Annahme:} $\exists n\in\MdN: z_n \ne w_n$. Sei $m$ der kleinste solche Index, also $z_m \ne w_m$ und $z_j = w_j$ für $j=1,\ldots ,m-1$. Etwa $z_m < w_m \folgt z_m - w_m < 0 \overset{z_m - w_m \in\MdZ}{\folgt} z_m - w_m \le -1$. $\forall n\in\MdN: z_n - w_n \le z_n \le g-1$. $\exists \nu \in\MdN$ mit $\nu \ge m+1$ und $z_\nu - w_\nu < g-1$. (andererenfalls $z_\nu - w_\nu = g-1 \ \forall \nu \ge m+1 \folgt z_\nu = w_\nu + g-1 \ \forall \nu \ge m+1 \folgt w_\nu = 0 \ \forall \nu \ge m+1 \folgt z_\nu = g-1 \ \forall \nu \ge m+1$. Widerspruch zu $(*)$). Dann:
$\displaystyle{0 = a-a = \reihe{\frac{z_n}{g^n}} - \reihe{\frac{w_n}{g^n}} = \reihe{\frac{z_n - w_n}{g^n}} = \sum_{n=m}^\infty{\frac{z_n - w_n}{g^n}}}$\\
$\displaystyle{= \underbrace{\frac{z_m - w_m}{g^m}}_{\le -\frac{1}{g^m}} + \underbrace{\sum_{n=m+1}^\infty{\frac{z_n-w_n}{g^n}}}_{< \sum_{n=m+1}^\infty{\frac{g-1}{g^n}}} < - \frac{1}{g^m} + \underbrace{\sum_{n=m+1}^\infty{\frac{g-1}{g^n}}}_{\gleichnach{15.1} \frac{1}{g^n}} = 0}$\\
$\folgt 0<0 \text{ Widerspruch.}$
@ -1651,7 +1651,7 @@ Die Menge der reellen Zahlen ist überabzählbar.
\begin{beweis}
Es genügt zu zeigen: $[0,1)$ ist überabzählbar.
\textbf{Annahme}: $[0,1)$ ist abzählbar, also $[0,1) = \{a_1,a_2,\ldots\}, a_j \ne a_k$ für $j\ne k$.
\textbf{Annahme}: $[0,1)$ ist abzählbar, also $[0,1) = \{a_1,a_2,\ldots\}, a_j \ne a_k$ für $j\ne k$.
Für $j \in\MdN$ sei $a_j = 0,z_1^{(j)} z_2^{(j)} z_3^{(j)}\ldots$ die 3-adische Entwicklung von $a_j$. ($z_k^{(j)} \in \{0,1,2\}$).
$$ z_k := \begin{cases} 1 & \text{falls } z_k^{(k)} \in \{0,2\} \\0 & \text{falls } z_k^{(k)}= 1 \end{cases}$$
Dann: $z_k \ne z_k^{(k)} \ \forall k \in\MdN$, $z_k \ne g-1 \ \forall k\in\MdN$. $a := 0,z_1z_2z_3\ldots = \reihe{\frac{z_n}{g^n}}.$ 15.2 $\folgt a \in [0,1) \folgt \exists m\in\MdN: a= a_m \folgt 0,z_1z_2z_3\ldots = 0,z_1^{(m)}z_2^{(m)}z_3^{(m)}\ldots$. 15.2 $\folgt z_j = z_j^{(m)} \ \forall j\in\MdN \folgt z_m = z_m^{(m)}$. Widerspruch!
@ -1712,7 +1712,7 @@ Für $\delta >0$: $D_\delta(x_n) = D \cap U_\delta(x_0)$. $\dot D_\delta(x_0) =
\begin{satz}[Grenzwertsätze bei Funktionen]
\begin{liste}
\item $\displaystyle\lim_{x\to x_0}f(x)$ existiert $\equizu$ für jede Folge $(x_n)$ in $D\backslash\{x_0\}$ mit $x_n \to x_0$ ist $f(x_n)$ konvergent.
\item Für $a\in\MdR$ gilt: $\displaystyle\lim_{x\to x_0}f(x)$ existiert und ist gleich $a$ $\equizu$ $\forall \ep>0 \ \exists \delta(\ep) > 0$ mit $(*)$ $|f(x) - a|< \ep \ \forall x \in \dot D_\delta(x_0)$.
\item Für $a\in\MdR$ gilt: $\displaystyle\lim_{x\to x_0}f(x)$ existiert und ist gleich $a$ $\equizu$ $\forall \ep>0 \ \exists \delta(\ep) > 0$ mit $(*)$ $|f(x) - a|< \ep \ \forall x \in \dot D_\delta(x_0)$.
\item \textit{Cauchykriterium}\indexlabel{Cauchykriterium!bei Funktionsgrenzwerten}: $\displaystyle\lim_{x\to x_0} f(x)$ existiert $\equizu$ $\forall \ep>0 \ \exists \delta >0$: $|f(x) - f(x')|<\ep \forall x,x' \in \dot D_\delta(x_0)$
\end{liste}
\end{satz}
@ -1744,8 +1744,8 @@ Zum Beispiel: (3) Sei $(x_n)$ Folge in $D\backslash\{x_0\}$ und $x_n \to x_0$. D
\begin{definition}
\begin{liste}
\item Sei $(a_n)$ eine Folge in $\MdR$.\\
$\lim a_n = \infty$ (oder $a_n \to \infty$) $:\equizu \forall c>0 \ \exists n_0 = n_0(c)\in\MdN: a_n > c \forall n\ge n_0$.\\
$\lim a_n = -\infty$ (oder $a_n \to -\infty$) $:\equizu \forall c<0 \ \exists n_0 = n_0(c)\in\MdN: a_n < c \forall n\ge n_0$.
$\lim a_n = \infty$ (oder $a_n \to \infty$) $:\equizu \forall c>0 \ \exists n_0 = n_0(c)\in\MdN: a_n > c \forall n\ge n_0$.\\
$\lim a_n = -\infty$ (oder $a_n \to -\infty$) $:\equizu \forall c<0 \ \exists n_0 = n_0(c)\in\MdN: a_n < c \forall n\ge n_0$.
\item $\displaystyle\lim_{x\to x_0} f(x) = \infty$ (oder $f(x) \to \infty\ (x\to x_0)$) $:\equizu$ für jede Folge $(x_n)$ in $D\backslash\{x_0\}$ und $x_n \to x_0$ gilt: $f(x_n) \to \infty$. \\
$\displaystyle\lim_{x\to x_0} f(x) = -\infty$ (oder $f(x) \to -\infty\ (x\to x_0)$) $:\equizu$ für jede Folge $(x_n)$ in $D\backslash\{x_0\}$ und $x_n \to x_0$ gilt: $f(x_n) \to -\infty$.
\item Sei $D$ nicht nach oben beschränkt. $\displaystyle\lim_{x\to \infty} f(x) = a$ (oder $f(x) \to a$) $:\equizu$ für jede Folge $(x_n)$ in $D$ mit $x_n\to \infty$ gilt: $f(x_n) \to a$ ($a = \pm\infty$ zugelassen). \\
@ -1896,8 +1896,8 @@ $B \subseteq \MdR$ heißt \indexlabel{offene Menge}\textbf{offen} $:\equizu \for
\begin{beweis}
\begin{liste}
\item Übung
\item "`\folgt "': Sei $(x_n)$ eine konvergente Folge in $\MdR\ \backslash\ B$ und $x_0:=\lim x_n$.Annahme: $x_0 \in B$. B offen $\folgt \exists \delta>0 : U_\delta(x_0) \subseteq B$. $x_n \to x_0 \folgt x_n \in U_\delta(x_0) \subseteq B \ffa n \in\MdN$, Widerspruch! "`$\Leftarrow$ "': Sei $x \in B$. Annahme: $U_\delta(x) \nsubseteq B \forall \delta>0$.
\folgt $U_{\frac{1}{n}}(x) \nsubseteq B \forall n \in \MdN
\item "`\folgt "': Sei $(x_n)$ eine konvergente Folge in $\MdR\ \backslash\ B$ und $x_0:=\lim x_n$.Annahme: $x_0 \in B$. B offen $\folgt \exists \delta>0 : U_\delta(x_0) \subseteq B$. $x_n \to x_0 \folgt x_n \in U_\delta(x_0) \subseteq B \ffa n \in\MdN$, Widerspruch! "`$\Leftarrow$ "': Sei $x \in B$. Annahme: $U_\delta(x) \nsubseteq B \forall \delta>0$.
\folgt $U_{\frac{1}{n}}(x) \nsubseteq B \forall n \in \MdN
\folgt \forall n \in \MdN \exists x_n \in U_{\frac{1}{n}}$ mit: $x_n \in \MdR\ \backslash\ B \folgt (x_n)$ ist eine Folge in $\MdR\ \backslash\ B: x_n \to x$. $\MdR\ \backslash\ B$ abgeschlossen \folgt $x \in \MdR\ \backslash\ B$, Widerspruch!
\item "`\folgt "': Sei $(x_n)$ Folge in $D$. $D$ beschränkt \folgt $(x_n)$ beschränkt. 8.2 \folgt $(x_n)$ enthält eine konvergente Teilfolge $(x_{n_k})$. $D$ abgeschlossen $\folgt \lim x_{n_k} \in D$. "`$\Leftarrow$ "': Übung. Sei $D$ beschränkt und abgeschlossen. Sei $s:=\sup D$. z.z.: $s \in D$ (analog zeigt man $\inf D \in D$). $\forall n \in \MdN$ ist $s-\frac{1}{n}$ keine obere Schranke von s. \folgt $\forall n \in\MdN\exists\ x_n \in D$ mit $s - \frac{1}{n} < x_n \le s \folgt x_n \to s$. D abgeschlossen \folgt $s \in D$
\end{liste}
@ -1933,7 +1933,7 @@ Sei $I \subseteq \MdR$ ein Intervall und $f \in C(I)$.
\end{beweis}
\begin{satz}[Der Logarithmus]
Sei $I=\MdR$ und $f(x)=e^x$. Bekannt: $f \in C(\MdR)$, f ist streng monoton wachsend und $f(I) = f(\MdR) = (0, \infty)$. Also existiert $f^{-1}: (0, \infty) \to \MdR$.
Sei $I=\MdR$ und $f(x)=e^x$. Bekannt: $f \in C(\MdR)$, f ist streng monoton wachsend und $f(I) = f(\MdR) = (0, \infty)$. Also existiert $f^{-1}: (0, \infty) \to \MdR$.
\[ \log x := \ln x := f^{-1}(x)\ (x \in (0, \infty))\ \text{\emph{Logarithmus}} \]
\end{satz}
@ -2005,7 +2005,7 @@ $(f_n)$ konvergiert punktweise auf $D$ gegen $f$.
Konvergiert $(f_n)$ auf $D$ punktweise gegen $f:D\to\MdR$, so bedeutet dies: Ist $\ep>0$ und $x\in D$, so existiert ein $n_0 = n_0(\ep,x)\in\MdN$: $|f_n(x)-f(x)|<\ep \ \forall n\ge n_0$
\begin{definition}
$(f_n)$ heißt auf $D$ \begriff{gleichmäßig konvergent}
$(f_n)$ heißt auf $D$ \begriff{gleichmäßig konvergent}
$:\equizu \exists \text{ Funktion } f:D\to\MdR$ für die gilt:\\
\hspace*{10mm} $\forall \ep>0 \ \exists n_0 = n_0(\ep) \in\MdN \ \forall n\ge n_0 \ \forall x\in D\colon |f_n(x) - f(x)|<\ep$.
@ -2475,24 +2475,24 @@ Dann: $\sum_{k=0}^\infty \frac{f^{(k)}(0)}{k!}x^k = 0 \ne f(x) \ \forall x\in\Md
\begin{definition}
Sei $n\in\MdN_0$, $f\in C^n(I)$ und $x_0 \in I$.\\
$T_n(x;x_0) := \sum_{k=0}^n \frac{f^{(k)}(x_0)}{k!}(x-x_0)^k$
$T_n(x;x_0) := \sum_{k=0}^n \frac{f^{(k)}(x_0)}{k!}(x-x_0)^k$
heißt das \begriff{k-te Taylorpolynom} von $f$ vom Grad $\leq k$.
\end{definition}
\begin{eigenschaftenNoCounter}
\begin{enumerate}
\item $p$ ist ein Polynom vom Grad $\leq n$ und es gilt:
\item $p$ ist ein Polynom vom Grad $\leq n$ und es gilt:
$p^{(k)}(x_0) = f^{(k)}(x_0)$ für $k=0, 1, \dots, n$
\item Ist $q$ ein Polynom vom Grad $\leq n$ und gilt $q^{(k)} (x_0) = f^{(k)} (x_0)$
für $k=0, 1, \dots, n$, so ist $p=q$.
\item Ist $f \in C^\infty(1)$, so ist $T_n(x, x_0)$ die n-te
\item Ist $f \in C^\infty(1)$, so ist $T_n(x, x_0)$ die n-te
Teilsumme der Taylorreihe $f$ (in $x_0$).
\end{enumerate}
\end{eigenschaftenNoCounter}
\begin{satz}[Satz von Taylor]
Voraussetzungen wie in obiger Definition. Weiter sei $f\ (n+1)$-mal
differenzierbar auf $I$ und $x\in I$. Dann existiert ein $\xi$
Voraussetzungen wie in obiger Definition. Weiter sei $f\ (n+1)$-mal
differenzierbar auf $I$ und $x\in I$. Dann existiert ein $\xi$
zwischen $x$ und $x_0$ mit:
\[ f(x) = T_n(x;x_0) + \underbrace{\frac{f^{(n+1)}(\xi)}{(n+1)!}(x-x_0)^{n+1}}_\text{Restglied nach Lagrange} \]
\end{satz}
@ -2501,7 +2501,7 @@ zwischen $x$ und $x_0$ mit:
Ohne Beschränkung der Allgemeinheit sei $x_0 = 0$ und $x>x_0$.\\
$\rho := (f(x) - T_n(x;0)) \frac{(n+1)!}{x^{n+1}} \folgt f(x) - T_n(x;0) = \frac{\rho}{(n+1)!}x^{n+1}$\\
Zu zeigen ist: $\exists \xi\in[0,x]: \rho = f^{(n+1)}(\xi).$ \\
Definiere $h: [0,x]\to \MdR$ durch $f(x) = f(x) - \sum_{k=0}^n \frac{f^{(k)}(t)}{k!} (x-t)^k - \rho\frac{(x-t)^{n+1}}{(n+1)!}$.
Definiere $h: [0,x]\to \MdR$ durch $f(x) = f(x) - \sum_{k=0}^n \frac{f^{(k)}(t)}{k!} (x-t)^k - \rho\frac{(x-t)^{n+1}}{(n+1)!}$.
Nachrechnen: $h(0) = h(x)$ und $h'(t) = \rho\frac{(x-t)^n}{n!} - \frac{f^{(n+1)}(t)}{n!}(x-t)^n$. \\
$0 = \frac{h(x)-h(0)}{x-0} \gleichnach{MWS} h'(\xi) \ \xi \in (0,x) \folgt \rho\frac{(x-\xi)^n}{n!} = \frac{f^{(n+1)}(\xi)}{n!}(x-\xi)^n \folgt \rho = f^{(n+1)}(\xi)$.
\end{beweis}
@ -2684,16 +2684,16 @@ $f \in R[a,b] \equizu \forall \ep>0\ \exists Z \in \Z: S_f(Z)-s_f(Z) < \ep$.
\begin{beweise}
\item O.B.d.A: $f$ ist wachsend auf $[a,b]$.\\
Sei $n\in\MdN$ und $Z_n=\{x_0,\cdots,x_n\}$ sei die
\begriff{äquidistante Zerlegung} von $[a,b]$ mit $n+1$
Teilpunkten. $x_j = a+j\frac{b-a}{n}\ (j=0,\cdots,n)$.
Dann: $|I_j|=\frac{b-a}{n}$. $m_j, M_j$ wie immer:
Sei $n\in\MdN$ und $Z_n=\{x_0,\cdots,x_n\}$ sei die
\begriff{äquidistante Zerlegung} von $[a,b]$ mit $n+1$
Teilpunkten. $x_j = a+j\frac{b-a}{n}\ (j=0,\cdots,n)$.
Dann: $|I_j|=\frac{b-a}{n}$. $m_j, M_j$ wie immer:
$S_f(Z_n)-s_f(Z_n)=\sum^n_{j=1}(\underbrace{M_j}_{=f(x_j)}-\underbrace{m_j}_{f(x_{j-1})})|I_j|=\sum_{j=1}^n(f(x_j)_-f(x_{j-1}))\frac{b-a}{n}=\frac{b-a}{n}(f(x_1)-f(x_0)+f(x_2)-f(x_1)+\cdots+f(x_n)-f(x_{n-1}))=\frac{b-a}{n}(f(x_n)-f(x_0))=\frac{b-a}{n}(f(b)-f(a))=:\alpha_n$. Sei $\ep>0$, dann: $\exists n\in\MdN: \alpha_n <\ep\folgtnach{23.3}$Behauptung.
\item Sei $f \in C[a, b]$ und $\ep>0$. $\exists \delta>0: (*)\ |f(t)-f(s)|<\frac{\ep}{b-a}\ \forall t,s\in[a,b]$ mit $|t-s|<\delta$. Sei $Z=\{x_0,\cdots,x_n\}\in\Z\ m_j,\ M_j,\ |I_J|$ seien wie immer; $z$ sei so gewählt, da"s $|I_j|<\delta\ (j=1,\cdots,n)$. Betrachte $I_j:$ 18.3$\folgt\ \exists s_j, t_j \in I_j: m_j=f(s_j),\ M_j=f(t_j)$. $|t_j-s_j|<\delta \folgtwegen{(*)}\underbrace{f(t_j)-f(s_j)}_{=M_j-m_j}<\frac{\ep}{b-a}\folgt S_f(Z)-s_f(Z)=\sum^n_{j=1}(\underbrace{M_j-m_j}_{\le\frac{\ep}{b-a}})|I_j|<\frac{\ep}{b-a}\sum^n_{j=1}|I_j|=\ep\folgtnach{23.3}f\in R[a,b]$
\end{beweise}
\begin{definition}
Sei $J\subseteq\MdR$ ein Intervall und $G,g: J\to\MdR$ seien Funktionen.
Sei $J\subseteq\MdR$ ein Intervall und $G,g: J\to\MdR$ seien Funktionen.
$G$ heißt eine \begriff{Stammfunktion} von $g$ auf $J$ :$\equizu$ G ist auf $J$ differenzierbar und $G'=g$ auf $J$.\\
\end{definition}
\textbf{Beachte:}
@ -2715,7 +2715,7 @@ Sei $[a,b]=[-1,1]$, $f(x):=\begin{cases}
\end{liste}
\begin{satz}[1. Hauptsatz der Differential- und Integralrechnung]
Es sei $f\in R[a,b]$ und $f$ besitze auf $[a,b]$ die Stammfunktion $F$. Dann:
Es sei $f\in R[a,b]$ und $f$ besitze auf $[a,b]$ die Stammfunktion $F$. Dann:
$$\int_a^bf(x)dx = F(b) - F(a) =: F(x)|_a^b =: [F(x)]_a^b$$
\end{satz}
@ -2738,19 +2738,19 @@ Aber: $\lim_{n\to\infty}\int_0^1f_ndx = 1 \ne 0 = \int_0^1fdx = \int_0^1(\lim_{n
\end{beispiele}
\begin{satz}[Integrierbarkeit gleichmäßig konvergierender Funktionsfolgen]
$(f_n)$ sei eine Folge in $R[a,b]$ und $(f_n)$ konvergiert auf $[a,b]$ \emph{gleichmäßig} gegen $f:[a,b]\to\MdR$. Dann ist $f\in R[a,b]$ und
$(f_n)$ sei eine Folge in $R[a,b]$ und $(f_n)$ konvergiert auf $[a,b]$ \emph{gleichmäßig} gegen $f:[a,b]\to\MdR$. Dann ist $f\in R[a,b]$ und
$$\lim_{n\to\infty}\int_a^bf_n(x)dx = \int_a^bfdx = \int_a^b(\lim_{n\to\infty} f_n)dx$$
$(f_n)$ sei eine Folge in $R[a,b]$ und $\sum_{n=1}^{\infty}f_n$ konvergiert auf $[a,b]$ \emph{gleichmäßig} gegen $f:[a,b]\to\MdR$. Dann ist $f\in R[a,b]$ und
$(f_n)$ sei eine Folge in $R[a,b]$ und $\sum_{n=1}^{\infty}f_n$ konvergiert auf $[a,b]$ \emph{gleichmäßig} gegen $f:[a,b]\to\MdR$. Dann ist $f\in R[a,b]$ und
$$\sum_{n=1}^\infty \int_a^bf_n(x)dx = \int_a^b \sum_{n=1}^\infty f_n(x)dx $$
\end{satz}
\begin{beweis}
1. Zu $\ep=1 \ \exists m \in \MdN$: $f_m-1<f<f_m+1$ auf $[a,b]$. $f_n$ beschränkt auf $[a,b]$. \\
2. $A_n := \int_a^bf_ndx$ $(n\in\MdN)$. Sei $\ep>0$. $\exists n_0\in\MdN: f_n-\ep<f<f_n+\ep$ auf $[a,b] \ \forall n\ge n_0 \folgt$ für $n\ge n_0$ folgt (wie im Beweis von 23.2(1)):
2. $A_n := \int_a^bf_ndx$ $(n\in\MdN)$. Sei $\ep>0$. $\exists n_0\in\MdN: f_n-\ep<f<f_n+\ep$ auf $[a,b] \ \forall n\ge n_0 \folgt$ für $n\ge n_0$ folgt (wie im Beweis von 23.2(1)):
$$\underbrace{\uint_a^b(f_n-\ep)dx}_{=A_n-\ep(b-a)} \le \underbrace{\uint_a^bfdx}_{=: A} \le \underbrace{\oint_a^bfxds}_{=: B} \le \underbrace{\oint_a^b(f_n+\ep)dx}_{=A_n+\ep(b-a)}$$
$\folgt |A_n - A| \le \ep(b-a)$, $|A_n -B|\le \ep (b-a)$ \\
$\forall n\in n_0 \folgt A_n \to A, A_n \to B \ (n\to\infty) \folgt A = B $ \\
$\folgt |A_n - A| \le \ep(b-a)$, $|A_n -B|\le \ep (b-a)$ \\
$\forall n\in n_0 \folgt A_n \to A, A_n \to B \ (n\to\infty) \folgt A = B $ \\
$\folgt f\in R[a,b]$ und $A_n \to \int_a^bfdx$
\end{beweis}
@ -2833,7 +2833,7 @@ $f,g: [a,b] \to \MdR$ seien Funktionen.
Sei $\ep > 0.$ Wähle $\alpha \in (a,b)$ mit $2\gamma(\alpha-a) < \ep/2.$
$f \in C[\alpha,b] \folgt f \in R[\alpha,b] \folgtnach{23.3}$ Es gibt eine Zerlegung $Z_1$ von $[\alpha,b]$ mit:\\
$f \in C[\alpha,b] \folgt f \in R[\alpha,b] \folgtnach{23.3}$ Es gibt eine Zerlegung $Z_1$ von $[\alpha,b]$ mit:\\
$S_f(Z_1) - s_f(Z_1) < \ep/2.\ Z:=Z_1 \cup \{a\} \folgt Z \in \Z$ und es gilt:\\
\begin{align*}
S_f(Z) - s_f(Z) &= \underbrace{\sup f([a,\alpha]) - \inf f([a,\alpha]))(\alpha-1)}_{\le 2 \gamma} + \underbrace{S_f(Z_1)-s_f(Z_1)}_{< \ep/2}\\
@ -2927,7 +2927,7 @@ Es ist $\frac{F(x_0+h)-F(x_0)}{h}\gleichnach{s.o.}\frac{1}{h}\int_{x_0}^{x_0+h}f
\end{beweise}
\begin{satz}[Anwendung des 2. Hauptsatzes auf stetige Funktionen]
Sei $J \subseteq \MdR$ ein beliebiges Intervall, $f \in C(J)$ und $\xi \in J$ (fest). $F:J\to\MdR$ sei definiert durch $F(x):=\int_{\xi}^xf(t)\dt$. Dann ist $F\in C^1(J)$ und $F'=f$ auf $J$.
Sei $J \subseteq \MdR$ ein beliebiges Intervall, $f \in C(J)$ und $\xi \in J$ (fest). $F:J\to\MdR$ sei definiert durch $F(x):=\int_{\xi}^xf(t)\dt$. Dann ist $F\in C^1(J)$ und $F'=f$ auf $J$.
\end{satz}
@ -3216,7 +3216,7 @@ $\folgt \exists c \in (1,\infty): \frac{f(x)}{g(x)} \ge \frac{1}{2}\ \forall x \
\begin{definition}
Sei $f:[a,b]\to\MdR$ und $Z=\{x_0,\ldots,x_n\} \in\Z$. $V_f(Z):=\sum_{j=1}^n|f(x_j)-f(x_{j-1})|$ ist die \begriff{Variation} von $f$ bezüglich Z.\\
\textbf{Beachte}: Sind $Z_1,Z_2 \in \Z$ und $Z_1 \subseteq Z_2\folgt V_f(Z_1) \le V_f(Z_2)$. $M_f=\{V_f(Z):Z \in \Z\}.\ f$ heißt von \begriff{beschränkter Variation}, in Zeichen: $f\in\BV[a,b]\ :\equizu M_f$ ist nach oben beschränkt. In diesem Fall heißt $V_f[a,b]:=\sup M_f$ die \begriff{Totalvariation} von $f$ (auf $[a,b]$).
\textbf{Beachte}: Sind $Z_1,Z_2 \in \Z$ und $Z_1 \subseteq Z_2\folgt V_f(Z_1) \le V_f(Z_2)$. $M_f=\{V_f(Z):Z \in \Z\}.\ f$ heißt von \begriff{beschränkter Variation}, in Zeichen: $f\in\BV[a,b]\ :\equizu M_f$ ist nach oben beschränkt. In diesem Fall heißt $V_f[a,b]:=\sup M_f$ die \begriff{Totalvariation} von $f$ (auf $[a,b]$).
\end{definition}
\begin{beispiel}
@ -3372,7 +3372,7 @@ Sei $g \in \BV[a,b]$ und $f \in R_{g}$. Dann: $${\left|\int_a^b fdg\right|}\le\g
\begin{beweis}
Sei $(Z, \xi) \in \Z^*, Z = \{x_0,\ldots,x_n\},\ \xi = (\xi_1,\ldots,\xi_n)$.\\
$|\sigma_f(Z,\xi, g)|=|\displaystyle\sum_{j=1}^nf(\xi_j)(g(x_j)-g(x_{j-1}))|\le\displaystyle\sum_{j=1}^n|f(\xi_j)||g(x_j)-g(x_{j-1})|\le\gamma V_g(Z)\le\gamma V_g[a, b]$
$|\sigma_f(Z,\xi, g)|=|\displaystyle\sum_{j=1}^nf(\xi_j)(g(x_j)-g(x_{j-1}))|\le\displaystyle\sum_{j=1}^n|f(\xi_j)||g(x_j)-g(x_{j-1})|\le\gamma V_g(Z)\le\gamma V_g[a, b]$
\end{beweis}
\begin{bezeichnungen}

File diff suppressed because it is too large Load diff

View file

@ -7,16 +7,16 @@
\semester{Wintersemeseter 10/11 und 12/13}
\scriptstate{complete}
\author{Die Mitarbeiter von \href{http://mitschriebwiki.nomeata.de/}{mitschriebwiki.nomeata.de}
\author{Die Mitarbeiter von \href{http://mitschriebwiki.nomeata.de/}{mitschriebwiki.nomeata.de}
und \href{https://github.com/MartinThoma/LaTeX-examples/tree/master/documents}{GitHub}}
\title{Analysis III - Bachelorversion}
\makeindex
\hypersetup{
pdfauthor = {Die Mitarbeiter von mitschriebwiki.nomeata.de und GitHub},
pdfkeywords = {Analysis},
pdftitle = {Analysis III}
}
\hypersetup{
pdfauthor = {Die Mitarbeiter von mitschriebwiki.nomeata.de und GitHub},
pdfkeywords = {Analysis},
pdftitle = {Analysis III}
}
\begin{document}
\maketitle
@ -29,10 +29,10 @@ und \href{https://github.com/MartinThoma/LaTeX-examples/tree/master/documents}{G
\chapter*{Vorwort}
\section*{Über dieses Skriptum}
Dies ist ein Mitschrieb der Vorlesung \glqq Analysis III\grqq\ von
Herrn Schmoeger im Wintersemester 2010 an der Universität Karlsruhe
(KIT). Die Mitschriebe der Vorlesung werden mit ausdrücklicher
Genehmigung von Herrn Schmoeger hier veröffentlicht, Herr Schmoeger
Dies ist ein Mitschrieb der Vorlesung \glqq Analysis III\grqq\ von
Herrn Schmoeger im Wintersemester 2010 an der Universität Karlsruhe
(KIT). Die Mitschriebe der Vorlesung werden mit ausdrücklicher
Genehmigung von Herrn Schmoeger hier veröffentlicht, Herr Schmoeger
ist für den Inhalt nicht verantwortlich.
Kapitel werden in Beweisen durch "`§"' abgekürzt.
@ -46,13 +46,13 @@ Im September 2012 wurde das Skript mit der Revisionsnummer 7132 von
mitschriebwiki auf \href{https://github.com/MartinThoma/LaTeX-examples/blob/master/documents/Analysis%20III}{GitHub} hochgeladen.
\section*{Wo}
Alle Kapitel inklusive \LaTeX-Quellen können unter
\href{http://mitschriebwiki.nomeata.de}{mitschriebwiki.nomeata.de}
Alle Kapitel inklusive \LaTeX-Quellen können unter
\href{http://mitschriebwiki.nomeata.de}{mitschriebwiki.nomeata.de}
abgerufen werden.
Dort ist ein \emph{Wiki} eingerichtet und von Joachim Breitner um die
Dort ist ein \emph{Wiki} eingerichtet und von Joachim Breitner um die
\LaTeX-Funktionen erweitert.
Das heißt, jeder kann Fehler nachbessern und sich an der Entwicklung
beteiligen. Auf Wunsch ist auch ein Zugang über \emph{Subversion}
beteiligen. Auf Wunsch ist auch ein Zugang über \emph{Subversion}
möglich.
Oder man geht auf \href{https://github.com/MartinThoma/LaTeX-examples/blob/master/documents/Analysis%20III/}{github},
@ -158,7 +158,7 @@ erstellt einen Fork und kann direkt Änderungen umsetzen.
\textbf{§ 10: Der Satz von Fubini}: Jan Ihrens\\
\textbf{§ 11: Der Transformationssatz}: Jan Ihrens, Rebecca Schwerdt\\
\textbf{§ 12: Vorbereitungen für die Integralsätze}: Rebecca Schwerdt\\
\textbf{§ 13: Der Integralsatz von Gauß\ im \(\mdr^{2}\)}: Benjamin Unger\\
\textbf{§ 13: Der Integralsatz von Gauß\ im \(\mdr^{2}\)}: Benjamin Unger\\
\textbf{§ 14: Flächen im \(\mdr^{3}\)}: Benjamin Unger\\
\textbf{§ 15: Der Integralsatz von Stokes}: Philipp Ost\\
\textbf{§ 16: \(\fl^{p}\)-Räume und \(\mathrm{L}^{p}\)-Räume}: Philipp Ost, Rebecca Schwerdt, Peter Pan, Jan Ihrens \\

View file

@ -1,17 +1,17 @@
In diesem Kapitel seien $X,Y,Z$ Mengen ($\ne\emptyset$) und
In diesem Kapitel seien $X,Y,Z$ Mengen ($\ne\emptyset$) und
$f: X\to Y,\; g:Y\to Z$ Abbildungen.
\begin{enumerate}
\index{Potenzmenge}
\index{Disjunktheit}
\item
\item
\begin{enumerate}
\item $\mathcal{P}(X):=\{A:A\subseteq X\}$ heißt
\item $\mathcal{P}(X):=\{A:A\subseteq X\}$ heißt
\textbf{Potenzmenge} von $X$.
\item Sei $\fm\subseteq\mathcal{P}(X)$, so heißt $\fm$
\textbf{disjunkt}, genau dann wenn $A\cap B=\emptyset$
\item Sei $\fm\subseteq\mathcal{P}(X)$, so heißt $\fm$
\textbf{disjunkt}, genau dann wenn $A\cap B=\emptyset$
für $A,B\in\fm$ mit $A\ne B$.
\item Sei $(A_j)$ eine Folge in $\mathcal{P}(X)$ (also
\item Sei $(A_j)$ eine Folge in $\mathcal{P}(X)$ (also
$A_j\subseteq X$), so heißt $(A_j)$ \textbf{disjunkt},
genau dann wenn $\{A_1,A_2,\dots\}$ disjunkt ist.\\
\textbf{Schreibweise}:\\
@ -22,16 +22,16 @@ $f: X\to Y,\; g:Y\to Z$ Abbildungen.
\sum_{j=1}^\infty a_j &=: \sum a_j
\end{align*}
\end{enumerate}
\item Sei $A\subseteq X$, $\mathds{1}_A : X \rightarrow R$
\item Sei $A\subseteq X$, $\mathds{1}_A : X \rightarrow R$
definiert durch:
\[\mathds{1}_A(x):= \begin{cases}
1 &\text{falls } x\in A\\
0 &\text{falls } x\in A^c
\end{cases}\]
wobei $A^c:=X\setminus A$. $\mathds{1}_A$ heißt die
\textbf{charakteristische Funktion} oder
\textbf{charakteristische Funktion} oder
\textbf{Indikatorfunktion von A}.
\item Sei $B\subseteq Y$ dann ist $f^{-1}(B):=\{x\in X: f(x)\in B\}$
\item Sei $B\subseteq Y$ dann ist $f^{-1}(B):=\{x\in X: f(x)\in B\}$
und es gelten folgende Eigenschaften:
\begin{enumerate}
\item $f^{-1}(B^c)=f^{-1}(B)^c$
@ -47,12 +47,12 @@ $f: X\to Y,\; g:Y\to Z$ Abbildungen.
\begin{definition}
\index{offen}
Sei $n \in \mdn$ und $\emptyset \neq X \subseteq \mdr^n$ und
Sei $n \in \mdn$ und $\emptyset \neq X \subseteq \mdr^n$ und
$A \subseteq X$.
$A$ heißt $\stackrel{\text{offen}}{\text{abgeschlossen}}$ in
$X :\Leftrightarrow \exists B \subseteq \mdr^n$.
$B$ ist $\stackrel{\text{offen}}{\text{abgeschlossen}}$ und
$A$ heißt $\stackrel{\text{offen}}{\text{abgeschlossen}}$ in
$X :\Leftrightarrow \exists B \subseteq \mdr^n$.
$B$ ist $\stackrel{\text{offen}}{\text{abgeschlossen}}$ und
$A = B \cap X$
\end{definition}
@ -61,18 +61,18 @@ $f: X\to Y,\; g:Y\to Z$ Abbildungen.
$f: X \rightarrow \mdr^n$.
\begin{enumerate}
\item $A$ ist offen in $X \Leftrightarrow \forall x \in A$
\item $A$ ist offen in $X \Leftrightarrow \forall x \in A$
ex. eine Umgebung $U$ von $x$ mit $U \cap X \subseteq A$
\item $A$ ist abgeschlossen in $X$\\
$\Leftrightarrow X \setminus A$ ist offen in $X$\\
$\Leftrightarrow$ für jede konvergente Folge $(a_k)$
$\Leftrightarrow$ für jede konvergente Folge $(a_k)$
in $A$ mit $\lim a_k \in X$ ist $\lim a_k \in A$
\item Die folgenden Aussagen sind äquivalent:
\begin{enumerate}
\item $f \in C(X, \mdr^m)$
\item für jede offene Menge $B \subseteq \mdr^m$ ist
\item für jede offene Menge $B \subseteq \mdr^m$ ist
$f^{-1}(B)$ offen in $X$
\item für jede abgeschlossene Menge $B \subseteq \mdr^m$ ist
\item für jede abgeschlossene Menge $B \subseteq \mdr^m$ ist
$f^{-1}(B)$ abgeschlossen in $X$
\end{enumerate}
\end{enumerate}

View file

@ -2,7 +2,7 @@ In diesem Kapitel sei $X \neq \emptyset$ eine Menge.
\begin{definition}
\index{$\sigma$-!Algebra}
Sei $\fa\subseteq\mathcal{P}(X)$, $\fa$ heißt eine
Sei $\fa\subseteq\mathcal{P}(X)$, $\fa$ heißt eine
\textbf{$\sigma$-Algebra} auf $X$, wenn gilt:
\begin{enumerate}
\item[($\sigma_1$)] $X\in\fa$
@ -14,11 +14,11 @@ In diesem Kapitel sei $X \neq \emptyset$ eine Menge.
\begin{beispieleX}
\begin{enumerate}
\item $\Set{X,\emptyset}$ und $\mathcal{P}(X)$ sind
\item $\Set{X,\emptyset}$ und $\mathcal{P}(X)$ sind
$\sigma$-Algebren auf $X$.
\item Sei $A\subseteq X$, dann ist $\Set{X,\emptyset, A, A^c}$
\item Sei $A\subseteq X$, dann ist $\Set{X,\emptyset, A, A^c}$
eine $\sigma$-Algebra auf $X$.
\item $\fa:=\Set{A\subseteq X | A \text{ abzählbar oder } A^c \text{ abzählbar}}$
\item $\fa:=\Set{A\subseteq X | A \text{ abzählbar oder } A^c \text{ abzählbar}}$
ist eine $\sigma$-Algebra auf $X$.
\end{enumerate}
\end{beispieleX}
@ -41,13 +41,13 @@ Sei $\fa$ eine $\sigma$-Algebra auf $X$, dann:
\begin{beweis}
\begin{enumerate}
\item \folgtnach{$\sigma_2$} $\emptyset=X^c\in\fa$.
\item $D:=\bigcap A_j$. $D^c=\bigcup A_j^c\in\fa$ (nach
($\sigma_2$) und ($\sigma_3$)), also gilt auch
\item $D:=\bigcap A_j$. $D^c=\bigcup A_j^c\in\fa$ (nach
($\sigma_2$) und ($\sigma_3$)), also gilt auch
$D=(D^c)^c\in\fa$.
\item \begin{enumerate}
\item \folgtnach{($\sigma_3$) mit $A_{n+j}:=\emptyset$ ($j\ge 1$)}
\item \folgtnach{($\sigma_3$) mit $A_{n+j}:=\emptyset$ ($j\ge 1$)}
$A_1\cup\dots\cup A_n\in\fa$.
\item \folgtnach{(2) mit $A_{n+j}:=X$ ($j\ge 1)$}
\item \folgtnach{(2) mit $A_{n+j}:=X$ ($j\ge 1)$}
$A_1\cap\dots\cap A_n\in\fa$.
\item $A_1\setminus A_2=A_1\cap A_2^c\in\fa$
\end{enumerate}
@ -56,8 +56,8 @@ Sei $\fa$ eine $\sigma$-Algebra auf $X$, dann:
\begin{lemma}
\label{Lemma 1.2}
Sei $\cf \neq \emptyset$ eine Menge von $\sigma$-Algebren auf $X$.
Dann ist
Sei $\cf \neq \emptyset$ eine Menge von $\sigma$-Algebren auf $X$.
Dann ist
\[\fa_0:=\bigcap_{\fa\in\cf}\fa\]
eine $\sigma$-Algebra auf $X$.
\end{lemma}
@ -70,7 +70,7 @@ Sei $\fa$ eine $\sigma$-Algebra auf $X$, dann:
\forall\fa\in\cf:A\in\fa &\implies \forall\fa\in\cf:A^c\in\fa\\
&\implies A^c\in\fa_0
\end{align*}
\item[($\sigma_3$)] Sei $(A_j)$ eine Folge in $\fa_0$, dann
\item[($\sigma_3$)] Sei $(A_j)$ eine Folge in $\fa_0$, dann
ist $(A_j)$ Folge in $\fa$ für alle $\fa\in\cf$, dann gilt:
\begin{align*}
\forall\fa\in\cf:\bigcap A_j\in\fa \implies \bigcap A_j\in\fa_0
@ -80,14 +80,14 @@ Sei $\fa$ eine $\sigma$-Algebra auf $X$, dann:
\begin{definition}
\index{Erzeuger}
Sei $\emptyset \neq \mathcal{E} \subseteq \mathcal{P}(X)$ und
$\cf:=\{\fa:\fa$ ist $\sigma$-Algebra auf $X$ mit
Sei $\emptyset \neq \mathcal{E} \subseteq \mathcal{P}(X)$ und
$\cf:=\{\fa:\fa$ ist $\sigma$-Algebra auf $X$ mit
$\mathcal{E}\subseteq\fa\}$. Definiere
\[\sigma(\mathcal{E}):=\bigcap_{\fa\in\cf}\fa\]
\folgtnach{1.2} $\sigma(\mathcal{E})$ ist eine $\sigma$-Algebra
auf $X$. $\sigma(\mathcal{E})$ heißt die
\textbf{von $\mathcal{E}$ erzeugte $\sigma$-Algebra}.
$\mathcal{E}$ heißt ein \textbf{Erzeuger} von
\folgtnach{1.2} $\sigma(\mathcal{E})$ ist eine $\sigma$-Algebra
auf $X$. $\sigma(\mathcal{E})$ heißt die
\textbf{von $\mathcal{E}$ erzeugte $\sigma$-Algebra}.
$\mathcal{E}$ heißt ein \textbf{Erzeuger} von
$\sigma(\mathcal{E})$.
\end{definition}
@ -95,12 +95,12 @@ Sei $\fa$ eine $\sigma$-Algebra auf $X$, dann:
\label{Lemma 1.3}
Sei $\emptyset\ne\mathcal{E}\subseteq\mathcal{P}(X)$.
\begin{enumerate}
\item $\mathcal{E}\subseteq\sigma(\mathcal{E})$.
$\sigma(\mathcal{E})$ ist die "`kleinste"'
\item $\mathcal{E}\subseteq\sigma(\mathcal{E})$.
$\sigma(\mathcal{E})$ ist die "`kleinste"'
$\sigma$-Algebra auf $X$, die $\mathcal{E}$ enthält.
\item Ist $\mathcal{E}$ eine $\sigma$-Algebra, so ist
\item Ist $\mathcal{E}$ eine $\sigma$-Algebra, so ist
$\sigma(\mathcal{E})=\mathcal{E}$.
\item Ist $\mathcal{E}\subseteq\mathcal{E}'$, so ist
\item Ist $\mathcal{E}\subseteq\mathcal{E}'$, so ist
$\sigma(\mathcal{E})\subseteq\sigma(\mathcal{E}')$.
\end{enumerate}
\end{lemma}
@ -108,19 +108,19 @@ Sei $\fa$ eine $\sigma$-Algebra auf $X$, dann:
\begin{beweis}
\begin{enumerate}
\item Klar nach Definition.
\item $\fa:=\mathcal{E}$, dann gilt
\item $\fa:=\mathcal{E}$, dann gilt
$\fa\subseteq\sigma(\mathcal{E})\subseteq\fa$.
\item $\mathcal{E}\subseteq\mathcal{E}'\subseteq\sigma(\mathcal{E}')$,
also folgt nach Definition
\item $\mathcal{E}\subseteq\mathcal{E}'\subseteq\sigma(\mathcal{E}')$,
also folgt nach Definition
$\sigma(\mathcal{E})\subseteq\sigma(\mathcal{E}')$.
\end{enumerate}
\end{beweis}
\begin{beispiel}
\begin{enumerate}
\item Sei $A\subseteq X$ und $\mathcal{E}:=\{A\}$. Dann ist
\item Sei $A\subseteq X$ und $\mathcal{E}:=\{A\}$. Dann ist
$\sigma(\mathcal{E})=\{X,\emptyset,A,A^c\}$.
\item $X:=\{1,2,3,4,5\}, \mathcal{E}:=\{\{1\},\{1,2\}\}$.
\item $X:=\{1,2,3,4,5\}, \mathcal{E}:=\{\{1\},\{1,2\}\}$.
Dann gilt:
\[\sigma(\mathcal{E}):=\{X,\emptyset, \{1\},\{2\},\{1,2\},\{3,4,5\},\{1,3,4,5\},\{2,3,4,5\}\}\]
\end{enumerate}
@ -128,11 +128,11 @@ Sei $\fa$ eine $\sigma$-Algebra auf $X$, dann:
\begin{erinnerung}
\index{Offenheit}\index{Abgeschlossenheit}
Sei $d\in\mdn, X\subseteq\mdr^d$. $A\subseteq X$ heißt
\textbf{offen} (\textbf{abgeschlossen}) in $X$, genau dann wenn
ein offenes (abgeschlossenes) $G\subseteq\mdr^d$ existiert mit
Sei $d\in\mdn, X\subseteq\mdr^d$. $A\subseteq X$ heißt
\textbf{offen} (\textbf{abgeschlossen}) in $X$, genau dann wenn
ein offenes (abgeschlossenes) $G\subseteq\mdr^d$ existiert mit
$A=X\cap G$.\\
Beachte: $A$ abgeschlossen in $X$ $\iff$ $X\setminus A$ offen in
Beachte: $A$ abgeschlossen in $X$ $\iff$ $X\setminus A$ offen in
$X$.
\end{erinnerung}
@ -142,27 +142,27 @@ Sei $\fa$ eine $\sigma$-Algebra auf $X$, dann:
Sei $X\subseteq\mdr^d$.
\begin{enumerate}
\item $\mathcal{O}(X):=\Set{A\subseteq X | A \text{ ist offen in } X}$
\item $\fb(X):=\sigma(\mathcal{O}(X))$ heißt
\item $\fb(X):=\sigma(\mathcal{O}(X))$ heißt
\textbf{Borelsche $\sigma$-Algebra} auf $X$.
\item $\fb_d:=\fb(\mdr^d)$. Die Elemente von $\fb_d$ heißen
\item $\fb_d:=\fb(\mdr^d)$. Die Elemente von $\fb_d$ heißen
\textbf{Borelsche Mengen} oder \textbf{Borel-Mengen}.
\end{enumerate}
\end{definition}
\begin{beispiel}
\begin{enumerate}
\item Sei $\emptyset \neq X\subseteq\mdr^d$. Ist $A\subseteq$
\item Sei $\emptyset \neq X\subseteq\mdr^d$. Ist $A\subseteq$
$\stackrel{\hbox{offen}}{\hbox{abgeschlossen}}$
in $X$, so ist $A\in\fb(X)$.
\item Ist $A\subseteq\mdr^d$
\item Ist $A\subseteq\mdr^d$
$\stackrel{\hbox{offen}}{\hbox{abgeschlossen}}$,
so ist $A\in\fb_d$.
\item Sei $d=1, A=\mdq$. $\mdq$ ist abzählbar, also
$\mdq=\{r_1,r_2,\dots\}$ (mit $r_i\ne r_j$ für $i\ne j$).
Also ist $\mdq=\bigcup \{r_j\}$. Sei nun $r\in\mdq$,
dann ist $B:=(-\infty,r)\cup(r,\infty)\in\fb_1$. Daraus
\item Sei $d=1, A=\mdq$. $\mdq$ ist abzählbar, also
$\mdq=\{r_1,r_2,\dots\}$ (mit $r_i\ne r_j$ für $i\ne j$).
Also ist $\mdq=\bigcup \{r_j\}$. Sei nun $r\in\mdq$,
dann ist $B:=(-\infty,r)\cup(r,\infty)\in\fb_1$. Daraus
folgt $\{r_j\}\in\fb_1$, also auch $\mdq\in\fb_1$.\\
Allgemeiner lässt sich zeigen:
Allgemeiner lässt sich zeigen:
$\mdq^d:=\{(x_1,\dots,x_n):x_j\in\mdq (j=1,\dots,n)\}\in\fb_d$.
\item Sei $x_0 \in \mdr^d, \Set{x_0}$ ist abgeschlossen
$\Rightarrow \Set{x_0} \in \fb$
@ -173,8 +173,8 @@ Sei $\fa$ eine $\sigma$-Algebra auf $X$, dann:
\index{Intervall}
\index{Halbraum}
\begin{enumerate}
\item Seien $I_1,\dots,I_d$ Intervalle in $\mdr$.
Dann heißt $I_1\times\dots\times I_d$ ein \textbf{Intervall}
\item Seien $I_1,\dots,I_d$ Intervalle in $\mdr$.
Dann heißt $I_1\times\dots\times I_d$ ein \textbf{Intervall}
in $\mdr^d$.
\item Seien $a=(a_1,\dots,a_d), b=(b_1,\dots,b_d)\in\mdr^d$.
\[a\le b:\iff a_j\le b_j \quad \forall j \in \Set{1, \dots, d}\]
@ -185,9 +185,9 @@ Sei $\fa$ eine $\sigma$-Algebra auf $X$, dann:
[a,b) &:= [a_1,b_1)\times[a_2,b_2)\times\dots\times[a_d,b_d)\\
[a,b] &:= [a_1,b_1]\times[a_2,b_2]\times\dots\times[a_d,b_d]
\end{align*}
mit der Festlegung $(a,b):=(a,b]:=[a,b):=\emptyset$, falls
mit der Festlegung $(a,b):=(a,b]:=[a,b):=\emptyset$, falls
$a_j=b_j$ für ein $j\in\{1,\dots,d\}$.
\item Für $k\in\{1,\dots,d\}$ und $\alpha\in\mdr$ definiere die
\item Für $k\in\{1,\dots,d\}$ und $\alpha\in\mdr$ definiere die
folgenden \textbf{Halbräume}:
\begin{align*}
H_k^-(\alpha) &:= \Set{(x_1,\dots,x_d)\in\mdr^d:x_k\le\alpha}\\
@ -211,7 +211,7 @@ die beiden Halbräume:\\
\fill[green!15] (a) -- (b) -- (c) -- (d) -- (a);
% Draw lines indicating intersection with y and x axis. Here we
% Draw lines indicating intersection with y and x axis. Here we
% use the perpendicular coordinate system
\draw[dotted] (yaxis |- a) node[left] {$a_2$}
-| (xaxis -| a) node[below] {$a_1$};
@ -274,10 +274,10 @@ Entsprechendes gilt für die anderen Typen von Intervallen und Halbräumen.
\end{satz}
\begin{beweis}
\[\fb_d
\stackrel{(1)}{\subseteq} \sigma(\ce_1)
\stackrel{(2)}{\subseteq} \sigma(\ce_2)
\stackrel{(3)}{\subseteq} \sigma(\ce_3)
\[\fb_d
\stackrel{(1)}{\subseteq} \sigma(\ce_1)
\stackrel{(2)}{\subseteq} \sigma(\ce_2)
\stackrel{(3)}{\subseteq} \sigma(\ce_3)
\stackrel{(4)}{\subseteq} \fb_d
\]
\begin{enumerate}
@ -293,57 +293,57 @@ Entsprechendes gilt für die anderen Typen von Intervallen und Halbräumen.
\[\exists N\in\mdn:\forall n\ge N: \forall j\in\{1,\dots,d\}:a_j<b_j-\frac1n\]
Definiere $c_n:=(\frac1n,\dots,\frac1n)\in\mdq^d$. Dann gilt:
\[(a,b)=\bigcup_{n\ge N}(a,b-c_n]\in\sigma(\ce_2)\]
Also auch $\ce_1\subseteq\sigma(\ce_2)$ und damit
Also auch $\ce_1\subseteq\sigma(\ce_2)$ und damit
$\sigma(\ce_1)\subseteq\sigma(\ce_2)$.
\item Seien $a = (a_1,\dots,a_d), b=(b_1,\dots,b_d) \in \mdq^d$
mit $a \leq b$.
\item Seien $a = (a_1,\dots,a_d), b=(b_1,\dots,b_d) \in \mdq^d$
mit $a \leq b$.
Nachrechnen:
\[(a,b] = \bigcap_{k=1}^d (H^-_k(b_k) \cap H^-_k(a_k)^c) \in \sigma(\ce_3). \]
Das heißt $\ce_2 \subseteq \sigma(\ce_3)$ und damit auch
$\sigma(\ce_2) \subseteq \sigma(\ce_3)$.
\item $H^-_k(\alpha)$ ist abgeschlossen, somit ist
$H^-_k(\alpha)^c$ offen und damit $H^-_k(\alpha)^c \in \fb_d$,
also auch $H^-_k(\alpha) \in \fb_d$. Damit ist
$\ce_3 \subseteq \fb_d \implies \sigma(\ce_3) \subseteq \fb_d$.
Das heißt $\ce_2 \subseteq \sigma(\ce_3)$ und damit auch
$\sigma(\ce_2) \subseteq \sigma(\ce_3)$.
\item $H^-_k(\alpha)$ ist abgeschlossen, somit ist
$H^-_k(\alpha)^c$ offen und damit $H^-_k(\alpha)^c \in \fb_d$,
also auch $H^-_k(\alpha) \in \fb_d$. Damit ist
$\ce_3 \subseteq \fb_d \implies \sigma(\ce_3) \subseteq \fb_d$.
\end{enumerate}
\end{beweis}
\begin{definition}
\index{Spur}
Sei $\emptyset \neq \fm \subseteq \mathcal{P}(X)$ und
$\emptyset \neq Y \subseteq X$.
\[\fm_Y := \{A \cap Y : A \in \fm\}\]
Sei $\emptyset \neq \fm \subseteq \mathcal{P}(X)$ und
$\emptyset \neq Y \subseteq X$.
\[\fm_Y := \{A \cap Y : A \in \fm\}\]
heißt die \textbf{Spur von $\fm$ in $Y$}.
\end{definition}
\begin{beispiel}
$X = \mdr^d, \fm \subseteq \sigma(\mdr^d), \; Y \subseteq X$.
$X = \mdr^d, \fm \subseteq \sigma(\mdr^d), \; Y \subseteq X$.
Dann: $(\co(\mdr^d))_Y = \sigma(Y)$
\end{beispiel}
\begin{satz}[Spuren und $\sigma$-Algebren]
\label{Satz 1.5}
Sei $\emptyset \neq Y \subseteq X$ und $\fa$ eine
Sei $\emptyset \neq Y \subseteq X$ und $\fa$ eine
$\sigma$-Algebra auf $X$.
\begin{enumerate}
\item $\fa_Y$ ist eine $\sigma$-Algebra auf $Y$.
\item $\fa_Y \subseteq \fa \iff Y \in \fa$
\item Ist $\emptyset \neq \ce \subseteq \mathcal{P}(X)$, so
\item Ist $\emptyset \neq \ce \subseteq \mathcal{P}(X)$, so
ist $\sigma(\ce_Y) = \sigma(\ce)_Y$.
\end{enumerate}
\end{satz}
\begin{beweis}
\begin{enumerate}
\item
\item
\begin{enumerate}
\item[($\sigma_1$)] Es ist $Y=Y\cap X\in\fa_Y$, da $X\in\fa$.
\item[($\sigma_2$)] Sei $B\in\fa_Y$, dann existiert ein
\item[($\sigma_2$)] Sei $B\in\fa_Y$, dann existiert ein
$A\in\fa$ mit $B=A\cap Y$.\\
Also ist
Also ist
$Y\setminus B=\overbrace{(X\setminus A)}^{\in\fa} \cap Y\in\fa_Y$.
\item[($\sigma_3$)] Sei $(B_j)$ eine Folge in $\fa_Y$, dann
existiert eine Folge $(A_j)\in\fa^\mdn$
\item[($\sigma_3$)] Sei $(B_j)$ eine Folge in $\fa_Y$, dann
existiert eine Folge $(A_j)\in\fa^\mdn$
mit $B_j=A_j\cap Y$. Es gilt:
\[\bigcup B_j=\bigcup(A_j\cap Y)=(\bigcup A_j)\cap Y\in\fa_Y\]
\end{enumerate}
@ -392,10 +392,10 @@ Außerdem sei $[0,+\infty]:=[0,\infty)\cup\{+\infty\}$.
\begin{cases}
\exists n \in \mdn \text{ mit } a_n = +\infty \text{ oder }\\
\sum a_n \text{ divergiert}
\end{cases}
\end{cases}
\]
\end{enumerate}
Wegen Ana I, 13.1 können Reihen der obigen Form beliebig umgeordnet
\end{enumerate}
Wegen Ana I, 13.1 können Reihen der obigen Form beliebig umgeordnet
werden, ohne dass sich ihr Wert verändert.
\end{definition}
@ -405,13 +405,13 @@ werden, ohne dass sich ihr Wert verändert.
\index{Maßraum}
\index{Maß!endliches}
\index{Wahrscheinlichkeitsmaß}\index{Maß!Wahrscheinlichkeits-}
Sei $\fa$ eine $\sigma$-Algebra auf $X$ und $\mu:\fa\to[0,+\infty]$
eine Abbildung. $\mu$ heißt ein \textbf{Maß} auf $\fa$, genau dann
Sei $\fa$ eine $\sigma$-Algebra auf $X$ und $\mu:\fa\to[0,+\infty]$
eine Abbildung. $\mu$ heißt ein \textbf{Maß} auf $\fa$, genau dann
wenn gilt:
\begin{enumerate}
\item[$(M_1)$] $\mu(\emptyset)=0$
\item[$(M_2)$] Ist $(A_j)$ eine disjunkte Folge in $\fa$, so ist
$\mu(\bigcup A_j)=\sum\mu(A_j)$. Diese Eigenschaft heißt
\item[$(M_2)$] Ist $(A_j)$ eine disjunkte Folge in $\fa$, so ist
$\mu(\bigcup A_j)=\sum\mu(A_j)$. Diese Eigenschaft heißt
\textbf{$\sigma$-Additivität}.
\end{enumerate}
In diesem Fall heißt $(X,\fa,\mu)$ ein \textbf{Maßraum}.\\
@ -424,7 +424,7 @@ Ein Maß $\mu$ heißt ein \textbf{Wahrscheinlichkeitsmaß} $:\Leftrightarrow\mu(
\index{Dirac-Maß}\index{Maß!Dirac-}
\index{Zählmaß}\index{Maß!Zähl-}
\begin{enumerate}
\item Sei $\fa:=\cp(X)$ und $x_0\in X$.
\item Sei $\fa:=\cp(X)$ und $x_0\in X$.
$\delta_{x_0}:\fa\to[0,+\infty]$ sei definiert durch:
\[\delta_{x_0}(A):=
\begin{cases}
@ -438,36 +438,36 @@ Ein Maß $\mu$ heißt ein \textbf{Wahrscheinlichkeitsmaß} $:\Leftrightarrow\mu(
1,\ x_0\in\bigcup A_j\\
0,\ x_0\not\in\bigcup A_j
\end{cases}\right\}=\sum\delta_{x_0}(A_j)\]
$\delta_{x_0}$ ist ein Maß auf $\fa$ und heißt
$\delta_{x_0}$ ist ein Maß auf $\fa$ und heißt
\textbf{Punktmaß} oder \textbf{Dirac-Maß}.
\item Sei $X:=\mdn$, $\fa:=\cp(X)$ und $(p_j)$ eine Folge in
\item Sei $X:=\mdn$, $\fa:=\cp(X)$ und $(p_j)$ eine Folge in
$[0,+\infty]$. Definiere $\mu:\fa\to[0,+\infty]$ durch:
\begin{align*}
\text{Für } A \in \fa: \quad
\text{Für } A \in \fa: \quad
\mu(A):=
\begin{cases}
0 &\text{, falls } A=\emptyset\\
\sum_{j\in A}p_j &\text{, falls } A\ne\emptyset
\end{cases}
\end{align*}
Übung: $\mu$ ist ein Maß auf $\fa=\cp(\mdn)$ und heißt ein \textbf{Zählmaß}.
Sind alle $p_j=1$, so ist $\mu(A)$ gerade die Anzahl der
Übung: $\mu$ ist ein Maß auf $\fa=\cp(\mdn)$ und heißt ein \textbf{Zählmaß}.
Sind alle $p_j=1$, so ist $\mu(A)$ gerade die Anzahl der
Elemente von $A$.
\item Sei $(X,\fa,\mu)$ ein Maßraum, $\emptyset\ne Y\subseteq X$
und $\fa_0\subseteq\fa$ eine $\sigma$-Algebra auf $Y$.
Definiere $\mu_0:\fa_0\to[0,+\infty]$ durch
\item Sei $(X,\fa,\mu)$ ein Maßraum, $\emptyset\ne Y\subseteq X$
und $\fa_0\subseteq\fa$ eine $\sigma$-Algebra auf $Y$.
Definiere $\mu_0:\fa_0\to[0,+\infty]$ durch
$\mu_0(A):=\mu(A)$ ($A\in\fa_0$).\\
Dann ist
Dann ist
$(Y,\fa_0,\mu_0)$ ein Maßraum.\\
Ist spezieller $Y\in\fa$, so ist $\fa_0:=\fa_Y\subseteq\fa$
und man definiert $\mu_{|Y}:\fa_Y\to[0,+\infty]$ durch
Ist spezieller $Y\in\fa$, so ist $\fa_0:=\fa_Y\subseteq\fa$
und man definiert $\mu_{|Y}:\fa_Y\to[0,+\infty]$ durch
$\mu_{|Y}(A):=\mu(A)$ ist ein Maß auf $\fa_Y$.
\end{enumerate}
\end{beispiel}
\begin{satz}
\label{Satz 1.7}
\((X,\fa,\mu)\) sei ein Maßraum, es seien \(A,B\in\fa\) und
\((X,\fa,\mu)\) sei ein Maßraum, es seien \(A,B\in\fa\) und
\((A_{j})\) sei eine Folge in \(\fa\). Dann:
\begin{enumerate}
\item \(A\subseteq B\,\implies\,\mu(A)\leq\mu(B)\)
@ -481,7 +481,7 @@ Ein Maß $\mu$ heißt ein \textbf{Wahrscheinlichkeitsmaß} $:\Leftrightarrow\mu(
\end{satz}
\begin{beweis}
\begin{enumerate}
% Eigentlich muesste es in folgender Zeile statt B=(B\setminus A)\cup A korrekt
% Eigentlich muesste es in folgender Zeile statt B=(B\setminus A)\cup A korrekt
% heissen: B=(B\setminus A)\cupdot A -- Spaeter
\item[(1)-(3)] \(B=(B\setminus A)\cup A\). Dann: \(\mu(B)=\underbrace{\mu(B\setminus A)}_{\geq0}+\mu(A)\geq\mu(A)\)
\item[(4)] % Das muesste jetzt eigentlich Punkt 4 sein

View file

@ -7,7 +7,7 @@ Für \(x\in\mdr^k\):
\[F(x):=\int_{\mdr^l}f(x,y)\,dy=\int_{\mdr^l}f^x(y)\,dy\]
Für \(y\in\mdr^l\):
\[G(y):=\int_{\mdr^k}f(x,y)\,dx=\int_{\mdr^k}f_y(x)\,dx\]
Dann sind $F,G$ messbar und
Dann sind $F,G$ messbar und
\[\int_{\mdr^d}f(z)\,dz=\int_{\mdr^k}F(x)\,dx=\int_{\mdr^l}G(y)\,dy\]
also
\begin{align*}
@ -26,7 +26,7 @@ Für \(x\in\mdr^k\) und \(\natn\) gilt:
und nach Fall 2 ist \(F_n\) messbar. \\
Aus \(0\leq f_n\leq f_{n+1}\) folgt \(0\leq F_n\leq F_{n+1}\) und \ref{Satz 4.6} liefert \(F_n\to F\) auf \(\mdr^k\). Dann gilt
\[\int_{\mdr^d}f(z)\,dz = \lim \int_{\mdr^d}f_n(z)\,dz \overset{Fall 2}= \lim \int_{\mdr^k}F_n(x)\,dx \overset{\ref{Satz 4.6}}=\int_{\mdr^k}F(x)\,dx\]
Genauso zeigt man
Genauso zeigt man
\[\int_{\mdr^d}(f(z)\,dz=\int_{\mdr^l}G(y)\,dy\]
\end{beweis}
@ -35,7 +35,7 @@ Genauso zeigt man
Es sei \(f\colon\mdr^d\to\imdr\) integrierbar. Dann existieren Nullmengen \(M\subseteq\mdr^k\) und \(N\subseteq\mdr^l\) mit
\begin{align*}
f^x\colon\mdr^l\to\imdr \text{ ist integrierbar für jedes } x\in\mdr^k\setminus M \\
f_y\colon\mdr^k\to\imdr \text{ ist integrierbar für jedes } y\in\mdr^l\setminus N
f_y\colon\mdr^k\to\imdr \text{ ist integrierbar für jedes } y\in\mdr^l\setminus N
\end{align*}
Setze
\begin{align*}
@ -60,22 +60,22 @@ Es gilt also wieder \((\ast)\) aus \ref{Satz 10.1}.
Wir zeigen nur die Aussagen über \(f^x\), $F$ und die erste der obigen beiden Gleichungen. Genauso zeigt man die Aussagen über \(f_n, G\) und die zweite Gleichung.\\
Aus \ref{Lemma 8.1} folgt, dass \(f^x\) messbar ist. Definiere
\begin{align*}
\Phi(x) := \int_{\mdr^l}\lvert f^x(y)\rvert\,dy
= \int_{\mdr^l}\lvert f(x,y)\rvert\,dy \ \text{ für } x\in\mdr^k
\Phi(x) := \int_{\mdr^l}\lvert f^x(y)\rvert\,dy
= \int_{\mdr^l}\lvert f(x,y)\rvert\,dy \ \text{ für } x\in\mdr^k
\end{align*}
Nach \ref{Satz 10.1} ist \(\Phi\) messbar und
Nach \ref{Satz 10.1} ist \(\Phi\) messbar und
\begin{align*}
\int_{\mdr^k}\Phi(x)\,dx
= \int_{\mdr^k}\left(\int_{\mdr^l}\lvert f(x,y)\rvert\,dy\right)dx \overset{\ref{Satz 10.1}}
= \int_{\mdr^d}\lvert f(z)\rvert\,dz
\int_{\mdr^k}\Phi(x)\,dx
= \int_{\mdr^k}\left(\int_{\mdr^l}\lvert f(x,y)\rvert\,dy\right)dx \overset{\ref{Satz 10.1}}
= \int_{\mdr^d}\lvert f(z)\rvert\,dz
< \infty
\end{align*}
(denn mit $f$ ist nach \ref{Satz 4.9} auch \(\lvert f\rvert\) integrierbar). Somit ist \(\Phi\) integrierbar.
Setze \(M:=\{\Phi = \infty \}\) was nach \ref{Satz 4.10} eine Nullmenge ist.
Setze \(M:=\{\Phi = \infty \}\) was nach \ref{Satz 4.10} eine Nullmenge ist.
Also gilt:
\begin{align*}
\int_{\mdr^l}\lvert f^x(y)\rvert\,dy
= \Phi(x) < \infty \ \text{ für jedes } x\in\mdr^k\setminus M
\int_{\mdr^l}\lvert f^x(y)\rvert\,dy
= \Phi(x) < \infty \ \text{ für jedes } x\in\mdr^k\setminus M
\end{align*}
Das heißt, \(\lvert f^x\rvert\) ist für jedes \(x\in\mdr^k\setminus M\) integrierbar und es gilt nach \ref{Satz 4.9} auch
\begin{align*}
@ -90,22 +90,22 @@ Setze
0 &\text{, falls } z\in M\times\mdr^l
\end{cases}
\end{align*}
Aus \ref{Lemma 9.3} folgt, dass \(\tilde f\) messbar ist. Klar ist, dass fast überall \(f=\tilde f\) gilt. Es ist
Aus \ref{Lemma 9.3} folgt, dass \(\tilde f\) messbar ist. Klar ist, dass fast überall \(f=\tilde f\) gilt. Es ist
\[\tilde f^x = \left(\mathds{1}_{(M\times\mdr^l)^C}\cdot f\right)^x\]
Das heißt \(\tilde f^x\) ist integrierbar für jedes \(x\in\mdr^k\). Dann gilt
\begin{align*}
F(x) \overset{\ref{Satz 5.3}}
= \int_{\mdr^l}\tilde f(x,y)\,dy
= \underbrace{\int_{\mdr^l}\tilde f_+ (x,y)\,dy}_{=:F^+(x)} - \underbrace{\int_{\mdr^l}\tilde f_- (x,y)\,dy}_{=:F^-(x)}
F(x) \overset{\ref{Satz 5.3}}
= \int_{\mdr^l}\tilde f(x,y)\,dy
= \underbrace{\int_{\mdr^l}\tilde f_+ (x,y)\,dy}_{=:F^+(x)} - \underbrace{\int_{\mdr^l}\tilde f_- (x,y)\,dy}_{=:F^-(x)}
\end{align*}
Nach \ref{Satz 10.1} sind \(F^+\) und \(F^-\) messbar. Die Dreiecksungleichung liefert nun
\begin{align*}
\lvert F(x)\rvert
\leq \int_{\mdr^l}\lvert \tilde f(x,y)\rvert\,dy
\overset{\ref{Satz 5.3}}= \int_{\mdr^l}\lvert f(x,y)\rvert\,dy
\lvert F(x)\rvert
\leq \int_{\mdr^l}\lvert \tilde f(x,y)\rvert\,dy
\overset{\ref{Satz 5.3}}= \int_{\mdr^l}\lvert f(x,y)\rvert\,dy
= \Phi(x) \ \text{ für } x\in\mdr^k
\end{align*}
Also ist \(\lvert F\rvert\leq\Phi\) und \(\Phi\) ist integrierbar. Aus \ref{Satz 4.9} folgt, dass $F$ und \(\lvert F\rvert\) integrierbar sind
Also ist \(\lvert F\rvert\leq\Phi\) und \(\Phi\) ist integrierbar. Aus \ref{Satz 4.9} folgt, dass $F$ und \(\lvert F\rvert\) integrierbar sind
und dann sind auch \(F^+\) und \(F^-\) integrierbar (zur Übung). Es folgt
\begin{align*}
\int_{\mdr^k}F(x)\,dx
@ -138,15 +138,15 @@ Gegeben: \(\emptyset\neq D\subseteq\fb_d\) und messbares \(f\colon D\to\imdr\).
Setze $f$ auf \(\mdr^d\) zu einer messbaren Funktion \(\tilde f\) fort (zum Beispiel wie in \ref{Lemma 9.3}).
Aus \ref{Satz 3.8} folgt dann, dass \(\mathds{1}_{D}\tilde f\) messbar ist und \ref{Satz 10.1} liefert
\begin{align*}
\int_{\mdr^d}\lvert \mathds{1}_{D}\tilde f\rvert\,dz
= \int_{\mdr^k}\left(\int_{\mdr^l}\lvert \mathds{1}_{D}\tilde f\rvert\,dy\right)dx
\int_{\mdr^d}\lvert \mathds{1}_{D}\tilde f\rvert\,dz
= \int_{\mdr^k}\left(\int_{\mdr^l}\lvert \mathds{1}_{D}\tilde f\rvert\,dy\right)dx
= \int_{\mdr^l}\left(\int_{\mdr^k}\lvert \mathds{1}_{D}\tilde f\rvert\,dx\right)dy
\end{align*}
Ist eines der drei obigen Integrale endlich, so ist \(\lvert \mathds{1}_{D}\tilde f\rvert\) integrierbar und
damit ist nach \ref{Satz 4.9} auch \(\mathds{1}_{D}\tilde f\) integrierbar.\\
Dann ist $f$ integrierbar und es folgt
\begin{align*}
\int_Df(z)\,dz
\int_Df(z)\,dz
& = \int_{\mdr^d}\left(\mathds{1}_{D}\tilde f\right)(z)\,dz \\
& \overset{\ref{Satz 10.2}}= \int_{\mdr^k}\left(\int_{\mdr^l}\left(\mathds{1}_{D}\tilde f\right)(x,y)\,dy\right)dx \\
& = \int_{\mdr^l}\left(\int_{\mdr^k}\left(\mathds{1}_{D}\tilde f\right)(x,y)\,dx\right)dy
@ -154,11 +154,11 @@ Dann ist $f$ integrierbar und es folgt
\begin{beispiel}
\begin{enumerate}
\item Sei \(D=[a_1,b_1]\times[a_2,b_2]\times\dots\times[a_d,b_d]\) mit \(a_i\leq b_i \ (i=1,\dots,d)\).
\item Sei \(D=[a_1,b_1]\times[a_2,b_2]\times\dots\times[a_d,b_d]\) mit \(a_i\leq b_i \ (i=1,\dots,d)\).
Es sei \(f\colon D\to\mdr\) stetig. $D$ ist kompakt, also gilt \(D\in\fb_d\).
Nach \ref{Satz 4.12}(2) ist \(f\in\mathfrak{L}^1(D)\) und aus obiger Bemerkung folgt
\begin{align*}
\int_Df(x_1,\dots,x_d)\,d(x_1,\dots,x_d)
\int_Df(x_1,\dots,x_d)\,d(x_1,\dots,x_d)
= \int_{a_d}^{b^d} \left(\dots \left( \int_{a_2}^{b^2} \left(\int_{a_1}^{b^1}f(x_1,\dots,x_d)\,dx_1\right)dx_2\right)\dots\right)dx_d
\end{align*}
Die Reihenfolge der Integrationen darf beliebig vertauscht werden. Aus \ref{Satz 4.13} folgt
@ -167,7 +167,7 @@ Die Reihenfolge der Integrationen darf beliebig vertauscht werden. Aus \ref{Satz
\textbf{Konkretes Beispiel}\\
Sei \(D:=[a,b]\times[c,d]\subseteq\mdr^2\), \(f\in C([a,b])\) und \(g\in C([c,d])\).
\begin{align*}
\int_Df(x)g(y)\,d(x,y)
\int_Df(x)g(y)\,d(x,y)
& = \int_c^d\left(\int_a^bf(x)g(y)\,dx\right)dy \\
& = \int_c^d\left(g(y)\left(\int_a^bf(x)\,dx\right)\right)dy \\
&= \left(\int_a^bf(x)\,dx\right) \left(\int_c^dg(y)\,dy\right)
@ -175,7 +175,7 @@ Sei \(D:=[a,b]\times[c,d]\subseteq\mdr^2\), \(f\in C([a,b])\) und \(g\in C([c,d
\item
Wir rechtfertigen die "'Kochrezepte"' aus Analysis II, Paragraph 15.
Seien \(a,b\in\mdr\) mit \(a<b\) und \(I:=[a,b]\). Weiter seien
\(h_1,h_2\in C(I)\) mit \(h_1\leq h_2\) auf \(I\) und
\(h_1,h_2\in C(I)\) mit \(h_1\leq h_2\) auf \(I\) und
\[A:=\{(x,y)\in\mdr^2: x\in I, h_1(x)\leq y\leq h_2(x)\}\]
Sei \(f\colon A\to\mdr\) stetig. Da \(h_1\) und \(h_2\) stetig
sind, ist \(A\) kompakt und somit gilt \(A\in\fb_2\). Aus
@ -187,15 +187,15 @@ Sei \(D:=[a,b]\times[c,d]\subseteq\mdr^2\), \(f\in C([a,b])\) und \(g\in C([c,d
0 &\text{, falls } (x,y)\notin A
\end{cases}
\]
Nach \ref{Lemma 9.3} ist \(\tilde f\) messbar. Setze
Nach \ref{Lemma 9.3} ist \(\tilde f\) messbar. Setze
\[M:=\max\{\lvert f(x,y)\rvert:(x,y)\in A\}\]
Dann gilt \(\lvert\tilde f\rvert \leq M\cdot\mathds{1}_A\).
Wegen \(\lambda_2(A)<\infty\) ist \(M\cdot\mathds{1}_A\)
Wegen \(\lambda_2(A)<\infty\) ist \(M\cdot\mathds{1}_A\)
integrierbar und nach \ref{Satz 4.9} ist \(\lvert\tilde f\rvert\)
und damit auch \(\tilde f\) integrierbar. Dann ist
\begin{align*}
\int_A f(x,y)\,d(x,y) &= \int_{\mdr^2}\tilde f(x,y)\,d(x,y) \\
& \overset{\ref{Satz 10.3}}=
& \overset{\ref{Satz 10.3}}=
\int_\mdr\left(\int_\mdr\tilde f (x,y)\,dy\right)dx \\
&=\int_a^b\left(\int^{h_2(x)}_{h_1(x)}f(x,y)\,dy\right)dx
\end{align*}
@ -211,7 +211,7 @@ Sei \(D:=[a,b]\times[c,d]\subseteq\mdr^2\), \(f\in C([a,b])\) und \(g\in C([c,d
\(\tilde f\) ist eine Fortsetzung von \(f\) auf \(X\times Y\).
\(\tilde f\) ist also messbar. Es ist
\begin{align*}
\int_D\lvert f\rvert\,d(x,y)
\int_D\lvert f\rvert\,d(x,y)
&=\int_Q\mathds{1}_D\cdot\lvert\tilde f\rvert\,d(x,y) \\
&\overset{\ref{Satz 10.1}}=
\int_X\left(\int_Y\mathds{1}_D(x,y)\frac1x\lvert\cos(xy)\rvert
@ -255,12 +255,12 @@ und daraus folgt \(\int_0^\infty e^{-xy}\,dy=\frac1x\)
1 &\text{, falls } x=0
\end{cases}\]
$g$ ist stetig auf \([0,\infty)\). Aus Analysis 1 ist bekannt, dass
\(\int_0^\infty g(x)\,dx\) konvergent, aber \textbf{ nicht }
absolut konvergent ist. Aus \ref{Satz 4.14} folgt, dass
\(\int_0^\infty g(x)\,dx\) konvergent, aber \textbf{ nicht }
absolut konvergent ist. Aus \ref{Satz 4.14} folgt, dass
\(g\notin\mathfrak{L}^1\left([0,\infty)\right)\)\\
\textbf{Behauptung: } \(\int^\infty_0 g(x)\,dx = \frac\pi{2}\)\\
\textbf{Beweis: } Setze \(X:=[0,R]\) mit \(R>0\), \(Y:=[0,\infty)\) und
\(D:=X\times Y\), sowie
\(D:=X\times Y\), sowie
\[f(x,y):= e^{-xy}\sin x \text{ für } (x,y)\in D\]
Es ist \(D\in\fb_2\) und $f$ stetig, also messbar. Es ist weiter
\(f\in\mathfrak{L}^1(D)\) (warum?) und
@ -301,8 +301,8 @@ und daraus folgt \(\int_0^\infty e^{-xy}\,dy=\frac1x\)
&\leq 2\int^\infty_0 e^{-yR}\,dy \\
&\overset{\text{Vorbemerkung}}=\frac2R
\end{align*}
Das heißt also \(\tilde I_R\to 0 \ (R\to\infty)\) und damit folgt
Das heißt also \(\tilde I_R\to 0 \ (R\to\infty)\) und damit folgt
die Behauptung durch
\[I_R=\frac{\pi}2-\tilde I_R\to\frac{\pi}2 \ (R\to\infty)\]
\[I_R=\frac{\pi}2-\tilde I_R\to\frac{\pi}2 \ (R\to\infty)\]
\end{enumerate}
\end{beispiel}

View file

@ -1,10 +1,10 @@
Die Sätze in diesem Kapitel geben wir \textbf{ohne} Beweis an. Es seien
\(X,Y\subseteq\mdr^d\) nichtleer und offen.
\(X,Y\subseteq\mdr^d\) nichtleer und offen.
\begin{definition}
\index{Diffeomorphismus}
Sei \(\Phi\colon X\to Y\) eine Abbildung. \(\Phi\) heißt
Sei \(\Phi\colon X\to Y\) eine Abbildung. \(\Phi\) heißt
\textbf{Diffeomorphismus} genau dann wenn \(\Phi\in C^1(X,\mdr^d)\), \(\Phi\)
ist bijektiv und \(\Phi^{-1}\in C^{1}(Y,\mdr^d)\).\\
Es gilt \[x=\Phi^{-1}(\Phi(x))\text{ für jedes } x\in X\]
@ -35,7 +35,7 @@ Sei \(A\subseteq\mdr^d\) und \(A^\circ:=\{x\in A :\text{ es existiert ein } r=r(
\end{erinnerung}
\begin{beispiel}
Sei \(A=\mdr\setminus\mdq\). Es ist \(A^\circ=\emptyset\) und
Sei \(A=\mdr\setminus\mdq\). Es ist \(A^\circ=\emptyset\) und
\(A\setminus A^\circ=A\). Aus \(\mdr=A\dot\cup\mdq\) folgt
\[\infty=\lambda_1(\mdr)=\lambda_1(A)+\lambda_1(\mdq)=\lambda_1(A)\]
Das heißt \(A\setminus A^\circ\) ist keine Nullmenge.
@ -66,7 +66,7 @@ Ist $f \in \fl^{1}(B)$ so gilt $(\ast\ast)$
\item Sei $T\colon \MdR^d \to \MdR^d$ linear und $\det T \neq 0$. Weiter sei $A \in \fb_d$ und $v \in \MdR^d$.
Dann ist $T(A) \in \fb_d$ und es gilt:
\[\lambda_d(T(A)+v) = \lvert\det T\rvert \cdot\lambda_d(A)\]
\item $\Phi\colon X \to Y$ sei ein Diffeomorphismus und $A \in \fb(X)$.
\item $\Phi\colon X \to Y$ sei ein Diffeomorphismus und $A \in \fb(X)$.
Dann ist $\Phi(A) \in \fb_d$ und es gilt:
\[\lambda_d(\Phi(A)) = \int_A |\det \Phi'(X)| \, dx\]
\item Sei $F \in C^1(X, \MdR^d)$ und $N \subseteq X$ eine Nullmenge.
@ -101,9 +101,9 @@ y = r \sin(\varphi)
\end{cases}\]
Definiere nun für $(r,\varphi) \in [0,\infty)\times[0,2\pi]$:
\[\Phi(r,\varphi) := (r \cos(\varphi), r \sin(\varphi))\]
Dann ist $\Phi \in C^1(\MdR^2, \MdR^2)$ und es gilt:
Dann ist $\Phi \in C^1(\MdR^2, \MdR^2)$ und es gilt:
\[\Phi'(r,\varphi) = \begin{pmatrix}
\cos(\varphi) & -r \sin(\varphi) \\
\cos(\varphi) & -r \sin(\varphi) \\
\sin(\varphi) & r \cos(\varphi)
\end{pmatrix}\]
d.h. falls $r > 0$ ist gilt:
@ -119,19 +119,19 @@ Mit \ref{Satz 11.2} folgt dann:
\begin{beispiel}
\begin{enumerate}
\item Sei $0 \le \rho < R$. Definiere
\item Sei $0 \le \rho < R$. Definiere
\[B := \{(x,y) \in \MdR^2 : \rho^2 \le x^2 + y^2 \le R^2\} \]
Dann gilt:
Dann gilt:
%% BILD: der Kreisfläche und Trafo
\begin{align*}
\lambda_2(B) &= \int_B 1 \text{ d}(x,y)\\
&= \int_A 1 \cdot r \text{ d}(r,\varphi)\\
\lambda_2(B) &= \int_B 1 \text{ d}(x,y)\\
&= \int_A 1 \cdot r \text{ d}(r,\varphi)\\
&\overset{\text{§\ref{Kapitel 10}}}= \int_{\rho}^{R} \left( \int_0^{2\pi} r \text{ d}\varphi \right) \text{ d}r\\
&= \left[ 2\pi \frac{1}{2} r^2 \right]_\rho^R\\
&= \pi (R^2 - \rho^2)
\end{align*}
\item Definiere
\item Definiere
\[B := \{ (x,y) \in \MdR^2 : x^2 + y^2 \le 1, y \ge 0 \}\]
%% BILD: der (Halb)Kreisfläche und Trafo
Dann gilt:
@ -162,7 +162,7 @@ Außerdem gilt:
&= \int_0^\rho \left( \int_0^\rho e^{-x^2} e^{-y^2} \text{ d}y \right) \text{ d}x \\
&= \left( \int_0^\rho e^{-x^2} \text{ d}x \right)^2
\end{align*}
Wegen $ B_\rho \subseteq Q_\rho \subseteq B_{\sqrt{2} \rho} $ und $f \ge 0$ folgt:
\begin{center}
\begin{tabular}{cccccc}
@ -172,7 +172,7 @@ $\implies$ &$h(\rho)$ &$\le$ &$\left( \int_0^\rho e^{-x^2} \text{ d}x \right)^2$
$\implies$ &$\sqrt{h(\rho)}$ &$\le$ &$\int_0^\rho e^{-x^2} \text{ d}x$ &$\le$ &$\sqrt{h(\sqrt{2} \rho)}$\\
\end{tabular}
\end{center}
Mit $\rho \to \infty$ folgt daraus
Mit $\rho \to \infty$ folgt daraus
\[\int_0^\infty e^{-x^2} \text{ d}x = \frac{\sqrt{\pi}}{2}\]
und damit die Behauptung.
\end{enumerate}

View file

@ -1,11 +1,11 @@
In diesem Kapitel sei $(x_0,y_0)\in\MdR^2$ (fest), es sei
$R:[0,2\pi]\to[0,\infty)$ stetig und stückweise stetig
differenzierbar und $R(0) = R(2\pi)$. Weiter sei
In diesem Kapitel sei $(x_0,y_0)\in\MdR^2$ (fest), es sei
$R:[0,2\pi]\to[0,\infty)$ stetig und stückweise stetig
differenzierbar und $R(0) = R(2\pi)$. Weiter sei
\begin{displaymath}
\gamma(t) := (x_0 + R(t)\cos t,y_0 + R(t)\sin t) \text{ } (t\in[0,2\pi])
\end{displaymath}
Dann ist $\gamma$ ein stückweise stetig differenzierbarer, geschlossener und rektifizierbarer Weg in $\MdR^2$. Es sei
\[B:= \{(x_0+r\cos t,y_0 + r\sin t): t\in [0,2\pi ], 0\le r\le R(t)\}\]
Dann ist $\gamma$ ein stückweise stetig differenzierbarer, geschlossener und rektifizierbarer Weg in $\MdR^2$. Es sei
\[B:= \{(x_0+r\cos t,y_0 + r\sin t): t\in [0,2\pi ], 0\le r\le R(t)\}\]
Dann ist $B$ kompakt, also $B\in\fb_2 $. Weiter ist $\partial B = \gamma([0,2\pi]) = \Gamma_\gamma$.\\
Sind $B$ und $\gamma$ wie oben, so heißt $B$ \begriff{zulässig}.
\index{zulässig}
@ -48,13 +48,13 @@ Mit Polarkoordinaten, Transformations-Satz und Fubini:
A = \int_0^{2\pi }(\int_0^{R(t)} u_x(r\cos t,r\sin t)r dr) dt
\end{displaymath}
\begin{enumerate}
\item $\beta(r,t) := u(r\cos t,r\sin t)$. Nachrechnen: $r\beta_r(r,t)\cos t - \beta_t(r,t)\sin t = u_x(r\cos t,r\sin t)r$. Also:
\item $\beta(r,t) := u(r\cos t,r\sin t)$. Nachrechnen: $r\beta_r(r,t)\cos t - \beta_t(r,t)\sin t = u_x(r\cos t,r\sin t)r$. Also:
\begin{displaymath}
A = \int_0^{2\pi} (\int_0^{R(t)} (r\beta_r(r,t)\cos t - \beta_t(r,t)\sin t) dr)dt
\end{displaymath}
\item $\int_0^{R(t)} r\beta_r(r,t) dr = r\beta(r,t)\vert_{r=0}^{r=R(t)} - \underbrace{\int_0^{R(t)} \beta(r,t) dr}_{=:\alpha(t)} = R(t)\beta(R(t),t) - \alpha(t) = R(t)u(\gamma(t)) -\alpha(t)$
\item $\Psi(s,t) := \int_0^s \beta(r,t)dr$. Mit dem zweiten Hauptsatz aus Analysis 1 folgt: $\Psi_s(s,t) = \beta(s,t)$ \\ 7.3 \folgt $\Psi_t(s,t) = \int_0^s \beta_t(r,t) dr$.\\
Dann: $\alpha(t) = \Psi(R(t),t)$, also
Dann: $\alpha(t) = \Psi(R(t),t)$, also
\begin{displaymath}
\alpha'(t) = \Psi_s(R(t),t)\cdot R'(t) + \Psi_t(R(t),t)\cdot 1 = R'(t)\underbrace{\beta(R(t),t)}_{=u(\gamma(t))} + \int_0^{R(t)} \beta_t(r,t) dr
\end{displaymath}

View file

@ -4,7 +4,7 @@
\index{Parameterbereich}
\index{Normalenvektor}
\index{Flächeninhalt}
Es sei $\emptyset \ne B\subseteq \MdR^2$ kompakt, $D\subseteq\MdR^2$ offen und $B\subseteq D$. Weiter sei $\varphi = (\varphi_1,\varphi_2,\varphi_3) \in C^1(D,\MdR^3)$ und $\varphi = \varphi(u,v)$. Dann heißt $\varphi_{|B}$ eine \textbf{Fläche} (im $\MdR^3$), $S:= \varphi(B)$ heißt \textbf{Flächenstück} und $B$ heißt \textbf{Parameterbereich} der Fläche. Es ist
Es sei $\emptyset \ne B\subseteq \MdR^2$ kompakt, $D\subseteq\MdR^2$ offen und $B\subseteq D$. Weiter sei $\varphi = (\varphi_1,\varphi_2,\varphi_3) \in C^1(D,\MdR^3)$ und $\varphi = \varphi(u,v)$. Dann heißt $\varphi_{|B}$ eine \textbf{Fläche} (im $\MdR^3$), $S:= \varphi(B)$ heißt \textbf{Flächenstück} und $B$ heißt \textbf{Parameterbereich} der Fläche. Es ist
\begin{displaymath}
\varphi' = \begin{pmatrix}\frac{\partial \varphi_1}{\partial u} & \frac{\partial\varphi_1}{\partial v}\\
\frac{\partial \varphi_2}{\partial u} & \frac{\partial\varphi_2}{\partial v}\\
@ -16,10 +16,10 @@
\gamma(t) &:= \varphi(t,v_0) &\gamma'(t) &= \varphi_u(t,v_0) &\gamma'(u_0) &= \varphi_u(u_0,v_0)\\
\tilde{\gamma}(t)&:= \varphi(u_0,t) &\tilde{\gamma}'(t) &= \varphi_v(u_0,v) &\tilde{\gamma}'(v_0) &= \varphi_v(u_0,v_0)
\end{align*}
Definere damit den \textbf{Normalenvektor} in $\varphi(u_0,v_0)$:
Definere damit den \textbf{Normalenvektor} in $\varphi(u_0,v_0)$:
\[N(u_0,v_0) := \varphi_u(u_0,v_0)\times\varphi_v(u_0,v_0)\]
Seien $\Delta u,\Delta v >0$ (aber "`klein"'). $a:= \Delta u\varphi_u(u_0,v_0)$, $b:= \Delta v\varphi_v(u_0,v_0)$.
\[P:= \{\lambda a+\mu b: \ \lambda,\mu\in [0,1]\}\]
\[P:= \{\lambda a+\mu b: \ \lambda,\mu\in [0,1]\}\]
Aus der Linearen Algebra folgt, der "`Inhalt"' von $P$ ist $\|a \times b\| = \Delta u\Delta v \|N(u_0,v_0)\|$.
\begin{displaymath}
I(\varphi) = \int_B \|N(u,v)\| d(u,v)
@ -31,12 +31,12 @@
$B:=[0,2\pi]\times[-\frac\pi2,\frac\pi2]$, $D=\MdR^2$\\
$\varphi(u,v) := (\cos u\cos v,\sin u\cos v,\sin v)$. Dann: $\varphi(B) = \{(x,y,z)\in\MdR^3:\ x^2+y^2+z^2 = 1\}$.\\
Nachrechnen: $N(u,v) = \cos v\varphi(u,v)$. Dann: $\|N(u,v)\| = |\cos v|\underbrace{\|\varphi(u,v)\|}_{=1} = \cos v\ \ \ \ ((u,v)\in B)$. \\
Damit gilt:
Damit gilt:
\[I(\varphi) = \int_B \cos v d(u,v) = \int_0^{2\pi} (\int_{-\frac\pi2}^{\frac\pi2}\cos v d(v)) d(u) = 4\pi\]
\end{beispiel}
\section{Explizite Parameterdarstellung}
Seien \(B\) und \(D\) wie in obiger Definition und \(f\in C^{1}(D,\,\mdr)\). Setze
Seien \(B\) und \(D\) wie in obiger Definition und \(f\in C^{1}(D,\,\mdr)\). Setze
\[\varphi(u,v):=(u,v,f(u,v))\quad((u,v)\in D)\]
Damit ist \(\varphi_{|B}\) eine Fläche (in expliziter Darstellung).
% hier Graphik einfuegen
@ -45,7 +45,7 @@ Dann ist \(S=\varphi(B)\) gleich dem Graph von \(f_{|B}\).
\[
\varphi_{u}=(1,0,f_{u}),\quad \varphi_{v}=(0,1,f_{v}),\quad N(u,v)=(-f_{u},-f_{v},1)\quad\text{(Nachrechnen!)}
\]
Damit gilt:
Damit gilt:
\[I(\varphi)=\int_{B}{(f_{u}^{2}+f_{v}^{2}+1)^{\frac{1}{2}}\mathrm{d}(u,v)}\]
\begin{beispiel}

View file

@ -1,13 +1,13 @@
In diesem Kapitel sei \(\emptyset\neq B\subseteq\mdr^{2}\), \(B\)
In diesem Kapitel sei \(\emptyset\neq B\subseteq\mdr^{2}\), \(B\)
kompakt, \(D\subseteq\mdr^{2}\) offen, \(B\subseteq D\)
und \(\varphi=(\varphi_{1},\varphi_{2},\varphi_{3})\in C^{1}(D,\mdr^{3})\). Das heißt: \(\varphi_{|B}\) ist eine Fläche mit
und \(\varphi=(\varphi_{1},\varphi_{2},\varphi_{3})\in C^{1}(D,\mdr^{3})\). Das heißt: \(\varphi_{|B}\) ist eine Fläche mit
Parameterbereich \(B\), \(S:=\varphi(B)\)
\begin{definition}
\index{Oberflächenintegral}
Definiere die folgenden \textbf{Oberflächenintegrale}:
\begin{enumerate}
\item Sei \(f:\,S\to\mdr\) stetig. Dann:
\item Sei \(f:\,S\to\mdr\) stetig. Dann:
\[
\int_{\varphi}{f\mathrm{d}\sigma}:=\int_{B}{f(\varphi(u,v))\lVert N(u,v)\rVert\mathrm{d}(u,v)}
\]
@ -28,7 +28,7 @@ F(\varphi(u,v))\cdot N(u,v)&=F(u,v,u^{2}+v^{2})\cdot(-2u,-2v,1)\\
&=-(u^{2}+v^{2})
\end{align*}
Also:
Also:
\[
\int_{\varphi}{F\cdot n\mathrm{d}\sigma}=-\int_{B}{(u^{2}+v^{2})\mathrm{d}(u,v)}=-\frac{\pi}{2}
\]
@ -47,7 +47,7 @@ Es sei \(B\) zulässig, \(\partial B=\Gamma_{\gamma}\), wobei \(\gamma=(\gamma_{
\begin{beispiel}
\(D,\,B,\,f,\,F\) und \(\varphi\) seien wie in obigem Beispiel.
% Bild einfuegen
Hier: \(\gamma(t)=(\cos t,\sin t)\quad(t\in [0,2\pi])\).
Hier: \(\gamma(t)=(\cos t,\sin t)\quad(t\in [0,2\pi])\).
Dann: \((\varphi\circ\gamma)(t)=\varphi(\cos t, \sin t)=(\cos t, \sin t, 1)\quad(t\in [0,2\pi])\).
Es ist \(\rot F=0\), also: \(\int_{\varphi}{\rot F\cdot n\mathrm{d}\sigma}=0\)
@ -61,7 +61,7 @@ Es ist \(\rot F=0\), also: \(\int_{\varphi}{\rot F\cdot n\mathrm{d}\sigma}=0\)
\end{beispiel}
\begin{beweis}
Sei \(\varphi:=\varphi\circ\gamma,\,\varphi=(\varphi_{1},\varphi_{2},\varphi_{3})\), also
Sei \(\varphi:=\varphi\circ\gamma,\,\varphi=(\varphi_{1},\varphi_{2},\varphi_{3})\), also
\(\varphi_{j}=\varphi_{j}\circ\gamma\quad(j=1,2,3)\).
Zu zeigen:

View file

@ -48,7 +48,7 @@ Für \(f\in\fl^{\infty}(X)\): \(\lVert f\rVert_{\infty}:=\esssup_{x\in X}\lVert
\end{definition}
\begin{bemerkung}
Es sei \(f\in\fl^{\infty}(X)\) und stetig. Außerdem habe jede in \(X\) offene, nichtleere Teilmenge positives Maß. Dann ist \(f\) auf \(X\) beschränkt und \(\sup_{x\in X}\lvert f(x)\rvert=\esssup_{x\in X}\lvert f(x)\rvert\).
Es sei \(f\in\fl^{\infty}(X)\) und stetig. Außerdem habe jede in \(X\) offene, nichtleere Teilmenge positives Maß. Dann ist \(f\) auf \(X\) beschränkt und \(\sup_{x\in X}\lvert f(x)\rvert=\esssup_{x\in X}\lvert f(x)\rvert\).
\end{bemerkung}
\begin{beweis}
Übung (ist \(N\subseteq X\) eine Nullmenge, so ist \(N^{\circ}=\emptyset\) und \(\overline{X\setminus N}=X\))
@ -57,10 +57,10 @@ Es sei \(f\in\fl^{\infty}(X)\) und stetig. Außerdem habe jede in \(X\) offene,
\begin{beispiel}
Sei \(d=1,\,X=[1,\infty),\,p>1\,(p<\infty),\,\alpha,\beta>0,\,f(x)=\frac{1}{x^{\alpha}},\,g(x)=\frac{1}{x^{\beta}}\)
\begin{enumerate}
\item \[f\in\fl^{p}(X)\overset{\text{\ref{Satz 4.14}}}{\iff}\int_{1}^{\infty}{\frac{1}{x^{\alpha p}}}\mathrm{d}x\]
\item \[f\in\fl^{p}(X)\overset{\text{\ref{Satz 4.14}}}{\iff}\int_{1}^{\infty}{\frac{1}{x^{\alpha p}}}\mathrm{d}x\]
konvergiert genau dann, wenn \(\alpha p>1\Leftrightarrow \alpha>\frac{1}{p}\)
\item
\[fg\in\fl^{1}(X)\overset{\text{\ref{Satz 4.14}}}{\iff}\int_{1}^{\infty}{\frac{1}{x^{\alpha+\beta}}\mathrm{d}x}\]
\[fg\in\fl^{1}(X)\overset{\text{\ref{Satz 4.14}}}{\iff}\int_{1}^{\infty}{\frac{1}{x^{\alpha+\beta}}\mathrm{d}x}\]
konvergiert genau dann, wenn $\alpha+\beta >1$
\end{enumerate}
\end{beispiel}
@ -92,7 +92,7 @@ Ist \(p=2\,(\implies p'=2)\), so heißt obige Ungleichung auch \textbf{Cauchy-Sc
\begin{itemize}
\item[Fall 1:] \(p=1\) (also \(p'=\infty\)) oder \(p=\infty\) (also \(p'=1\)). Etwa \(p=1,\,p'=\infty\).
Sei \(c>0\) und \(N_{c}\subseteq X\) Nullmenge mit: \(\lvert g(x)\rvert\leq c\,\forall x\in X\setminus N_{c}\).
Sei \(c>0\) und \(N_{c}\subseteq X\) Nullmenge mit: \(\lvert g(x)\rvert\leq c\,\forall x\in X\setminus N_{c}\).
\(\tilde{g}:=\mathds{1}_{X\setminus N_{c}}\cdot g\)
Dann: \(g=\tilde{g}\) fast überall und \(\lvert\tilde{g}\rvert\leq c\) auf \(X\). Weiter: \(fg=f\tilde{g}\) fast überall,
@ -102,7 +102,7 @@ Dann:
\[
\int_{X}{\lvert fg\rvert\mathrm{d}x}=\int_{X}{\lvert f\tilde{g}\rvert\mathrm{d}x}=\int_{X}{\lvert f\rvert\underbrace{\lvert\tilde{g}\rvert}_{\leq c}\mathrm{d}x}\leq\int_{X}{\lvert f\rvert\mathrm{d}x}=c\cdot\lVert f\rVert_{1}<\infty
\]
Also: \(fg\in\fl^{1}(X)\) und \(\lVert fg\rVert_{1}\leq c\lVert f\rVert_{1}\). Übergang zum Infimum über alle \(c>0\)
Also: \(fg\in\fl^{1}(X)\) und \(\lVert fg\rVert_{1}\leq c\lVert f\rVert_{1}\). Übergang zum Infimum über alle \(c>0\)
liefert: \(\lVert fg\rVert_{1}\leq\lVert g\rVert_{\infty}\cdot\lVert f\rVert_{1}\)
\item[Fall 2:] Sei \(1<p<\infty\). Ist \(\lVert f\rVert_{p}=0\) oder \(\lVert g\rVert_{p'}=0\), so ist \(f=0\) fast überall
oder \(g=0\) fast überall. Daraus folgt: \(\lvert fg\rvert=0\) fast überall.
@ -146,7 +146,7 @@ Nullmengen und \(\lvert f(x)\rvert\leq c_{1}\forall x\in X\setminus N_{1},\,\lve
\item[Fall 3:] Sei \(1<p<\infty\) und \(f,\,g\in\fl^{p}(X)\). Es ist \(\lvert f+g\rvert^{p}\leq(\lvert f\rvert+\lvert g\rvert)^{p}\leq\left(2\max\{\lvert f\rvert,\,\lvert g\rvert\}\right)^{p}\leq 2^{p}\left(\lvert f\rvert^{p}+\lvert g\rvert^{p}\right)\)
auf \(X\). Mit \ref{Satz 4.9} folgt: \(\lvert f+g\rvert^{p}\in\fl^{1}(X)\implies f+g\in\fl^{p}(X)\)\\
\(p'=\frac{p}{p-1};\,h:=\lvert f+g\rvert^{p-1}\), dann: \(h^{p'}=\left(\lvert f+g\rvert^{p-1}\right)^{\frac{p}{p-1}}=\lvert f+g\rvert^{p}\in\fl^{1}(X)\). Dann ist \(h\in\fl^{p'}(X)\). Also: \(h\in\fl^{p'}(X),\,f\in\fl^{p}(X)\)
\(p'=\frac{p}{p-1};\,h:=\lvert f+g\rvert^{p-1}\), dann: \(h^{p'}=\left(\lvert f+g\rvert^{p-1}\right)^{\frac{p}{p-1}}=\lvert f+g\rvert^{p}\in\fl^{1}(X)\). Dann ist \(h\in\fl^{p'}(X)\). Also: \(h\in\fl^{p'}(X),\,f\in\fl^{p}(X)\)
(und \(\frac{1}{p}+\frac{1}{p'}=1\)).
Mit der Hölderschen Ungleichung folgt:
@ -201,7 +201,7 @@ Also gilt:
\begin {beispiel}
\begin{enumerate}
\item Sei $X:=(0,1]$, $1\le p<q<\infty$ (also $\frac 1q<\frac1p$) und $f(x):=\frac 1{x^\alpha}$ $(\alpha>0)$. Dann gilt nach
\item Sei $X:=(0,1]$, $1\le p<q<\infty$ (also $\frac 1q<\frac1p$) und $f(x):=\frac 1{x^\alpha}$ $(\alpha>0)$. Dann gilt nach
\ref{Satz 4.14} und Analysis I:
\begin{align*}
f\in\fl^p(X)&\iff\int_0^1\frac1{x^{\alpha p}}\text{ d}x \text{ konvergiert}\\
@ -229,13 +229,13 @@ Dann ist $f\in\fl^p(X)$ und es gilt
Aus (i) und (ii) folgt: $|f|^p \leq g$ f.ü.
Im Kapitel 5 haben wir gesehen, dass dann gilt:
\[ \int_X |f|^p \text{ d}x \leq \int_X g \text{ d}x < \infty \]
(denn $g$ ist nach Voraussetzung integrierbar).
(denn $g$ ist nach Voraussetzung integrierbar).
Daraus folgt: $f \in \fl^p(X)$.
Setze $g_n := |f_n - f|^p$. Aus (i): $g_n \to 0$ f.ü. Es sind $f_n, f \in \fl^p(X)$ (ersteres nach Voraussetzung, zweiteres haben wir gerade gezeigt), und weil $\fl^p(X)$ ein reeller Vektorraum ist (\ref{Satz 16.1}(2)), folgt:
\[ f_n - f \in \fl^p(X) \]
Also $g_n \in \fl^1(X)$.
Es ist
Es ist
\[ 0 \leq g_n \leq \left( |f_n| + |f| \right)^p \leq \left( g^{\frac{1}{p}} + g^{\frac{1}{p}} \right)^p = \left( 2g^{\frac{1}{p}} \right)^p = 2^p g \quad\text{f.ü.} \]
Mit \ref{Satz 6.2} folgt schließlich:
\[ \underbrace{\int_X g_n \text{ d}x}_{=\|f_n - f\|_p^p} \to 0. \]
@ -256,19 +256,19 @@ und die Addition
zu einem Vektorraum über $\mdr$ wird.
\end{definition}
Setze für $\hat f \in L^1(X)$:
Setze für $\hat f \in L^1(X)$:
\[\int_X \hat f(x) \text{ d}x := \int_X f(x) \text{ d}x\]
dabei ist diese Definition unabhängig von der Wahl des Repräsentanten $f \in \fl^1(X)$ von $\hat f$, denn: ist auch noch $g \in \fl^1(X)$ und $\hat g = \hat f$, so ist $f - g \in \cn$, also $f-g = 0$ f.ü. und damit: $\int_X f \text{ d}x = \int_X g \text{ d}x$.
Für $\hat f \in L^p(X)$ definiere
Für $\hat f \in L^p(X)$ definiere
\[\| \hat f \|_p := \| f \|_p\]
wobei diese Definition unabhängig ist von der Wahl des Repräsentanten $f \in \fl^p(X)$ von $\hat f$.
Für $\hat f, \hat g \in L^2(X)$ setze
Für $\hat f, \hat g \in L^2(X)$ setze
\[( \hat f | \hat g ) := \int_X f(x)g(x) \text{ d}x\]
(auch diese Definition ist Repräsentanten-unabhängig) (Beachte: $f\cdot g \in \fl^1(X)$ )
\textbf{Dann gilt:}
\textbf{Dann gilt:}
\index{Ungleichung!Cauchy-Schwarz}
\begin{enumerate} \item $L^p(X)$ ist unter $\| \cdot \|_p$ ein normierter Raum (NR).
\item Für $\hat f, \hat g \in L^2(X)$ gilt:
@ -296,7 +296,7 @@ so heißt $B$ ein \textbf{Prähilbertraum}. Ist $B$ ein Banachraum mit $(*)$, so
Seien \(f,f_n\in\fl^p(X)\)
\begin{enumerate}
\item \(\| f_n-f\|_p = \| \hat{f_n}-\hat f\|_p\to 0\) genau
dann, wenn \((\hat{f_n})\) eine konvergente Folge im normierten Raum \(L^p(X)\)
dann, wenn \((\hat{f_n})\) eine konvergente Folge im normierten Raum \(L^p(X)\)
mit dem Grenzwert \(\hat f\) ist.
\item \((\hat f_n)\) ist eine \textbf{Cauchyfolge} (CF) in \(L^p(X)\) genau dann, wenn für jedes $\ep>0$ ein $n_0\in\mdn$ exitiert mit:
\begin{align*}
@ -331,26 +331,26 @@ Sei \(X=[0,1]\) und \((I_n)\) sei die folgende Folge von Intervallen:
\[I_1=\left[0,1\right], I_2=\left[0,\frac12\right], I_3=\left[\frac12,1\right], I_4=\left[0,\frac14\right],
I_5=\left[\frac14,\frac12\right], I_6=\left[\frac12, \frac34\right], I_7=\left[\frac34,1\right], \dots\]
Es sei \(f_n:=\mathds{1}_{I_n}\), sodass \(\int_X f_n\,dx=\int_{I_n}1\,dx=\lambda_1(I_n)\to 0\).
Also \(\hat f_n\in L^1(X)\) und \(\| \hat f_n-\hat 0\|_1\to 0\).
Ist \(x\in X\), so gilt: \(x\in I_n\) für unendlich viele \natn. Daraus folgt, dass eine Teilfolge
Also \(\hat f_n\in L^1(X)\) und \(\| \hat f_n-\hat 0\|_1\to 0\).
Ist \(x\in X\), so gilt: \(x\in I_n\) für unendlich viele \natn. Daraus folgt, dass eine Teilfolge
\(I_{n_j}\) mit \(x\in I_{n_j}\) für jedes \(j\in\mdn\) existiert. Somit ist \(f_{n_j}(x)=1\) für jedes \(j\in\mdn\)
und deshalb gilt fast überall \(f_n\nrightarrow 0\).
\end{beispiel}
\begin{beweis}[von \ref{Satz 16.4}]
Setze \(\ep_j:=\frac1{2^j}\ (j\in\mdn)\).
Zu \(\ep_1\) existiert ein \(n_1\in\mdn\) mit \(\| f_l-f_{n_1}\|_p<\ep_1\)
für alle \(l\geq n_1\).
Zu \(\ep_2\) existiert ein \(n_2\in\mdn\) mit \(n_2>n_2\) und
Setze \(\ep_j:=\frac1{2^j}\ (j\in\mdn)\).
Zu \(\ep_1\) existiert ein \(n_1\in\mdn\) mit \(\| f_l-f_{n_1}\|_p<\ep_1\)
für alle \(l\geq n_1\).
Zu \(\ep_2\) existiert ein \(n_2\in\mdn\) mit \(n_2>n_2\) und
\(\| f_l-f_{n_2}\|_p<\ep_2\) für alle \(l\geq n_2\).
Etc.\\
Wir erhalten eine Teilfolge \((f_{n_j})\) mit
Wir erhalten eine Teilfolge \((f_{n_j})\) mit
\[(+)\ \ \ \| f_l-f_{n_j}\|_p<\ep_j \text{ für alle } l\geq n_j \text{ mit } j\in\mdn\]
Setze \(g_j:=f_{n_{j+1}}-f_{n_j}\ (j\in\mdn)\). Klar: \(g_l\in\fl^p(X)\).
Für \(N\in\mdn\): \[S_N:=\int_X\left(\sum^N_{j=1}\lvert g_j(x)\rvert^p\right)^{\frac1p}\]
Dann:
\begin{align*}
S_N=\left\lvert\left\lvert\sum^N_{j=1}\lvert g_j\rvert\right\rvert\right\rvert_p
S_N=\left\lvert\left\lvert\sum^N_{j=1}\lvert g_j\rvert\right\rvert\right\rvert_p
\leq \sum^N_{j=1}\| g_j\|_p
\overset{\text{(+)}}\leq \sum^N_{j=1}\ep_j
=\sum^N_{j=1}\frac1{2^j}
@ -368,10 +368,10 @@ Somit ist \(g^p\) ist integrierbar. Aus \ref{Satz 5.2} folgt, dass eine Nullmeng
existiert mit \(0\leq g^p(x)<\infty\) für alle \(x\in X\setminus N_1\). Es ist dann auch
\(0\leq g(x)<\infty\) für alle \(x\in X\setminus N_1\) und somit folgt nach Konstruktion von $g$, dass
\(\sum^\infty_{j=1}g_j\,dx\) konvergiert absolut in jedem \(x\in X\setminus N_1\).
Aus Analysis I folgt, dass damit \(\sum^\infty_{j=1}g_j\,dx\) in jedem
Aus Analysis I folgt, dass damit \(\sum^\infty_{j=1}g_j\,dx\) in jedem
\(x\in X\setminus N_1\) konvergiert.
Für \(m\in\mdn\):
Für \(m\in\mdn\):
\[\sum^{m-1}_{j=1}g_j=f_{n_m}-f_{n_1} \implies f_{n_m}=\sum^{m-1}_{j=1}g_j + f_{n_1} \]
Deshalb ist \((f_{n_m})\) konvergent (in \mdr) für alle \(x\in X\setminus N_1\).
\begin{align*}
@ -381,18 +381,18 @@ f(x):=
0 &, x\in N_1
\end{cases}
\end{align*}
Aus \S 3 ist bekannt, dass $f$ messbar ist. Klar: \(f_{n_m}\to f\) fast überall und
Aus \S 3 ist bekannt, dass $f$ messbar ist. Klar: \(f_{n_m}\to f\) fast überall und
\(f(X)\subseteq\mdr\).
Es ist \(f_{n_m}=\sum^{m-1}_{j=1}g_j + f_{n_1}\) und somit
Es ist \(f_{n_m}=\sum^{m-1}_{j=1}g_j + f_{n_1}\) und somit
\[\lvert f_{n_m}\rvert = \lvert f_{n_1}\rvert + \sum^{m-1}_{j=1}g_j \leq \lvert f_{n_1}\rvert +
\lvert g\rvert\]
Wie im Beweis von Satz \ref{Satz 16.1} folgern wir
\[\lvert f_{n_m}\rvert^p\leq 2^p\left(\lvert f_{n_1}\rvert^p+g^p\right)=:\tilde g \]
\(f_{n_1}\in\fl^p(X)\), \(g^p\) ist integrierbar. Aus \ref{Satz 16.3} folgt, dass \(f\in\fl^p(X)\)
und \[\| f_{n_m}-f\|_p\to 0 \ (m\to\infty)\]
Sei nun \(\ep>0\). Wähle \(m\in M\) so, dass \(\frac1{2^m}<\frac\ep2\) und
Sei nun \(\ep>0\). Wähle \(m\in M\) so, dass \(\frac1{2^m}<\frac\ep2\) und
\(\| f-f_{n_m}\|_p<\frac\ep2\).
Für \(l\geq n_m\) gilt:
Für \(l\geq n_m\) gilt:
\[\| f_l-f\|_p= \| f_l-f_{n_m}+f_{n_m}-f\|_p
\leq \| f_l-f_{n_m}\|_p + \| f_{n_m}-f\|_p
\overset{\text{(+)}}< \frac1{2^m}+\frac\ep2 <\ep\]
@ -416,7 +416,7 @@ Dann ist fast überall \(f=g\).
\item[\textbf{1.}]
Aus Bemerkung (3) vor \ref{Satz 16.4} folgt, dass \((\hat f_n)\) ist eine Cachyfolge in
\(L^p(X)\). Wegen \ref{Satz 16.4} existiert dann ein \(\varphi\in\fl^p(X)\) und eine Teilfolge
\((f_{n_j})\) mit: \(f_{n_j}\to\varphi\) fast überall und
\((f_{n_j})\) mit: \(f_{n_j}\to\varphi\) fast überall und
\(\| f_n-\varphi\|_p\to0\)
\begin{align*}
\| f-\varphi\|_p
@ -434,14 +434,14 @@ Dann ist fast überall \(f=g\).
\[g_{j_k}(x)\to g(x) \text{ für alle } x\in X\setminus N_2\]
\end{enumerate}
Wir wissen, dass \(N:=N_1\cup N_2\) eine Nullmenge ist. Sei nun \(x\in X\setminus N\). Dann
folgt aus dem ersten Schritt \(f_{n_j}(x)\to f(x)\) und daraus
folgt aus dem ersten Schritt \(f_{n_j}(x)\to f(x)\) und daraus
\[ \underbrace{f_{n_{j_k}}(x)}_{=g_{n_{j_k}}(x)}\to f(x) \]
Aus dem Zweiten Schritt folgt dann, dass \(f_{n_{j_k}}(x)\to g(x)\) und somit \(f(x)=g(x)\).
\end{beweis}
\begin{bemerkung}
Seien \(f_n,f\in\fl^p(X)\) und es gelte \(\| f_n-f\|_p\to 0\ \ (n\to\infty)\). Der
Beweis von \ref{Satz 16.5} zeigt, dass eine Teilfolge \((f_{n_j})\) von \((f_n)\) existiert mit
Beweis von \ref{Satz 16.5} zeigt, dass eine Teilfolge \((f_{n_j})\) von \((f_n)\) existiert mit
\(f_{n_j}\to f\) fast überall.
\end{bemerkung}
@ -457,7 +457,7 @@ Sei \((f_n)\) wie im Beispiel vor \ref{Satz 16.4}. Also \(\| f_n-0\|_p\to 0\), a
\begin{beispiel}
%Bild einfügen
Sei \(X=[0,1]\) und \(f_n\) sei wie im Bild. \(f_n\) ist stetig, also messbar.
Sei \(X=[0,1]\) und \(f_n\) sei wie im Bild. \(f_n\) ist stetig, also messbar.
\[\int_X f_n\,dx=1 \text{ für alle } \natn\]
Somit ist \(f_n\in\fl^1(X)\).
\[f_n(x)\to
@ -465,7 +465,7 @@ Somit ist \(f_n\in\fl^1(X)\).
0, x\in(0,1]\\
1, x=0
\end{cases}\]
Damit gilt fast überall \(f_n\to0\), aber
Damit gilt fast überall \(f_n\to0\), aber
\(\| f_n-0\|_1=1\nrightarrow0 \ \ (n\to\infty)\)
\end{beispiel}
@ -481,9 +481,9 @@ Seien \((E,\|\cdot\|_1), (F,\|\cdot\|_2)\) normierte Räume.
\[\sum^\infty_{n=1}x_n:=\lim_{n\to\infty}s_n\]
\item \(\Phi\colon E\to F\) sei eine Abbildung. \(\Phi\) heißt \textbf{stetig} in \(x_0\in E\)
genau dann, wenn für jede konvergente Folge \((x_n)\) in $E$ mit \(x_n\to x_0\)
gilt: \[\Phi(x_n)\to\Phi(x_0)\]
gilt: \[\Phi(x_n)\to\Phi(x_0)\]
\(\Phi\) heißt auf $E$ stetig genau dann, wenn \(\Phi\) ist in jedem \(x\in E\) stetig.
\item Für $(x,y)\in E\times E$ setze
\item Für $(x,y)\in E\times E$ setze
\[\|(x,y)\|:=\sqrt{\|x\|_1^2+\|y\|_1^2}\]
Dann ist $\|\cdot\|$ eine Norm auf $E\times E$ (nachrechnen!). Weiter gilt, dass $E\times E$ genau dann ein Banachraum ist, wenn $E$ einer ist. Für eine Folge $((x_n,y_n))$ in $E\times E$ und $(x,y)\in E\times E$ gilt
\[(x_n,y_n)\stackrel{\|\cdot\|}\to (x,y) \iff x_n\stackrel{\|\cdot\|}\to x \wedge y_n\stackrel{\|\cdot\|}\to y\]
@ -506,7 +506,7 @@ Für den Rest dieser Vorlesung schreiben wir (meist) $f$ statt $\hat f$ und iden
\item Die Abbildung $\Phi:L^p(X)\to\mdr$, definiert durch
\[\Phi(f):=\|f\|_p\]
ist stetig auf $L^p(X)$. D.h. für $f_n,f\in L^p(X)$ mit $f_n\stackrel{\|\cdot\|_p}\to f$ gilt $\|f_n\|_p\to\|f\|_p$, also
\[\int_X|f_n|^p\text{ d}x\to\int_X|f|^p\text{ d}x\]
\[\int_X|f_n|^p\text{ d}x\to\int_X|f|^p\text{ d}x\]
\begin{beweis}
Aus Analysis II §17 folgt:
\[| \|f_n\|_p-\|f\|_p |\le \|f_n-f\|_p\stackrel{n\to\infty}\to 0\]
@ -553,13 +553,13 @@ Es genügt den Fall $f\ge 0$ zu betrachten (also $f=f_+$, $f_-\equiv 0$). Sei al
\begin{align*}
0\le\varphi_n&\le (|f_n|+|f|)^p\\
&=|f_n+f|^p\le (2f)^p\\
&=2^pf^p=:g
&=2^pf^p=:g
\end{align*}
Dann ist $g\in L^1(X)$ integrierbar.\\
Aus \ref{Satz 4.9} folgt:
\begin{align*}
\varphi\in L^1(X)&\implies f_n-f\in L^p(X)\\
&\implies f_n=(f_n-f)+f\in L^p(X)
&\implies f_n=(f_n-f)+f\in L^p(X)
\end{align*}
Aus \ref{Satz 6.2} folgt:
\[\int_X\varphi_n\text{ d}x\to 0 \implies \|f_n-f\|_p^p\to 0\]

View file

@ -12,7 +12,7 @@ Aus 3.2 folgt: $f$ ist messbar genau dann, wenn $u$ und $v$ messbar sind.
\begin{definition}
\index{integrierbar}\index{Integral}
Sei $f$ messbar. $f$ heißt \textbf{integrierbar} (ib.) genau dann, wenn $u$ und $v$ integrierbar sind.
In diesem Fall setze
In diesem Fall setze
\[ \int_X f \text{ d}x := \int_X u \text{ d}x + i\int_X v \text{ d}x \quad ( \in \MdC) \]
\end{definition}
@ -23,22 +23,22 @@ Hieraus und aus 4.9 folgt: $f$ ist integrierbar genau dann, wenn $|f|$ integrier
\[ \fl^p(X, \MdC) := \{ f : X \to \MdC | f \text{ ist messbar und } \int_X |f|^p \text{ d}x < \infty \} \]
(Achtung: mit den Betragsstrichen in ob. Integral ist der komplexe Betrag gemeint!)
\[ \cn := \{ f: X \to \MdC | f \text{ ist messbar und } f = 0 \text{ f.ü.} \} \]
$\fl^p(X,\MdC )$ ist ein komplexer Vektorraum (siehe 17.1) und $\cn$ ist ein Untervektorraum von $\fl^p(X,\MdC )$.
$\fl^p(X,\MdC )$ ist ein komplexer Vektorraum (siehe 17.1) und $\cn$ ist ein Untervektorraum von $\fl^p(X,\MdC )$.
\[ L^p(X,\MdC ) := \fl^p(X,\MdC)\diagup\cn \]
\end{definition}
\begin{definition}
\index{orthogonal}
Für $f,g \in L^2(X,\MdC )$ setze
Für $f,g \in L^2(X,\MdC )$ setze
\[(f | g) := \int_X f(x) \overline{g(x)} \text{ d}x\]
sowie
sowie
\[f \bot g :\Longleftrightarrow (f | g) = 0 \quad \text{ ($f$ und $g$ sind \textbf{orthogonal}).} \]
( $\overline{z}$ bezeichne hierbei die komplex Konjugierte von $z$, vgl. Lineare Algebra).
\end{definition}
\textbf{Klar:} \begin{enumerate}
\item $L^p(X,\MdC )$ ist mit $\| f \|_p := (\int_X |f|^p \text{ d}x )^{\frac{1}{p}}$ ein komplexer normierter Raum (NR).
\item $(f | g)$ definiert ein Skalarprodukt auf $L^2(X,\MdC)$. Es ist
\item $(f | g)$ definiert ein Skalarprodukt auf $L^2(X,\MdC)$. Es ist
\[(f | g) = \overline{(g | f)}, \]
\[ (f | f) = \int_X f(x) \overline{f(x)} \text{ d}x = \int_X |f(x)|^2 \text{ d}x = \| f \|_2^2 \text{, also:} \]
\[ \| f\|_2 = \sqrt{(f|f)} \quad (f,g \in L^2(X,\MdC )) \]
@ -56,16 +56,16 @@ sowie
\begin{enumerate}
\item Seien \(f,g\colon X\to\mdc\) integrierbar und \(\alpha,\beta\in\mdc\). Dann gelten:
\begin{enumerate}
\item[(i)] \(\alpha f+\beta g\) ist integrierbar und
\item[(i)] \(\alpha f+\beta g\) ist integrierbar und
\[\int_X(\alpha f+\beta g)\,dx = \alpha\int_Xf\,dx+\beta\int_Xg\,dx\]
\item[(ii)] \(\text{Re}\left(\int_Xf\,dx\right) = \int_X\text{Re}(f)\,dx\ \) und
\(\ \text{Im}\left(\int_Xf\,dx\right) = \int_X\text{Im}(f)\,dx\)
\item[(iii)] \(\overline f\) ist integrierbar und
\item[(iii)] \(\overline f\) ist integrierbar und
\[\int_X\overline f\,dx=\overline{\int_Xf\,dx}\]
\end{enumerate}
\item Die Sätze \ref{Satz 16.1} bis \ref{Satz 16.3} und das Beispiel \ref{Beispiel 16.6} gelten in
\item Die Sätze \ref{Satz 16.1} bis \ref{Satz 16.3} und das Beispiel \ref{Beispiel 16.6} gelten in
\(L^p(X,\mdc)\).
\item \(L^p(X,\mdc)\) ist ein komplexer Banachraum, \(L^2(X,\mdc)\) ist ein komplexer
\item \(L^p(X,\mdc)\) ist ein komplexer Banachraum, \(L^2(X,\mdc)\) ist ein komplexer
Hilbertraum.
\end{enumerate}
\end{satz}
@ -79,12 +79,12 @@ Sei \(X=[0,2\pi]\). Für \(k\in\MdZ\) und \(t\in\mdr\) setzen wir
Dann gilt: \(b_k,e_k\in L^2([0,2\pi],\mdc)\) und \[\int_0^{2\pi}e_0(t)\,dt=2\pi\]
Für \(k\in\MdZ\) und \(k\neq0\) ist
\begin{align*}
\int_0^{2\pi}e_k(t)\,dt=\left.\frac1{ik}e^{ikt}\right\rvert_0^{2\pi}
\int_0^{2\pi}e_k(t)\,dt=\left.\frac1{ik}e^{ikt}\right\rvert_0^{2\pi}
= \frac1{ik}\left(e^{2\pi ki}-1\right)=0
\intertext{Damit ist}
(b_k\mid b_l) = \int^{2\pi}_0 b_k\overline{b_l}\,dt = \frac1{2\pi}\int_0^{2\pi}e^{ikt}e^{-ilt}\,dt
= \frac1{2\pi}\int_0^{2\pi}e^{i(k-l)t}\,dt =
\begin{cases}
\begin{cases}
1 ,\text{falls } k=l\\
0 ,\text{falls }k\neq l
\end{cases}
@ -95,19 +95,19 @@ Zur Übung: \(\{b_k\mid k\in\MdZ\}\) ist linear unabhängig in \(L^2([0,2\pi],\m
\end{wichtigesbeispiel}
\begin{definition}
Sei \((\alpha_k)_{k\in\MdZ}\) eine Folge in \(\mdc\) und \((f_k)_{k\in\MdZ}\) eine Folge in
Sei \((\alpha_k)_{k\in\MdZ}\) eine Folge in \(\mdc\) und \((f_k)_{k\in\MdZ}\) eine Folge in
\(L^2(X,\mdc)\).
\begin{enumerate}
\item Für \(n\in\mdn_0\) setze
\item Für \(n\in\mdn_0\) setze
\[s_n:=\sum^n_{k=-n}\alpha_k = \sum_{\lvert k\rvert\leq n}\alpha_k
=\alpha_{-n}+\alpha_{-(n-1)}+\dots+\alpha_0+\alpha_1+\dots+\alpha_n\]
Existiert \(\lim_{n\to\infty}s_n\) in \(\mdc\), so schreiben wir
\(\sum_{k\in\MdZ}\alpha_k:=\lim_{n\to\infty}s_n\)
\item Für \(n\in\mdn_0\) setze
\item Für \(n\in\mdn_0\) setze
\[\sigma_n:=\sum^n_{k=-n}f_k=\sum_{\lvert k\rvert\leq n}f_k\]
Gilt für ein \(f\in L^2(X,\mdc)\):
Gilt für ein \(f\in L^2(X,\mdc)\):
\(\| f-\sigma_n\|_2\overset{n\to\infty}\longrightarrow 0\), so schreiben
wir \[f\overset{\|\cdot\|_2}=\sum_{k\in\MdZ}f_k \ \ \
wir \[f\overset{\|\cdot\|_2}=\sum_{k\in\MdZ}f_k \ \ \
\left(=\lim_{n\to\infty}\sigma_n \text{ im Sinne der } L^2\text{-Norm}\right)\]
\end{enumerate}
\end{definition}
@ -119,19 +119,19 @@ Sei \(\{b_k\mid k\in\MdZ\}\) wie in \ref{Beispiel 17.2}. \(\{b_k\mid k\in\MdZ\}\
\(f\in L^2([0,2\pi],\mdc)\) eine Folge \[(c_k)_{k\in\MdZ}=(c_k(f))_{k\in\MdZ}\] gibt, mit
\[(\ast)\ \ \ \ \ \ \ \ \ f\overset{\|\cdot\|_2}=\sum_{k\in\MdZ}c_kb_k \]
\textbf{Frage:} Ist \(\{b_k\mid k\in\MdZ\}\) eine ONB von \(L^2([0,2\pi],\mdc)\)?\\
\textbf{Antwort:} Ja! In \ref{Satz 18.5} werden wir sehen, dass \((\ast)\) gilt mit
\textbf{Antwort:} Ja! In \ref{Satz 18.5} werden wir sehen, dass \((\ast)\) gilt mit
\(c_k=(f\mid b_k)\).
\end{definition}
\chapter{Fourierreihen}
\label{Kapitel 18}
In diesem Kapitel sei stets \(X=[0,2\pi]\), \(L^2:=L^2([0,2\pi],\mdc)\) und
In diesem Kapitel sei stets \(X=[0,2\pi]\), \(L^2:=L^2([0,2\pi],\mdc)\) und
\(L^2_\mdr:=L^2([0,2\pi],\mdr)\). Weiter sei \(\{b_k\mid k\in\MdZ\}\) wie in \ref{Beispiel 17.2}.
\begin{satz}
\label{Satz 18.1}
Ist \(f\in L^2\) und gilt mit einer Folge \((c_k)_{k\in\MdZ}\) in \(\mdc\):
Ist \(f\in L^2\) und gilt mit einer Folge \((c_k)_{k\in\MdZ}\) in \(\mdc\):
\(f\overset{\|\cdot\|_2}=\sum_{k\in\MdZ}c_kb_k \), so gilt:
\[c_k=(f\mid b_k) \text{ für alle } k\in\MdZ\]
\end{satz}
@ -139,7 +139,7 @@ Ist \(f\in L^2\) und gilt mit einer Folge \((c_k)_{k\in\MdZ}\) in \(\mdc\):
\begin{beweis}
Für \(n\in\mdn_0\) setze \[\sigma_n:=\sum_{\lvert k\rvert\leq n}c_kb_k\] Aus der Voraussetzung folgt
\(\| \sigma_n-f\|_2\to 0\) für \(n\to\infty\). Sei \(j\in\MdZ\) und \(n\in\mdn\) mit
\(n\geq \lvert j\rvert\). Es gilt einerseits
\(n\geq \lvert j\rvert\). Es gilt einerseits
\[(\sigma_n\mid b_j) = \sum_{\lvert k\rvert\leq n}c_k(b_k\mid b_j)=c_j, \text{ da gilt: }
(b_k\mid b_j)=
\begin{cases}
@ -163,11 +163,11 @@ Sei \(f\in L^2\), \(n\in\mdn_0\) und \(k\in\MdZ\).
\to0\]
\item \((f\mid b_k)\) heißt \textbf{k-ter Fourierkoeffizient von f}.
\item \(\sum_{k\in\MdZ}(f\mid b_k)b_k\) heißt \textbf{Fourierreihe von f}.
\item Für \(n_0\in\mdn_0\) setze
\(E_n:=[b_{-n},b_{-(n-1)},\dots,b_0,b_1,\dots,b_n]\)
\item Für \(n_0\in\mdn_0\) setze
\(E_n:=[b_{-n},b_{-(n-1)},\dots,b_0,b_1,\dots,b_n]\)
(lineare Hülle). Es ist dann \[\dim E_n=2n+1\]
\textbf{Beachte: } Für \(v\in E_n\) gilt \(v(0)=v(2\pi)\).
\end{enumerate}
\end{enumerate}
\end{definition}
\begin{satz}
@ -177,7 +177,7 @@ Sei \(f\in L^2\), \(n\in\mdn_0\) und \(k\in\MdZ\).
Seien \(f_1,\dots,f_n,f\in L^2\).
\begin{enumerate}
\item Gilt \(f_\mu\perp f_\nu\) für \(\mu\neq\nu\) (\(\mu,\nu=1,\dots,n\)),
so gilt der Satz des Pythagoras
so gilt der Satz des Pythagoras
\[\| f_1+\dots+f_n\|^2_2=
\| f_1\|^2_2+\dots+
\| f_n\|^2_2\]
@ -186,15 +186,15 @@ Seien \(f_1,\dots,f_n,f\in L^2\).
L^2\to E_n\\
S_nf:=\sum_{\lvert k\rvert\leq n}(f\mid b_k)b_k
\end{cases}\]
ist linear und für jedes \(v\in E_n\) gilt \(S_nv=v\) und
ist linear und für jedes \(v\in E_n\) gilt \(S_nv=v\) und
\((f-S_nf)\perp v\) mit \(f\in L^2\).
\item Die \textbf{Besselsche Ungleichung} lautet:
\[\| S_nf\|^2_2
=\sum_{\lvert k\rvert\leq n}\lvert(f\mid b_k)\rvert^2
=\| f\|_2^2-\|(f-S_nf)\|^2_2
\leq\| f\|^2_2\]
\item Für alle \(v\in E_n\) gilt:
\[\| f-S_nf\|_2\leq\| f-v\|_2
\item Für alle \(v\in E_n\) gilt:
\[\| f-S_nf\|_2\leq\| f-v\|_2
\]
\end{enumerate}
\end{satz}
@ -207,26 +207,26 @@ Seien \(f_1,\dots,f_n,f\in L^2\).
&= (f_1+f_2\mid f_1+f_2) \\
&= (f_1\mid f_1)+(f_1\mid f_2)+(f_2\mid f_1)+(f_2\mid f_2) \\
&= (f_1\mid f_1)+(f_2\mid f_2) \\
&=\| f_1\|^2_2+\| f_2\|^2_2
&=\| f_1\|^2_2+\| f_2\|^2_2
\end{align*}
\item Übung!
\item Es gilt
\begin{align*}
\| S_nf\|^2_2
&= \left\lvert\left\lvert\sum_{\lvert k\rvert\leq n}(f\mid b_k)b_k\right\rvert
\right\rvert^2_2
\right\rvert^2_2
\overset{(1)}=
\sum_{\lvert k\rvert\leq n}\|(f\mid b_k)b_k\rvert
\rvert^2_2
\rvert^2_2
= \sum_{\lvert k\rvert\leq n}\lvert(f\mid b_k)\rvert^2\| b_k\rvert
\rvert^2_2
\rvert^2_2
= \sum_{\lvert k\rvert\leq n}\lvert(f\mid b_k)\rvert^2
\end{align*}
und
\begin{align*}
\| f\|^2_2
\| f\|^2_2
= \|\underbrace{(f-S_nf)}_{\underset{(2)}\perp E_n}
+\underbrace{S_nf}_{\in E_n}\|^2_2
+\underbrace{S_nf}_{\in E_n}\|^2_2
= \| f-S_nf\|^2_2 + \| S_nf\|^2_2
\end{align*}
\item Sei \(v\in E_n\). Dann gilt:
@ -244,17 +244,17 @@ Seien \(f_1,\dots,f_n,f\in L^2\).
\begin{wichtigebemerkung}
\label{Bemerkung 18.3}
Es sei \(\mdk\in\{\mdr,\mdc\},\,a,b\in\mdr,\,I:=[a,b]\,(a<b)\) und \(f_{n},\,f,\,g\in C(I,\mdk)\); es war
Es sei \(\mdk\in\{\mdr,\mdc\},\,a,b\in\mdr,\,I:=[a,b]\,(a<b)\) und \(f_{n},\,f,\,g\in C(I,\mdk)\); es war
\(\lVert f\rVert_{\infty}:=\max_{t\in I}\lvert f(t)\rvert\).
\begin{enumerate}
\item \((f_{n})\) konvergiert auf \(I\) gleichmäßig gegen \(f\) genau dann, wenn
\item \((f_{n})\) konvergiert auf \(I\) gleichmäßig gegen \(f\) genau dann, wenn
\(\lVert f_{n}-f\rVert_{\infty}\to 0\,(n\to\infty)\) (vgl. Analysis I/II).
\item \(f\in\mathrm{L}^{p}(I,\mdk)\) und \(\lVert f\rVert_{p}\leq(b-a)^{\frac{1}{p}}\lVert f\rVert_{\infty}\) (siehe \ref{Satz 16.2}).
\item Gilt \(f=g\) fast überall, so ist \(f=g\) auf \(I\).
\begin{beweis}
Es existiert eine Nullmenge \(N\subseteq I:\,f(x)=g(x)\,\forall x\in I\setminus N\).\\
Sei \(x_{0}\in\mdn\). Für \(\ep>0\) gilt: \(U_{\ep}(x_{0})\cap I\not\subseteq N\) (andernfalls:
\(\lambda_{1}(N)\geq\lambda_{1}(U_{\ep}(x_{0})\cap I)>0\)). Das heißt, es existiert ein
Sei \(x_{0}\in\mdn\). Für \(\ep>0\) gilt: \(U_{\ep}(x_{0})\cap I\not\subseteq N\) (andernfalls:
\(\lambda_{1}(N)\geq\lambda_{1}(U_{\ep}(x_{0})\cap I)>0\)). Das heißt, es existiert ein
\(x_{\ep}\in U_{\ep}(x_{0})\cap I:\,x_{\ep}\not\in N\). Also:
\(\forall n\in\mdn\,\exists x_{n}\in U_{\frac{1}{n}}(x_{0})\cap I:\, x_{n}\not\in N\). Also: \(x_{n}\to x_{0}\).\\
Dann: \(f(x_{0})=\lim_{n\to\infty}f(x_{n})=\lim_{n\to\infty}g(x_{n})=g(x_{0})\)
@ -287,9 +287,9 @@ Sei \(f\in\mathrm{L}^{2}\). Dann gilt: \(f\overset{\lVert\cdot\rVert_{2}}{=}\sum
\begin{beweis}
Zu zeigen: \(\lVert f-S_{n}f\rVert_{2}\to0\,(n\to\infty)\). Die Parsevalsche Gleichung folgt dann aus \ref{Satz 18.2}.\\
Sei \(\ep>0\). Wende \ref{Satz 16.8}(2) auf \(\Re f\) und \(\Im f\) an. Dies liefert eine stetige Funktion
Sei \(\ep>0\). Wende \ref{Satz 16.8}(2) auf \(\Re f\) und \(\Im f\) an. Dies liefert eine stetige Funktion
\(g:\,(0,2\pi)\to\mdc\) mit: \(K:=\supp(g)\subseteq(0,2\pi)\), \(K\) kompakt und \(\lVert f-g\rVert_{2}<\ep\).\\
Setze \(g(0):=g(2\pi):=0\). Dann ist \(g\) stetig auf \([0,2\pi]\). Satz \ref{Satz 18.4} liefert nun:
Setze \(g(0):=g(2\pi):=0\). Dann ist \(g\) stetig auf \([0,2\pi]\). Satz \ref{Satz 18.4} liefert nun:
\(\exists n\in\mdn\exists v\in\mathrm{E}_{n}:\,\lVert g-v\rVert_{\infty}<\ep\).\\
Damit: \(\lVert g-v\rVert_{2}\leq\sqrt{2\pi}\lVert g-v\rVert_{\infty}<\sqrt{2\pi}\ep\). Somit:
\begin{align*}
@ -344,9 +344,9 @@ Fourierreihe von \(f\) ist eine \textbf{Sinusreihe}.
\begin{enumerate}
\item \(f(t):=\begin{cases}1,&0\leq t\leq\pi\\-1,&\pi<t\leq 2\pi\end{cases}\)
\(f\) ist ungerade, also \(\alpha_{k}=0\,\forall k\in\mdn_{0}\). Es ist
\(f\) ist ungerade, also \(\alpha_{k}=0\,\forall k\in\mdn_{0}\). Es ist
\(\beta_{k}=\frac{2}{\pi}\int_{0}^{\pi}{\sin(kt)\mathrm{d}t}=\begin{cases}0,&k\text{ gerade}\\\frac{4}{k\pi},&k\text{ ungerade}\end{cases}\).\\
Damit:
Damit:
\[
f\overset{\lVert\cdot\rVert_{2}}{=}\frac{4}{\pi}\sum_{j=0}^{\infty}{\frac{\sin((2j+1)\cdot)}{2j+1}}
\]

View file

@ -3,7 +3,7 @@
In diesem Kapitel sei \(X\) eine Menge, \(X\neq\emptyset\).
\begin{definition}
\index{Ring}
Sei \(\emptyset\neq \fr \subseteq \cp(X)\).
Sei \(\emptyset\neq \fr \subseteq \cp(X)\).
$\fr$ heißt ein \textbf{Ring} auf \(X\), genau dann wenn gilt:
\begin{enumerate}
\item[(R1)] \(\emptyset \in \fr\)
@ -12,7 +12,7 @@ In diesem Kapitel sei \(X\) eine Menge, \(X\neq\emptyset\).
\end{definition}
\textbf{Hinweis}: $(\fr, \cup, \setminus)$ ist kein Ring im Sinne
der linearen Algebra, $(\fr, \cup)$ kein Inverses Element hat und
der linearen Algebra, $(\fr, \cup)$ kein Inverses Element hat und
$(\fr, \cup)$ nicht kommutativ ist.
\begin{definition}
@ -21,7 +21,7 @@ $(\fr, \cup)$ nicht kommutativ ist.
Sei \(d\in\MdN\).
\begin{enumerate}
\item \(\ci_d :=\Set{(a,b] | a,b \in \MdR^{d}, \, a \leq b} (\emptyset \in \ci_d)\).
Seien \(a=(a_{1},\dots,a_{d}),\,b=(b_{1},\dots,b_{d})\in\MdR^d\)
Seien \(a=(a_{1},\dots,a_{d}),\,b=(b_{1},\dots,b_{d})\in\MdR^d\)
und \(I:=(a,b] \in \ci_{d}\)
\[
\lambda_{d}(I)= \begin{cases}
@ -31,7 +31,7 @@ $(\fr, \cup)$ nicht kommutativ ist.
\item \(\cf_d:=\Set{\bigcup_{j=1}^{n}I_{j} | n\in\MdN,\,I_{1},\dots,I_{n}\in \ci_d}\) (\textbf{Menge der Figuren})
\end{enumerate}
\end{definition}
Ziel dieses Kapitels: Fortsetzung von \(\lambda_{d}\) auf \(\cf_{d}\)
Ziel dieses Kapitels: Fortsetzung von \(\lambda_{d}\) auf \(\cf_{d}\)
und dann auf \(\fb_d\) (\(\leadsto\) Lebesgue-Maß)
Beachte: \(\ci_{d}\subseteq\cf_{d}\subseteq\fb_{d}\overset{1.4}{\implies}\fb_{d}=\sigma(\ci_{d})=\sigma(\cf_{d})\)
@ -40,7 +40,7 @@ Beachte: \(\ci_{d}\subseteq\cf_{d}\subseteq\fb_{d}\overset{1.4}{\implies}\fb_{d}
Seien \(I,I'\in\ci_{d}\) und \(A\in\cf_{d}\). Dann:
\begin{enumerate}
\item \(I\cap I'\in\ci_{d}\)
\item \(I\setminus I'\in\cf_{d}.\)
\item \(I\setminus I'\in\cf_{d}.\)
Genauer: \(\exists\left\{I_{1}',\dots,I_{l}'\right\}\subseteq\ci_{d}\) disjunkt:
\(I\setminus I'=\bigcup_{j=1}^{l}{I_{j}'}\) % \bigcupdot
\item \(\exists\left\{I_{1}',\dots,I_{l}'\right\}\subseteq\ci_{d}\) disjunkt: \(A=\bigcup_{j=1}^{l}{I_{j}'}\)
@ -67,7 +67,7 @@ Beachte: \(\ci_{d}\subseteq\cf_{d}\subseteq\fb_{d}\overset{1.4}{\implies}\fb_{d}
\(I=I_{1}\times I_{2},\,I'=I_{1}'\times I_{2}'\)
% Graphik einfuegen!
Nachrechnen:
Nachrechnen:
\[
I\setminus I'=(I_{1}\setminus I_{1}')\times I_{2}\dot \cup(I_{1}\cap I_{1}')\times(I_{2}\setminus I_{2}')
\]
@ -76,11 +76,11 @@ Beachte: \(\ci_{d}\subseteq\cf_{d}\subseteq\fb_{d}\overset{1.4}{\implies}\fb_{d}
Daraus folgt die Behauptung für \(d+1\)
\end{itemize}
\item \begin{itemize}
\item[\underline{Vor.:}] Sei $n \in \mdn$ und
\(A=\bigcup_{j=1}^{n}{I_{j}}\) mit
\(I_{1},\dots,I_{d}\in\ci_{d}\)
\item[\underline{Vor.:}] Sei $n \in \mdn$ und
\(A=\bigcup_{j=1}^{n}{I_{j}}\) mit
\(I_{1},\dots,I_{d}\in\ci_{d}\)
\item[\underline{Beh.:}] Es existiert
\(\{I_{1}',\dots,I_{l}'\}\subseteq\ci_{d}\) disjunkt:
\(\{I_{1}',\dots,I_{l}'\}\subseteq\ci_{d}\) disjunkt:
\(A=\bigcup_{j=1}^{l}{I_{j}'}\)
\item[\underline{Bew.:}] mit Induktion nach $n$:
\begin{itemize}
@ -93,7 +93,7 @@ Beachte: \(\ci_{d}\subseteq\cf_{d}\subseteq\fb_{d}\overset{1.4}{\implies}\fb_{d}
Dann: \(A=I_{n+1}\cup\bigcup_{j=1}^{l}{I_{j}'}=I_{n+1}\cup\bigcup_{j=1}^{l}{(I_{j}'\setminus I_{n+1})}\) % \cupdot
Wende (2) auf jedes \(I_{j}'\setminus I_{n+1}\) an \((j=1,\dots,l)\):
Wende (2) auf jedes \(I_{j}'\setminus I_{n+1}\) an \((j=1,\dots,l)\):
\(I_{j}'\setminus I_{n+1}=\bigcup_{j=1}^{l_{j}}{I_{j}''}\quad(I_{j}''\in\ci_{d})\)
Damit folgt:
@ -124,14 +124,14 @@ ohne Beweis:
\begin{lemma}[Unabhängigkeit von der Darstellung]
\label{Lemma 2.2}
Sei \(A\in\cf_{d}\) und \(\{I_{1},\dots,I_{n}\}\subseteq\ci_{d}\) disjunkt und
\(\{I_{1}',\dots,I_{m}'\}\subseteq\ci_{d}\) disjunkt mit
\(\{I_{1}',\dots,I_{m}'\}\subseteq\ci_{d}\) disjunkt mit
\(\bigcup_{j=1}^{n}{I_{j}}=A=\bigcup_{j=1}^{m}{I_{j}'}\). Dann:
\[
\sum_{j=1}^{n}{\lambda_{d}(I_{j})}=\sum_{j=1}^{m}{\lambda_{d}(I_{j}')}
\]
\end{lemma}
\begin{definition}
Sei \(A\in\cf_{d}\) und \(A=\bigcup_{j=1}^{n}{I_{j}}\) mit
Sei \(A\in\cf_{d}\) und \(A=\bigcup_{j=1}^{n}{I_{j}}\) mit
\(\{I_{1},\dots,I_{n}\}\subseteq\ci_{d}\)
disjunkt (beachte Lemma \ref{Lemma 2.1}, Punkt 3).
\[
@ -147,23 +147,23 @@ ohne Beweis:
\item \(A\cap B=\emptyset\implies\lambda_{d}(A\cup B)=\lambda_{d}(A)+\lambda_{d}(B)\)
\item \(A\subseteq B\implies\lambda_{d}(A)\leq\lambda_{d}(B)\)
\item \(\lambda_{d}(A\cup B)\leq\lambda_{d}(A)+\lambda_{d}(B)\)
\item Sei \(\delta>0\). Es existiert \(C\in\cf_{d}:\overline{C}\subseteq B\)
\item Sei \(\delta>0\). Es existiert \(C\in\cf_{d}:\overline{C}\subseteq B\)
und \(\lambda_{d}(B\setminus C)\leq\delta\).
\item Ist \(B_{n+1}\subseteq B_{n}\forall n\in\mdn\) und
\(\bigcap B_{n}=\emptyset\), so gilt:
\item Ist \(B_{n+1}\subseteq B_{n}\forall n\in\mdn\) und
\(\bigcap B_{n}=\emptyset\), so gilt:
\(\lambda_{d}(B_{n})\to 0\,(n\to \infty)\)
\end{enumerate}
\end{satz}
\begin{beweis}
\begin{enumerate}
\item Aus Lemma \ref{Lemma 2.1} folgt: Es existiert
\item Aus Lemma \ref{Lemma 2.1} folgt: Es existiert
\(\{I_{1},\dots,I_{n}\}\subseteq\ci_{d}\)
disjunkt und es existiert \(\{I_{1}',\dots,I_{m}'\}\subseteq\ci_{d}\) disjunkt:
\(A=\bigcup_{j=1}^{n}{I_{j}},\,B=\bigcup_{j=1}^{m}{I_{j}'}\).
\(J:=\{I_{1},\dots,I_{n},I_{1}',\dots,I_{m}'\}\subseteq\ci_{d}\). Aus
\(A\cap B=\emptyset\) folgt: \(J\) ist disjunkt. Dann:
\(J:=\{I_{1},\dots,I_{n},I_{1}',\dots,I_{m}'\}\subseteq\ci_{d}\). Aus
\(A\cap B=\emptyset\) folgt: \(J\) ist disjunkt. Dann:
\(A\cup B=\bigcup_{I\in J}{I}\) % Hier auch wieder: \bigcupdot
Also:
@ -187,7 +187,7 @@ Dann:
Aus der Definition von Kompaktheit (Analysis II, \S 2) folgt:
\(\exists m\in\mdn:\,\bigcup_{j=1}^{m}{\overline{C}_{j}^{c}}\supseteq\overline{B}_{1}\)
Dann: \(\bigcap_{j=1}^{m}{\overline{C}_{j}}\subseteq\overline{B}_{1}^{c}\).
Andererseits: \(\bigcap_{j=1}^{m}{\overline{C}_{j}}\subseteq\bigcap_{j=1}^{m}{B_{j}}\subseteq B_{1}\subseteq\overline{B}_{1}\).
Andererseits: \(\bigcap_{j=1}^{m}{\overline{C}_{j}}\subseteq\bigcap_{j=1}^{m}{B_{j}}\subseteq B_{1}\subseteq\overline{B}_{1}\).
Also: \(\bigcap_{j=1}^{m}{\overline{C}_{j}}=\emptyset\). Das heißt:
\(\bigcap_{j=1}^{n}{\overline{C}_{j}}=\emptyset \quad \forall n\geq m\)
@ -198,7 +198,7 @@ Also: \(\bigcap_{j=1}^{m}{\overline{C}_{j}}=\emptyset\). Das heißt:
\begin{beweis} (induktiv)
\begin{itemize}
\item[I.A.] \(\lambda_{d}(B_{1}\setminus D_{1})=\lambda_{d}(B_{1}\setminus C_{1})\overset{\eqref{eq: Abschaetzung Mass -- Beweis Satz 2.3.(5)}}{\leq}\frac{\ep}{2}=\left(1-\frac{1}{2}\right)\ep\) \checkmark
\item[I.V.] Sei \(n\in\mdn\) und es gelte
\item[I.V.] Sei \(n\in\mdn\) und es gelte
$\lambda_{d}(B_{n}\setminus D_{n})\leq\left(1-\frac{1}{2^{n}}\right)\ep$
\item[I.S.] \begin{align*}
\lambda_{d}(B_{n+1}\setminus D_{n+1})&=\lambda_{d}\left((B_{n+1}\setminus D_{n})\cup(B_{n+1}\setminus C_{n+1})\right)\\
@ -216,7 +216,7 @@ Für \(n\geq m:\,D_{n}=\emptyset\,\implies\,\lambda_{d}(B_{n})=\lambda_{d}(B_{n}
\begin{definition}
\index{Prämaß}
Es sei \(\fr\) ein Ring auf \(X\). Eine Abbildung \(\mu:\fr\to[0,\infty]\)
Es sei \(\fr\) ein Ring auf \(X\). Eine Abbildung \(\mu:\fr\to[0,\infty]\)
heißt ein \textbf{Prämaß} \ auf \(\fr\), wenn gilt:
\begin{enumerate}
\item \(\mu(\emptyset)=0\)
@ -240,7 +240,7 @@ Für \(n\geq 2\):
\[
\lambda_{d}(A)=\lambda_{d}(A_{1}\cup\dots\cup A_{n-1}\cup B_{n})\overset{\ref{Satz 2.3}.(1)}{=}\sum_{j=1}^{n-1}{\lambda_{d}(A_{j})}+\lambda_{d}(B_{n})
\]
Daraus folgt:
Daraus folgt:
\[
\sum_{j=1}^{n-1}{\lambda_{d}(A_{j})}=\lambda_{d}(A)-\lambda_{d}(B_{n})\quad\forall n\geq 2
\]
@ -268,7 +268,7 @@ Sei \(\emptyset\neq\ce\subseteq\cp(X)\), es seien \(\nu,\,\mu\) Maße auf
Es gelte:
\begin{enumerate}
\item \(E,F\in\ce\implies E\cap F\in\ce\quad\text{(durchschnittstabil)}\)
\item $\exists$ eine Folge \((E_{n})\) in \(\ce\): \(\bigcup{E_{n}}=X\)
\item $\exists$ eine Folge \((E_{n})\) in \(\ce\): \(\bigcup{E_{n}}=X\)
und \(\mu(E_{n})<\infty \quad \forall n\in\mdn\).
\item \(\mu(E)=\nu(E) \quad \forall E\in\ce\)
\end{enumerate}
@ -286,18 +286,18 @@ und wird ebenfalls mit \(\lambda_{d}\) bezeichnet.
\folgtnach{(\ref{Lemma 2.1}) und (\ref{Satz 2.4})}: \(\lambda_{d}\) ist ein
Prämaß\ auf \(\fr:=\cf_{d}\); es ist \(\sigma(\fr)=\fb_{d}\).
\folgtnach{\ref{Satz 2.5}}: \(\lambda_{d}\) kann zu einem Maß auf
\(\sigma(\cf_{d}) = \fb_{d}\) fortgesetzt werden. Für diese
\folgtnach{\ref{Satz 2.5}}: \(\lambda_{d}\) kann zu einem Maß auf
\(\sigma(\cf_{d}) = \fb_{d}\) fortgesetzt werden. Für diese
Fortsetzung schreiben wir wieder $\lambda_d$, also
$\lambda_d: \fb_{d} \rightarrow [0, +\infty]$
Sei \(\nu\) ein weiteres Maß\ auf \(\fb_{d}\) mit:
Sei \(\nu\) ein weiteres Maß\ auf \(\fb_{d}\) mit:
\(\nu(A)=\lambda_{d}(A)\,\forall A\in\cf_{d}\). \(\ce:=\ci_{d}\). Dann:
\(\sigma(\ce)\overset{\ref{Satz 1.4}}{=}\fb_{d}\).
\begin{enumerate}
\item \(E,F\in\ce\overset{\ref{Lemma 2.1}}{\implies}E\cap F\in\ce\)
\item \(E_{n}:=(-n,n]^{d}\)
Klar:
Klar:
\begin{align*}
\bigcup E_{n}&=\mdr^{d}\\
\lambda_{d}(E_{n})&=(2n)^{d}<\infty
@ -426,9 +426,9 @@ Also auch:
\end{beweis}
\textbf{Auswahlaxiom:}\\
Sei $\emptyset\ne\Omega$ Indexmenge, es sei $\Set{X_\omega | \omega\in\Omega}$
ein disjunktes System von nichtleeren Mengen $X_\omega$. Dann
existiert ein $C\subseteq\bigcup_{\omega\in\Omega}X_\omega$, sodass
Sei $\emptyset\ne\Omega$ Indexmenge, es sei $\Set{X_\omega | \omega\in\Omega}$
ein disjunktes System von nichtleeren Mengen $X_\omega$. Dann
existiert ein $C\subseteq\bigcup_{\omega\in\Omega}X_\omega$, sodass
$C$ mit jedem $X_j$ genau ein Element gemeinsam hat.
\begin{satz}[Satz von Vitali]
@ -450,7 +450,7 @@ Es ist $\mdq^d\cap[-1,1]^d=\{q_1,q_2,\dots\}$ mit $q_i\ne q_j$ für $(i\ne j)$.
\end{align*}
\begin{beweis}
Sei $x\in[0,1]^d$. Wähle $y\in C$ mit $y\in[x]$, dann ist $x\sim y$, also $x-y\in\mdq^d\cap[-1,1]^d$. D.h.:
\[\exists n\in\mdn: x-y=q_n\implies x=q_n+y\in q_n+C\]
\[\exists n\in\mdn: x-y=q_n\implies x=q_n+y\in q_n+C\]
\end{beweis}
Außerdem ist $\Set{q_n+C | n\in\mdn}$ disjunkt.
\begin{beweis}

View file

@ -18,8 +18,8 @@ Seien die Bezeichnungen wie in obiger Definition, dann gilt:
\begin{enumerate}
\item $f$ sei $\fa$-$\fb$-messbar, $\fa'$ eine weitere $\sigma$-Algebra auf $X$ mit $\fa\subseteq\fa'$ und $\fb'$ sei eine $\sigma$-Algebra auf $Y$ mit $\fb'\subseteq\fb$.\\
Dann ist $f$ $\fa'$-$\fb'$-messbar.
\item Sei $X_0\in\fa$, dann gilt $\fa_{X_0}\subseteq\fa$ nach
\ref{Satz 1.5}. Nun sei $f:X\to Y$ $\fa$-$\fb$-messbar, dann ist
\item Sei $X_0\in\fa$, dann gilt $\fa_{X_0}\subseteq\fa$ nach
\ref{Satz 1.5}. Nun sei $f:X\to Y$ $\fa$-$\fb$-messbar, dann ist
$f_{\mid X_0}:X_0\to Y$ $\fa_{X_0}$-$\fb$-messbar.
\end{enumerate}
\end{bemerkung}
@ -66,7 +66,7 @@ Dann: \(\fb=\sigma(\ce)\subseteq\fd\). Ist \(B\in\fb\), so ist \(B\in\fd\), also
\index{messbar!Borel}\index{messbar}
Sei \(X\in\fb_{d}\). Ist \(f:\,X\to\mdr^{k}\) \(\fb(X)-\fb_{k}-\)messbar, so heißt \(f\) \textbf{(Borel-)messbar}.
\end{definition}
Ab jetzt sei stets \(\emptyset \neq X\in\fb_{d}\).
Ab jetzt sei stets \(\emptyset \neq X\in\fb_{d}\).
(Erinnerung: \(\fb(X)=\Set{A\in\fb_{d} | A\subseteq X}\))
\begin{satz}
@ -80,7 +80,7 @@ Seien \(f,\,g:\,X\to\mdr^{k}\) Abbildungen und \(\alpha,\beta\in\mdr\).
\item Sei \(k=1\) und \(f\) und \(g\) seien messbar. Dann:
\begin{enumerate}
\item \(f \cdot g\) ist messbar
\item Ist \(f(x)\neq 0 \quad \forall x\in X\), so ist
\item Ist \(f(x)\neq 0 \quad \forall x\in X\), so ist
\(\frac{1}{f}\) messbar
\item \(\Set{x\in X | f(x)\stackrel{>}{\geq} g(x)} \in \fb(X)\)
\end{enumerate}
@ -96,13 +96,13 @@ Seien \(f,\,g:\,X\to\mdr^{k}\) Abbildungen und \(\alpha,\beta\in\mdr\).
stetig, also messbar.
Es ist \(g=\vp\circ f\). \folgtnach{\ref{Satz 3.1}.(1)} \(g\) ist messbar.
\item
\item
\begin{itemize}
\item["`\(\Rightarrow:\)"'] Für \(j=1, \dots,k\) sei
\(p_{j}:\mdr^{k}\to\mdr\) definiert durch
\item["`\(\Rightarrow:\)"'] Für \(j=1, \dots,k\) sei
\(p_{j}:\mdr^{k}\to\mdr\) definiert durch
\(p_{j}(x_{1},\dots,x_{k}):=x_{j}\)
\(p_{j}\) ist stetig, also messbar. Es ist
\(f_{j}=p_{j}\circ f\) \folgtnach{\ref{Satz 3.1}.(1)}
\(p_{j}\) ist stetig, also messbar. Es ist
\(f_{j}=p_{j}\circ f\) \folgtnach{\ref{Satz 3.1}.(1)}
\(f_{j}\) ist messbar.
\item["`\(\Leftarrow:\)"'] Sei \(I=(a,b]=\prod_{j=1}^{k}{(a_{j},b_{j}]}\in I_{k}\quad (a=(a_{1},\dots,a_{k}),\,b=(b_{1},\dots,b_{k}),\,a\leq b)\)\\
Dann: \(f^{-1}(I)=\bigcap_{j=1}^{k}{\underbrace{f_{j}^{-1}(\underbrace{(a_{j},b_{j}]}_{\in\fb_{1}}}_{\in\fb(X)}}\in\fb(X)\)
@ -114,7 +114,7 @@ Es ist \(g=\vp\circ f\). \folgtnach{\ref{Satz 3.1}.(1)} \(g\) ist messbar.
\(\vp\) ist stetig, also messbar. Es ist \(\alpha f+\beta g=\vp\circ h\)
\folgtnach{\ref{Satz 3.1}.(1)} \(\alpha f+\beta g\) ist messbar.
\item
\item
\begin{enumerate}
\item \(h:=(f,g):\,X\to\mdr^{2k}\) ist messbar (nach (2)); \(\vp(x,y):=xy\), \(\vp\) ist stetig, also messbar.
@ -130,10 +130,10 @@ Es ist \(fg=\vp\circ h\) \folgtnach{\ref{Satz 3.1}.(1)} \(fg\) ist messbar.
\begin{folgerungen}
\label{Lemma 3.3}
Seien \(A,\,B\in\fb(X),\,A\cap B=\emptyset\) und \(X=A\cup B\).
Seien \(A,\,B\in\fb(X),\,A\cap B=\emptyset\) und \(X=A\cup B\).
Weiter seien \(f:A\to\mdr^{k}\) und
\(g:B\to\mdr^{k}\) messbar.\\
Dann ist \(h:X\to\mdr^{k}\), definiert durch
Dann ist \(h:X\to\mdr^{k}\), definiert durch
\[
h(x):=\begin{cases}f(x)&x\in A\\g(x)&x\in B\end{cases},
\]
@ -184,7 +184,7 @@ In \(\imdr\) gelten folgende Regeln, wobei \(a\in\mdr\):
\begin{definition}
\begin{enumerate}
\item Sei \((x_{n})\) eine Folge in
\item Sei \((x_{n})\) eine Folge in
\(\imdr\). \(x_{n}\rightarrow+\infty:\Leftrightarrow\forall c\in\mdr\,\exists n_{c}\in\mdn:x_{n}\geq c\quad\forall n\geq n_{c}\)\\
Analog für \(-\infty\).
\item Seien \(f,g: X\to\imdr\) Funktionen. Dann:
@ -208,7 +208,7 @@ Analog für \(-\infty\).
\begin{definition}
\index{Borel!$\sigma$-Algebra}\index{messbar}
\(\ifb_{1}:=\Set{B\cup E | B\in\fb_{1},\,E\subseteq\Set{-\infty,+\infty}}\).
\(\ifb_{1}:=\Set{B\cup E | B\in\fb_{1},\,E\subseteq\Set{-\infty,+\infty}}\).
Dann: \(\fb_{1}\subseteq\ifb_{1}\)\\
Übung: \(\ifb_{1}\) ist eine \(\sigma\)-Algebra auf \(\imdr\).\\
Klar: \(\fb_{1} \subseteq \ifb_{1}\)
@ -258,11 +258,11 @@ Die folgenden Beweise erfolgen exemplarisch für einen der Unterpunkte und funkt
\item Es gilt:
\[\forall a \in \mdq\colon \{f\le a\}=\Set{x\in X | f(x)\le a}=f^{-1}(\underbrace{[-\infty,a]}_{\ce_1}) (*)\]
Die Äquivalenz folgt dann aus (1) und \ref{Satz 3.1}.
\item Die Funktion $f:X\to\imdr$ kann aufgefasst werden als Funktion $\overline{f}:X\to\imdr$. Es ist $f$ genau dann $\fb(X)$-$\fb_1$-messbar wenn $\overline{f}$ $\fb(X)$-$\overline{\fb_1}$-messbar ist.
\item Die Funktion $f:X\to\imdr$ kann aufgefasst werden als Funktion $\overline{f}:X\to\imdr$. Es ist $f$ genau dann $\fb(X)$-$\fb_1$-messbar wenn $\overline{f}$ $\fb(X)$-$\overline{\fb_1}$-messbar ist.
\end{enumerate}
\end{beweis}
\begin{bemerkung}\
\begin{bemerkung}\
\begin{enumerate}
\item Ist $X \subseteq \mdr$ ein Intervall und $f: \bar X \rightarrow \mdr$ monoton, so ist
$f$ messbar (vgl. 3. ÜB)
@ -284,11 +284,11 @@ Es ist $|f(x)|=1 \quad \forall x \in \mdr^d$, also $|f| = \mathds{1}_{\mdr^d}$.
\begin{definition}
Sei $M\subseteq\imdr$.
\begin{enumerate}
\item Ist $M=\emptyset$ oder $M=\{-\infty\}$, so sei
\item Ist $M=\emptyset$ oder $M=\{-\infty\}$, so sei
\[\sup M:=-\infty\]
\item Ist $M\setminus\{-\infty\}\ne\emptyset$ und nach oben beschränkt (also insbesondere $\infty\not\in M$), so sei
\item Ist $M\setminus\{-\infty\}\ne\emptyset$ und nach oben beschränkt (also insbesondere $\infty\not\in M$), so sei
\[\sup M:= \sup (M\setminus\{-\infty\})\]
\item Ist $M\setminus\{-\infty\}$ nicht nach oben beschränkt oder $\infty\in M$, so sei
\item Ist $M\setminus\{-\infty\}$ nicht nach oben beschränkt oder $\infty\in M$, so sei
\[\sup M:=\infty\]
\item Es sei $\inf M:=-\sup(-M)$, wobei $-M:=\Set{-m | m\in M}$.
\end{enumerate}
@ -343,7 +343,7 @@ Also ist $\sup_{n\in\mdn} f_n$ messbar. Analog lässt sich die Messbarkeit von $
Sei $X=I$ ein Intervall in $\mdr$ und $f:I\to\mdr$ sei auf $I$ differenzierbar.\\
Für $x\in I,n\in\mdn$ sei $f_n:= n(f(x-\frac1n)-f(x))$. Da $f$ stetig ist, ist auch jedes $f_n$ stetig, also insbesondere messbar und es gilt:
\[f_n(x)=\frac{f(x-\frac1n)-f(x)}{\frac1n}\stackrel{n\to\infty}{\to}f'(x)\]
Aus \ref{Satz 3.5}(2) folgt, dass $f'$ messbar ist.
Aus \ref{Satz 3.5}(2) folgt, dass $f'$ messbar ist.
\end{beispiel}
\begin{definition}
@ -435,13 +435,13 @@ Sei $f:X\to\imdr$ eine Funktion, dann ist $f$ genau dann messbar, wenn eine Folg
Dann ist $\varphi_n$ $(\fb_1)_{[0,\infty]}$-$\fb_1$-messbar, außerdem gilt:
\begin{align*}
\forall t\in[0,\infty]\forall n\in\mdn&: 0\le\varphi_1\le\dots\le t\\
\forall t\in[0,n]\forall n\in\mdn&: t-\frac1{2^n}\le\varphi_n(t)\le t
\forall t\in[0,n]\forall n\in\mdn&: t-\frac1{2^n}\le\varphi_n(t)\le t
\end{align*}
und es ist $\varphi_n(t)\stackrel{n\to\infty}\to t$ für alle $t\in[0\infty]$. Setze $f_n:=\varphi_n\circ f$. Dann leistet $(f_n)$ das gewünschte.
\item Es ist $f=f_+-f_-$ und $f_+,f_-\ge0$ auf $X$. Seien $(g_n),(h_n)$ zulässige Folgen für $f_+$ bzw. $f_-$. Definiere $f_n:=g_n-h_n$. Dann ist klar, dass gilt:
\[\forall x\in X: f_n(x)=g_n(x)-h_n(x)\stackrel{n\to\infty}\to f_+(x)-f_-(x)=f(x)\]
Weiter gilt:
\[|f_n|\le g_n+h_n\le f_++f_-=|f|\]
\item Ohne Beweis.
\item Ohne Beweis.
\end{enumerate}
\end{beweis}

View file

@ -53,7 +53,7 @@ Sei $f:X\to[0,\infty]$ messbar. $(f_n)$ sei eine für $f$ zulässige Folge. Das
\end{align*}
\end{definition}
\begin{bemerkung}\
\begin{bemerkung}\
\begin{enumerate}
\item In \ref{Satz 4.3} werden wir sehen, dass $(*)$ unabhängig ist von der Wahl der für $f$ zulässigen Folge $(f_n)$.
\item $(f_n(x))$ ist wachsend für alle $x\in X$, d.h.:
@ -92,8 +92,8 @@ Es folgt \(x\in B_n\) für jedes \(n\geq n(x)\).\\
\textbf{Fazit:} \(X=\bigcup B_n\). \[A_j=A_j\cap X=A_j\cap\left(\bigcup B_n\right) = \bigcup(A_j\cap B_n) \text{ und } A_j\cap B_n\subseteq A_j\cap B_{n+1} \]
Aus \ref{Satz 1.7} folgt \(\lambda(A_j)=\lim\limits_{n\to\infty}\lambda(A_j\cap B_n)\). Das liefert:
\begin{align*}
\int\limits_Xg\,dx &= \sum\limits_{j=1}^m y_j\lambda(A_j)
= \sum\limits_{j=1}^m y_j\lim\limits_{n\to\infty}\lambda(A_j\cap B_n)\\
\int\limits_Xg\,dx &= \sum\limits_{j=1}^m y_j\lambda(A_j)
= \sum\limits_{j=1}^m y_j\lim\limits_{n\to\infty}\lambda(A_j\cap B_n)\\
&=\lim\limits_{n\to\infty}\sum\limits_{j=1}^m y_j\lambda(A_j\cap B_n)
\overset{\ref{Satz 4.1}}= \lim\limits_{n\to\infty} \int\limits_X \mathds{1}_{B_n}g\,dx\\
&\leq \lim\limits_{n\to\infty} \int\limits_X \alpha f_n\,dx
@ -127,7 +127,7 @@ Dann ist wegen \ref{Satz 3.7} und \(\alpha , \beta \geq 0\), dass \((h_n)\) zul
\begin{enumerate}
\item["'$\implies$"'] Sei \(\int_Xf\,dx=0\) und \(A_n:=\{f>\frac{1}{n}\}\). Dann ist \(A=\bigcup A_n\) und \(f\geq\frac{1}{n}\mathds{1}_{A_n}\). Damit folgt:
\begin{align*}
0 = \int_Xf\,dx
0 = \int_Xf\,dx
\overset{\text{(2)}}\geq \int_X\frac1{n}\mathds{1}_{A_n}\,dx
=\frac1{n}\lambda(A_n)
\intertext{Es ist also \(\lambda(A_n)=0\) und damit gilt weiter}
@ -212,7 +212,7 @@ Sei $X \in \fb_1$, $f(x) := \begin{cases} 1&,x\in X\cap\MdQ\\ 0&,x\in X\setminus
$X, \MdQ \in \fb_1 \implies X \cap \MdQ \in \fb_1 \implies f$ ist messbar.
\[0 \leq \int_X f(x) \text{ d}x = \int_X \mathds{1}_{X\cap\MdQ} \text{ d}x = \lambda(X\cap\MdQ) \leq \lambda(\MdQ) = 0\]
\textbf{Das heißt:} $f \in \fl^1(X)$, $\int_X f \text{ d}x = 0$.
Ist speziell $X = [a,b]\quad (a<b)$, so gilt: $f \in \fl^1([a,b])$, aber $f \not\in R([a,b])$.
Ist speziell $X = [a,b]\quad (a<b)$, so gilt: $f \in \fl^1([a,b])$, aber $f \not\in R([a,b])$.
\end{beispiel}
\begin{satz}[Charakterisierung der Integrierbarkeit]
@ -258,9 +258,9 @@ Sei $f:X\to\imdr$ integrierbar und $N := \{\lvert f \rvert = +\infty\} = \{x\in
\end{folgerungen}
\begin{beweis}
$\ref{Satz 3.4} \implies N \in \fb(X).$ $n\mathds{1}_N \leq \lvert f \rvert$ für alle $n\in \MdN$. Dann:
$\ref{Satz 3.4} \implies N \in \fb(X).$ $n\mathds{1}_N \leq \lvert f \rvert$ für alle $n\in \MdN$. Dann:
\[n \cdot \lambda(N) = \int_X n\mathds{1}_N \text{ d}x \stackrel{4.5}{\leq} \int_X \lvert f \rvert \text{ d}x \stackrel{4.9}{<} \infty \text{ für alle } n \in \mdn\]
Also: $0 \leq n\lambda(N) \leq \int_X \lvert f \rvert \text{ d}x \quad \forall n \in \mdn \implies \lambda(N) = 0$
Also: $0 \leq n\lambda(N) \leq \int_X \lvert f \rvert \text{ d}x \quad \forall n \in \mdn \implies \lambda(N) = 0$
\end{beweis}
\begin{satz}
@ -277,13 +277,13 @@ $f, g: X \to \imdr$ seien integrierbar und es sei $\alpha \in \mdr$.
\item $\lvert \int_X f \text{ d}x \rvert \leq \int_X \lvert f \rvert \text{ d}x$. (Dreiecksungleichung für Integrale)
\item Sei $\emptyset\ne Y \in \fb(X)$. Dann sind die Funktionen $f_{|Y}: Y \to \imdr$ und $\mathds{1}_Y\cdot f: X \to \imdr$ integrierbar und
\[\int_Y f(x) \text{ d}x := \int_Y f_{|Y} (x) \text{ d}x = \int_X(\mathds{1}_Y \cdot f)(x) \text{ d}x\]
\item Sei $\lambda(X) < \infty$ und $h: X \to \mdr$ sei messbar und beschränkt. Dann: $h \in \fl^1(X)$ und $\lvert \int_X h \text{ d}x\rvert \leq \|h\|_\infty \lambda(X) \quad$ (mit $\|h\|_\infty := \sup\{|h(x)| : x\in X\}$)
\item Sei $\lambda(X) < \infty$ und $h: X \to \mdr$ sei messbar und beschränkt. Dann: $h \in \fl^1(X)$ und $\lvert \int_X h \text{ d}x\rvert \leq \|h\|_\infty \lambda(X) \quad$ (mit $\|h\|_\infty := \sup\{|h(x)| : x\in X\}$)
\end{enumerate}
\end{satz}
\begin{beweis}
\begin{enumerate}
\item folgt aus \(\alpha f)_{\pm}=\alpha f_{\pm}\), falls \(\alpha\geq0\) und \(\alpha f)_{\pm}=-\alpha f_{\mp}\), falls
\begin{enumerate}
\item folgt aus \(\alpha f)_{\pm}=\alpha f_{\pm}\), falls \(\alpha\geq0\) und \(\alpha f)_{\pm}=-\alpha f_{\mp}\), falls
\(\alpha<0\).
\item Es gilt \(f+g=\underbrace{f_{+}+g_{+}}_{=:u}-\underbrace{(f_{-}+g_{-})}_{=:v}=u-v\). Dann:
\[
@ -309,11 +309,11 @@ Es folgt:
\[
\int_{X}{f\mathrm{d}x}=\int_{X}{f_{+}\mathrm{d}x}-\int_{X}{f_{-}\mathrm{d}x}\overset{\ref{Satz 4.5}}{\leq}\int_{X}{g_{+}\mathrm{d}x}-\int_{X}{g_{-}\mathrm{d}x}=\int_{X}{g\mathrm{d}x}
\]
\item Es ist \(\pm f\leq\lvert f\rvert\). Mit Aussage (1) und (5) folgt:
\item Es ist \(\pm f\leq\lvert f\rvert\). Mit Aussage (1) und (5) folgt:
\(\pm\int_{X}{f\mathrm{d}x}=\int_{X}{(\pm f)\mathrm{d}x}\leq\int_{X}{\lvert f\rvert\mathrm{d}x}\).\\
Es ist \(\int_{X}{f\mathrm{d}x}=\lvert\int_{X}{f\mathrm{d}x}\rvert\) oder \(-\int_{X}{f\mathrm{d}x}=\lvert\int_{X}{f\mathrm{d}x}\rvert\)
\item Mit Bemerkung (2) vor \ref{Satz 3.1} und Satz \ref{Satz 3.6}.(2) folgt: \(f_{|Y}\) und \(\mathds{1}_{Y}\cdot f\) sind
messbar. Es gilt: \((f_{|Y})_{\pm}=(f_{\pm})_{|Y}\) und \((\mathds{1}_{Y}\cdot f)_{\pm}=\mathds{1}\cdot f_{\pm}\). Weiterhin
messbar. Es gilt: \((f_{|Y})_{\pm}=(f_{\pm})_{|Y}\) und \((\mathds{1}_{Y}\cdot f)_{\pm}=\mathds{1}\cdot f_{\pm}\). Weiterhin
gilt \(0\leq\mathds{1}_{Y}f_{\pm}\leq f_{\pm}\). Mit \ref{Satz 4.9} folgt dann, daß\ \(\mathds{1}_{Y}f_{\pm}\) integrierbar
ist. Dann:
\begin{align*}
@ -325,7 +325,7 @@ Es folgt: \(f_{|Y}\) ist integrierbar und \(\int_{Y}{f_{|Y}\mathrm{d}x}=\int_{Y}
\[
\int_{X}{\lvert h\rvert\mathrm{d}x}\leq\int_{X}{\lVert h\rVert_{\infty}\mathds{1}_{X}\mathrm{d}x}=\lVert h\rVert_{\infty}\lambda(X)<\infty
\]
Damit: \(\lvert h\rvert\) ist integrierbar und mit \ref{Satz 4.9} auch \(h\). Da \(h\) beschränkt ist, folgt:
Damit: \(\lvert h\rvert\) ist integrierbar und mit \ref{Satz 4.9} auch \(h\). Da \(h\) beschränkt ist, folgt:
\(h\in\fl^{1}(X)\). Schließlich:
\[
\left\lvert\int_{X}{h\mathrm{d}x}\right\rvert\leq\int_{X}{\lvert h\rvert\mathrm{d}x}\leq\lVert h\lVert_{\infty}\lambda(X)
@ -345,7 +345,7 @@ Damit: \(\lvert h\rvert\) ist integrierbar und mit \ref{Satz 4.9} auch \(h\). Da
\begin{beweis}
\begin{enumerate}
\item Aus \ref{Satz 4.11}(7) folgt: $f$ ist integrierbar über $A$ und integrierbar über $B$. Es ist
\item Aus \ref{Satz 4.11}(7) folgt: $f$ ist integrierbar über $A$ und integrierbar über $B$. Es ist
\[ \int_X f(x) \text{ d}x = \int_X \left( \mathds{1}_{A\cup B} \cdot f \right)(x) \text{ d}x = \int_X \left( \left( \mathds{1}_A + \mathds{1}_B \right) f\right)(x) \text{ d}x \]
\[= \int_X \left(\mathds{1}_A f + \mathds{1}_B f \right)(x) \text{ d}x \stackrel{4.11(2)}{=} \int_X \mathds{1}_A f \text{ d}x + \int_X \mathds{1}_B f \text{ d}x \stackrel{4.11(7)}{=} \int_A f \text{ d}x + \int_B f \text{ d}x.\]
@ -364,8 +364,8 @@ Sei $\natn$, $t_j^{(n)}:=a+j\frac{b-a}{n}$ ($j=0,\dots,n$) und $I_j^{(n)}:=\left
\begin{align*}
S_n:=\sum^n_{j=1} f \left(t_j^{(n)}\right) \underbrace{ \frac{b-a}{n}}_{= \lambda_1 \left(I_j^{(n)}\right)} \text{ ist Riemannsche Zwischensumme für R-} \int_a^bf(x)\,dx.
\end{align*}
Aus Analysis I folgt $S_n\to\text{R-}\int_a^bf(x)\,dx$ ($n\to\infty$).
Definiere $f_n:=\sum^n_{j=1}f \left(t_j^{(n)} \right) \mathds{1}_{I_j^{(n)}} $. Dann ist $f_n$ einfach und
Aus Analysis I folgt $S_n\to\text{R-}\int_a^bf(x)\,dx$ ($n\to\infty$).
Definiere $f_n:=\sum^n_{j=1}f \left(t_j^{(n)} \right) \mathds{1}_{I_j^{(n)}} $. Dann ist $f_n$ einfach und
\[\int_X f_n(x)\,dx=\sum_{j=1}^n f \left(t_j^{(n)} \right) \lambda_1 \left(I_j^{(n)}\right)=S_n\]
$f$ ist auf $X$ gleichmäßig stetig also konvergiert $f_n$ auf $X$ gleichmäßig gegen $f$ (Übung!), also gilt:
\[\lVert f_n-f \rVert_{\infty}=\text{sup} \left \{ \lvert f_n(x)-f(x) \rvert : x\in X \right\} \to 0 \ (n\to \infty)\]

View file

@ -56,7 +56,7 @@ Seien $f:X\to\imdr$ messbare Funktionen.
\item Ist $f$ integrierbar, so ist $f$ fast überall endlich.
\item Ist $f \ge0$ auf $X$, so ist $\int_X f(x)\text{ d}x=0$ genau dann wenn fast überall $f=0$.
\item Ist $f$ integrierbar und $N\subseteq X$ eine Nullmenge, so gilt:
\[\int_N f(x)\text{ d}x=0\]
\[\int_N f(x)\text{ d}x=0\]
\end{enumerate}
\end{satz}
@ -64,9 +64,9 @@ Seien $f:X\to\imdr$ messbare Funktionen.
\begin{enumerate}
\item ist gerade \ref{Folgerung 4.10}.
\item ist gerade \ref{Satz 4.5}(3)
\item Setze $g:=\mathds{1}_N f$. Aus \ref{Satz 4.11} folgt, dass g integrierbar ist, also ist nach \ref{Satz 4.9} auch $\lvert g \rvert$ integrierbar. Für $x\in X\setminus N$ gilt:
\item Setze $g:=\mathds{1}_N f$. Aus \ref{Satz 4.11} folgt, dass g integrierbar ist, also ist nach \ref{Satz 4.9} auch $\lvert g \rvert$ integrierbar. Für $x\in X\setminus N$ gilt:
\[g(x)=\lvert g(x) \rvert =0\]
D.h. $\lvert g \rvert =0$ fast überall. Aus (2) folgt damit $\int_X \lvert g \rvert \,dx = 0$. Dann ist mit \ref{Satz 4.11}: \[\left\lvert\int_X g\,dx \right\rvert \leq \int_X \lvert g \rvert \,dx =0\]
D.h. $\lvert g \rvert =0$ fast überall. Aus (2) folgt damit $\int_X \lvert g \rvert \,dx = 0$. Dann ist mit \ref{Satz 4.11}: \[\left\lvert\int_X g\,dx \right\rvert \leq \int_X \lvert g \rvert \,dx =0\]
und somit $\int_X g\,dx=0$.
\end{enumerate}
\end{beweis}
@ -78,7 +78,7 @@ $f,g:X\to\imdr$ seien messbar.
\item Ist $f$ integrierbar und gilt fast überall $f=g$, so ist $g$ integrierbar und es gilt:
\[\int_Xf\,dx=\int_Xg\,dx\]
\item Ist $f$ integrierbar und $g:=\mathds{1}_{\{ \lvert f \rvert <\infty \}}\cdot f$, so ist $g$ integrierbar und es gilt: \[\int_Xf\,dx=\int_Xg\,dx\]
\item Sind $f$ und $g$ beide $\geq0$ auf $X$, und ist fast überall $f=g$, so ist
\item Sind $f$ und $g$ beide $\geq0$ auf $X$, und ist fast überall $f=g$, so ist
\[\int_Xf\,dx=\int_Xg\,dx\]
\end{enumerate}
\end{satz}
@ -86,14 +86,14 @@ $f,g:X\to\imdr$ seien messbar.
\begin{beweis}
\begin{enumerate}
\item Nach Voraussetzung existiert eine Nullmenge $N\subseteq X$, sodass gilt:
\[\forall x\in X\setminus N:f(x)=g(x)\]
\[\forall x\in X\setminus N:f(x)=g(x)\]
Aus \ref{Satz 5.2}(3) folgt dann $\int_N f\,dx=0$.
Sei $x\in X\setminus N$ Dann gilt:
\[\left( \mathds{1}_N \lvert g \rvert \right)(x)=\mathds{1}_N(x)\cdot \lvert g(x) \rvert=0\]
Sei $x\in X\setminus N$ Dann gilt:
\[\left( \mathds{1}_N \lvert g \rvert \right)(x)=\mathds{1}_N(x)\cdot \lvert g(x) \rvert=0\]
D.h.: Fast überall ist $\mathds{1}_N \lvert g \rvert =0$. Aus \ref{Satz 5.2}(2) folgt $\int_N \lvert g \rvert\,dx=\int_X\mathds{1}_N\cdot \lvert g \rvert\,dx=0$.
Dann gilt:
\begin{align*}
\int_X \lvert g\rvert\,dx & = \int_X \left(\mathds{1}_N \lvert g\rvert + \mathds{1}_{X\setminus N} \lvert g\rvert \right)\,dx\\
\int_X \lvert g\rvert\,dx & = \int_X \left(\mathds{1}_N \lvert g\rvert + \mathds{1}_{X\setminus N} \lvert g\rvert \right)\,dx\\
&= \int_X\mathds{1}_N \lvert g\rvert\,dx + \int _X\mathds{1}_{X\setminus N} \lvert g\rvert\,dx\\
&= \int_X \mathds{1}_{X\setminus N} \lvert g \rvert\,dx\\
& \leq\int_X \lvert f\rvert\,dx \overset{\ref{Satz 4.9}}< \infty
@ -141,15 +141,15 @@ Ist \(g\) wie in (2), so muss \(g\) nicht messbar sein (ein Beispiel gibt es in
\begin{beweis}
\begin{enumerate}
\item Es existiert eine Nullmenge \(N_{1}\subseteq X:\,(f_{n}(x))\) konvergiert in \(\imdr\) für alle
\item Es existiert eine Nullmenge \(N_{1}\subseteq X:\,(f_{n}(x))\) konvergiert in \(\imdr\) für alle
\(x\in X\setminus N_{1}\).
\[
f(x)=\begin{cases}0&x\in N_{1}\\\lim_{n\to\infty}{f_{n}(x)}&x\in X\setminus N_{1}\end{cases}
\]
\(g_{n}:=\mathds{1}_{X\setminus N}\cdot f_{n}\), \(g_{n}\) ist messbar und \(g_{n}(x)\to f(x)\) für alle \(x\in X\).
Mit \ref{Satz 3.5} folgt: \(f\) ist messbar.
\item Es existiert eine Nullmenge \(N_{2}\subseteq X:\,f_{n}(x)\to g(x)\,\forall x\in X\setminus N_{2}\).
\(N=N_{1}\cup N_{2}\). Aus \ref{Lemma 5.1} folgt: \(N\) ist eine Nullmenge.
\item Es existiert eine Nullmenge \(N_{2}\subseteq X:\,f_{n}(x)\to g(x)\,\forall x\in X\setminus N_{2}\).
\(N=N_{1}\cup N_{2}\). Aus \ref{Lemma 5.1} folgt: \(N\) ist eine Nullmenge.
Für \(x\in X\setminus N:\,f(x)=g(x)\).
\end{enumerate}
@ -159,19 +159,19 @@ Für \(x\in X\setminus N:\,f(x)=g(x)\).
\label{Satz 5.5}
Sei \((f_{n})\) eine Folge messbarer Funktionen \(f_{n}:\,X\to[0,+\infty]\) und für jedes \(n\in\mdn\) gelte:
\(f_{n}\leq f_{n+1}\) fast überall. Dann existiert eine messbare Funktion
\(f:X\to[0,+\infty]\) mit: \(f_{n}\to f\) fast überall und
\(f:X\to[0,+\infty]\) mit: \(f_{n}\to f\) fast überall und
\[\int_{X}{f\mathrm{d}x}=\lim_{n\to\infty}{\int_{X}{f_{n}\mathrm{d}x}}\]
\end{satz}
\begin{beweis}
Zu jedem \(n\in\mdn\) existiert eine Nullmenge
\(N_{n}:\,f_{n}(x)\leq f_{n+1}(x)\;\forall x\in X\setminus N_{n}\).\\
Zu jedem \(n\in\mdn\) existiert eine Nullmenge
\(N_{n}:\,f_{n}(x)\leq f_{n+1}(x)\;\forall x\in X\setminus N_{n}\).\\
\(N:=\bigcup_{n=1}^{\infty}{N_{n}}\) \folgtnach{\ref{Lemma 5.1}} \(N\) ist eine
Nullmenge.
Dann: \(f_{n}(x)\leq f_{n+1}(x)\forall x\in X\setminus N\forall n\in\mdn\).
\(\hat{f}_{n}:=\mathds{1}_{X\setminus N}\cdot f_{n}\), \(\hat{f}_{n}\) ist
\(\hat{f}_{n}:=\mathds{1}_{X\setminus N}\cdot f_{n}\), \(\hat{f}_{n}\) ist
messbar, \(\forall n\in\mdn: \hat{f}_{n}\leq\hat{f}_{n+1}\) auf $X$.
\(f(x):=\lim_{n\to\infty}{\hat{f}_{n}(x)}\,(x\in X)\) \folgtnach{\ref{Satz 3.5}}

View file

@ -41,7 +41,7 @@ Dann gilt \(f=\mathds{1}_{X\setminus N}\cdot f\) fast überall.
&\overset{(1)}{\leq}\liminf_{n\to\infty}\int_{X}{\mathds{1}_{X\setminus N}f_{n}\mathrm{d}x}\\
&\overset{\text{\ref{Satz 5.3}.(3)}}{=}\liminf_{n\to\infty}\int_{X}{f_{n}\mathrm{d}x}
\end{align*}
\item folgt aus (2). Nach Voraussetzung gilt
\item folgt aus (2). Nach Voraussetzung gilt
\[
0\leq\int_{X}{f\mathrm{d}x}\overset{\text{(2)}}{\leq}\liminf_{n\to\infty}\int_{X}{f_{n}\mathrm{d}x}<\infty
\]
@ -67,8 +67,8 @@ Dann sind alle \(f_{n}\) integrierbar und es existiert ein \(f\in\fl^{1}(X)\) mi
\[
\int_{X}{f_{n}\mathrm{d}x}=n\cdot\lambda_{1}\left(\left(0,\frac{1}{n}\right)\right)=n\cdot\frac{1}{n}=1\quad\forall n\in\mdn
\]
Es gilt \(f_{n}\to f:=0\) punktweise und \(\int_{X}{f\mathrm{d}x}=0 \neq 1 = \int_{X}{f_{n}\mathrm{d}x}\).
$\Rightarrow$ \ref{Satz 6.2} ist ohne die integrierbare Majorante
Es gilt \(f_{n}\to f:=0\) punktweise und \(\int_{X}{f\mathrm{d}x}=0 \neq 1 = \int_{X}{f_{n}\mathrm{d}x}\).
$\Rightarrow$ \ref{Satz 6.2} ist ohne die integrierbare Majorante
$g$ im allgemeinen falsch.
\item Sei $X = [1, \infty), \alpha > 1, f_n(x) := \frac{1}{x^\alpha} \sin{\frac{x}{n}} (x \in X, n \in \mathbb{N})$.\\
Berechne $\lim_{n \rightarrow \infty} \int_X f_n(x) \mathrm{d}x$\\
@ -132,9 +132,9 @@ Also gilt auch:
\begin{beispiel}
Sei \(X:=[1,\infty)\) und \(f_n(x):=\frac1{x^\frac32}\sin\left(\frac xn \right) \) für alle \(x\in X, n\in\mdn\) mit \(f_n(x)\to f(x)\equiv 0\) für jedes \(x\in X\).
Dann ist \(\lvert f_n(x) \rvert\leq \frac1{x^\frac32}\) für jedes \(x\in X\) und $\natn$.
Dann ist \(\lvert f_n(x) \rvert\leq \frac1{x^\frac32}\) für jedes \(x\in X\) und $\natn$.
Definiere nun \[g(x):=\frac1{x^\frac32}\]
Aus Analysis I ist bekannt, dass \(\int^\infty_1 g(x)\,dx\) (absolut) konvergent ist
Aus Analysis I ist bekannt, dass \(\int^\infty_1 g(x)\,dx\) (absolut) konvergent ist
und aus \ref{Satz 4.14} folgt \[g\in\mathfrak{L}^1(X) \text{ sowie } \int_X g(x)\,dx = \text{R-}\int^\infty_1 g(x)\,dx\]
Weiter folgen aus \ref{Satz 6.2}:
\[\int_X f_n\,dx\to 0 \text{ und } \int_X\lvert f_n\rvert\,dx\to 0 \ (n\to\infty) \]
@ -155,17 +155,17 @@ Weiter folgen aus \ref{Satz 6.2}:
\begin{beweis}
\begin{enumerate}
\item Sei \(x\in X\). Es exisitert ein $m\in\mdn$, für das \(x\in A_m\) ist und somit auch \(x\in A_n \) für jedes \(n\geq m\). Nach der Definition von $f_n$ gilt dann \(f_n(x)=f(x)\) für jedes \(n\geq m\) und somit \(f_n\to f\) auf $X$. Damit gilt auch \[\lvert f_n\rvert\to\lvert f\rvert \text{ auf } X\] Durch die Konstruktion der $f_n$ ergibt sich:
\item Sei \(x\in X\). Es exisitert ein $m\in\mdn$, für das \(x\in A_m\) ist und somit auch \(x\in A_n \) für jedes \(n\geq m\). Nach der Definition von $f_n$ gilt dann \(f_n(x)=f(x)\) für jedes \(n\geq m\) und somit \(f_n\to f\) auf $X$. Damit gilt auch \[\lvert f_n\rvert\to\lvert f\rvert \text{ auf } X\] Durch die Konstruktion der $f_n$ ergibt sich:
\[ \lvert f_n\rvert=\lvert \mathds{1}_{A_n}f\rvert=\mathds{1}_{A_n}\lvert f\rvert \leq \mathds{1}_{A_{n+1}}\lvert f\rvert=\lvert f_{n+1}\rvert \]
Dann gilt:
\[ \int_X \lvert f\rvert\,dx \overset{\ref{Satz 4.6}}=\lim\int_X \lvert f_n\rvert\,dx = \lim\int_{A_n} \lvert f\rvert\,dx \overset{Vor.}<\infty \]
Es folgt, dass \(\lvert f\rvert\) integrierbar ist und somit ist nach \ref{Satz 4.9} auch $f$ integrierbar. Da \(\lvert f_n\rvert \leq \lvert f\rvert\) auf $X$ für jedes \(\natn\) gilt, ist $f$ eine
Es folgt, dass \(\lvert f\rvert\) integrierbar ist und somit ist nach \ref{Satz 4.9} auch $f$ integrierbar. Da \(\lvert f_n\rvert \leq \lvert f\rvert\) auf $X$ für jedes \(\natn\) gilt, ist $f$ eine
integrierbare Majorante und es folgt mit \ref{Satz 6.2}:
\[ \int_Xf\,dx = \lim\int_Xf_n\,dx = \lim\int_{A_n}f\,dx \]
\item Setze \(A_n:=[a,n]\ (\natn)\) und es gelte o.B.d.A.: \(a\leq 1\). Dann gilt:
\[ \int_{A_n}\lvert f\rvert\,dx \overset{\ref{Satz 4.13}}= \text{R-}\int^n_a \lvert f\rvert\,dx \overset{Vor.}\longrightarrow \text{R-}\int^\infty_a \lvert f\rvert\,dx \]
D.h.\(\left(\int_{A_n}\lvert f\rvert\,dx\right)\) ist beschränkt. Definiere \(f_n:=\mathds{1}_{A_n}f\) mit \ref{Satz 4.13} folgt daraus, dass $f_n$ integrierbar ist. Weiter folgt
aus (1) \(f\in\mathfrak{L}^1(X)\) (denn es ist \(f(X)\subseteq\mdr\)) und
aus (1) \(f\in\mathfrak{L}^1(X)\) (denn es ist \(f(X)\subseteq\mdr\)) und
\[ \text{L-}\int_Xf\,dx = \lim\int_{A_n}f\,dx \overset{\ref{Satz 4.13}}= \lim\left(\text{R-}\int^n_a f\,dx \right) = \text{R-}\int^\infty_a f\,dx. \]
\end{enumerate}
\end{beweis}
@ -177,12 +177,12 @@ Weiter folgen aus \ref{Satz 6.2}:
\begin{folgerung}
\label{Folgerung 6.4}
\begin{enumerate}
\item \((f_n)\) sei eine Folge integrierbarer Funktionen \(f_n\colon X\to\imdr\), \(g\colon X\to[0,+\infty]\) sei ebenfalls integrierbar und
\item \((f_n)\) sei eine Folge integrierbarer Funktionen \(f_n\colon X\to\imdr\), \(g\colon X\to[0,+\infty]\) sei ebenfalls integrierbar und
\[g_n:=f_1+f_2+\dots+f_n \ (\natn)\]
Weiter sei $N$ eine Nullmenge in $X$ so, dass \((g_n(x))\) für jedes \(x\in X\setminus N\) in $\imdr$ konvergiert und
Weiter sei $N$ eine Nullmenge in $X$ so, dass \((g_n(x))\) für jedes \(x\in X\setminus N\) in $\imdr$ konvergiert und
\[\lvert g_n(x)\rvert \leq g(x) \text{ für jedes } \natn \text{ und } x\in X\setminus N\]
Setzt man
\[f(x):=\sum^\infty_{j=1}f_j(x):=
\[f(x):=\sum^\infty_{j=1}f_j(x):=
\begin{cases}
0, & \text{falls } x\in N \\
\lim\limits_{n\to\infty}g_n(x), & \text{falls } x\in X\setminus N
@ -198,7 +198,7 @@ Weiter folgen aus \ref{Satz 6.2}:
\begin{enumerate}
\item Fast überall gelten \(g_n\to f\) und für jedes \(\natn\) auch \(\lvert g_n\rvert \leq g\). Aus \ref{Satz 6.2} folgt
\begin{align*}
\int_X \left(\sum^\infty_{j=1}f_j(x)\right) \,dx
\int_X \left(\sum^\infty_{j=1}f_j(x)\right) \,dx
&= \int_Xf\,dx \\
&\overset{\ref{Satz 6.2}}= \lim\int_Xg_n\,dx \\
&= \lim\int_X\left(\sum^n_{j=1}f_j\right)\,dx \\

View file

@ -6,7 +6,7 @@ Sei \(U\in\fb_k, t_0\in U\) und es sei \(f\colon U\times X\to \mdr\) eine Funkti
\begin{enumerate}
\item Für jedes \(t\in U\) ist \(x\mapsto f(t,x)\) messbar.
\item Es existiert eine Nullmenge \(N\subseteq X\) so, dass \(t\mapsto f(t,x)\) für jedes \(x\in X\setminus N\) stetig in $t_0$ ist.
\item Es existiert eine integrierbare Funktion \(g\colon X\to [0,\infty]\) und zu jedem \(t\in U\) existiert eine Nullmenge \(N_t\subseteq X\) so, dass für
\item Es existiert eine integrierbare Funktion \(g\colon X\to [0,\infty]\) und zu jedem \(t\in U\) existiert eine Nullmenge \(N_t\subseteq X\) so, dass für
jedes \(t\in U\) und jedes \(x\in X\setminus N_t\) gilt: \[ \lvert f(t,x)\rvert \leq g(x) \]
\end{enumerate}
Dann ist \(x\mapsto f(t,x)\) für jedes \(t\in U\) integrierbar. Ist \(F\colon U\to\mdr\) definiert durch
@ -17,31 +17,31 @@ so ist $F$ stetig in $t_0$.
Also: \[ \lim\limits_{t\to t_0}\int_X f(t,x)\,dx = \lim\limits_{t\to t_0}F(t)=F(t_0) = \int_X f(t_0,x)\,dx =\int_X\lim\limits_{t\to t_0} f(t,x)\,dx \]
\begin{beweis}
Aus (1) und (3) folgt, dass \(x\mapsto f(t,x)\) für jedes \(t\in U\) integrierbar ist (zur Übung). Sei \((t_n)\) eine Folge in $U$ mit \(t_n\to t_0\) und
Aus (1) und (3) folgt, dass \(x\mapsto f(t,x)\) für jedes \(t\in U\) integrierbar ist (zur Übung). Sei \((t_n)\) eine Folge in $U$ mit \(t_n\to t_0\) und
\[g_n(x):=f(t_n,x) \ (\natn, x\in X) \]
Setze \[ \tilde N := N\cup \left(\bigcup^\infty_{n=1}N_{t_n} \right) \]
Aus \ref{Lemma 5.1} folgt, dass \(\tilde N\) eine Nullmenge ist. Voraussetzung (2) liefert \(g_n(x)\to f(t_0,x)\) für jedes \(x\in X\setminus\tilde N\), also gilt
Aus \ref{Lemma 5.1} folgt, dass \(\tilde N\) eine Nullmenge ist. Voraussetzung (2) liefert \(g_n(x)\to f(t_0,x)\) für jedes \(x\in X\setminus\tilde N\), also gilt
\[g_n(x)\to f(t_0,x) \text{ fast überall auf } X\]
Voraussetzung (3) liefert \(\lvert g_n(x)\rvert = \lvert f(t_n,x)\rvert \leq g(x) \) für jedes \(\natn\) und \(x\in X\setminus\tilde N\). Aus \ref{Satz 6.2} folgt
\[ F(t_n) = \int_X f(t_n,x)\,dx = \int_Xg_n\,dx \longrightarrow \int_X f(t_0,x)\,dx = F(t_0) \]
\end{beweis}
\textbf{Bezeichnung}\\
Sei \(I\subseteq\mdr\) ein Intervall, \(a:=\inf I\) und \(b:=\sup I\), wobei \(a=-\infty\) oder \(b=+\infty\) zugelassen sind. Weiter sei \(f\colon I\to\imdr\) integrierbar
(oder $f$ ist messbar und \(\geq 0\)) und
Sei \(I\subseteq\mdr\) ein Intervall, \(a:=\inf I\) und \(b:=\sup I\), wobei \(a=-\infty\) oder \(b=+\infty\) zugelassen sind. Weiter sei \(f\colon I\to\imdr\) integrierbar
(oder $f$ ist messbar und \(\geq 0\)) und
\[\int\limits^b_af(x)\,dx:=\int\limits_{(a,b)}f_{|(a,b)}(x)\,dx \]
Dann ist
Dann ist
\[ \int_I f(x) dx = \int_{(a,b)} f(x) dx\]
Ist z.B. \(I=[a,b)\), dann gilt, da \(\{a\}\) eine Nullmenge ist: \[\int_If\,dx=\int_{\{a\}}f\,dx + \int_{(a,b)}f\,dx= \int_{(a,b)}f\,dx \]
Ist z.B. \(I=[a,b)\), dann gilt, da \(\{a\}\) eine Nullmenge ist: \[\int_If\,dx=\int_{\{a\}}f\,dx + \int_{(a,b)}f\,dx= \int_{(a,b)}f\,dx \]
\begin{folgerung}
\label{Folgerung 7.2}
Sei \(I\subseteq\mdr\) ein Intervall, \(a=\inf I\) und \(f\colon I\to\mdr\) sei integrierbar. Definiert man \(F\colon I\to\mdr\) durch
Sei \(I\subseteq\mdr\) ein Intervall, \(a=\inf I\) und \(f\colon I\to\mdr\) sei integrierbar. Definiert man \(F\colon I\to\mdr\) durch
\[F(t):=\int^t_a f(x)\,dx,\] so ist \(F\in C(I)\).
\end{folgerung}
\begin{beweis}
Für \(x,t\in I\) definiere \(h(t,x):=\mathds{1}_{(a,t)}f(x)\). Dann ist \(F(t)=\int_I h(t,x)\,dx\) und
Für \(x,t\in I\) definiere \(h(t,x):=\mathds{1}_{(a,t)}f(x)\). Dann ist \(F(t)=\int_I h(t,x)\,dx\) und
\[\lvert h(t,x)\rvert = \mathds{1}_{(a,t)}\cdot \lvert f(x)\rvert \leq \lvert f(x)\rvert \text{ für alle } t,x\in I\]
Aus \ref{Satz 4.9} folgt, dass \(\lvert f\rvert\) integrierbar ist. Sei \(t_0\in I\) und \(N:=\{t_0\}\), also eine Nullmenge.
Dann ist \(t\mapsto h(t,x)\) für jedes \(x\in I\setminus N\) stetig in \(t_0\) (zur Übung). Die Behauptung folgt aus \ref{Satz 7.1}.

View file

@ -44,7 +44,7 @@ C_y= \begin{cases}
{\emptyset, \text{falls } x\notin A}\\
{B, \text{falls } x\in A}
\end{cases}
\end{align*}
\end{align*}
\begin{lemma}
\label{Lemma 8.3}
@ -74,11 +74,11 @@ folgt aus \ref{Lemma 8.1} und \ref{Lemma 8.3}.
\end{beweis}
%vielleicht funktioniert die nummerierung jetzt
\begin{defusatz}[ohne Beweis]
\begin{defusatz}[ohne Beweis]
\label{Satz 8.5}
Sei \(C\in\fb_d\). Die Funktionen \(\varphi_C\) und \(\psi_C\) seien unter Beachtung von \ref{Lemma 8.2} definiert durch:
\begin{align*}
\varphi_C(x):=\lambda_l(C^x) \ \ (x\in\mdr^k) & & \psi_C(x):=\lambda_k(C_y) \ \ (y\in\mdr^l)
\varphi_C(x):=\lambda_l(C^x) \ \ (x\in\mdr^k) & & \psi_C(x):=\lambda_k(C_y) \ \ (y\in\mdr^l)
\end{align*}
Dann sind \(\varphi_C\) und \(\psi_C\) messbar.
\end{defusatz}

View file

@ -26,7 +26,7 @@ Das heißt:
&\overset{Ana I}= \pi r^2
\end{align*}
\item Sei \(\emptyset\neq X\subseteq\mdr^d\). $X$ sei kompakt, also \(X\in\fb_d\). Weiter sei \(f\colon X\to[0,\infty)\) stetig, woraus mit \ref{Satz 4.11} \(f\in\mathfrak{L}^1(X)\) folgt.
Setze \[C:=\{(x,y):x\in X, 0\leq y\leq f(x)\}\]
Setze \[C:=\{(x,y):x\in X, 0\leq y\leq f(x)\}\]
$C$ ist kompakt und somit gilt: \(C\in\fb_{d+1}\).\\
Ist \(x\notin X\), so ist \(C^x=\emptyset\), also \(\lambda_1(C^x)=0\).\\
Ist \(x\in X\), so ist \(C^x=[0,f(x)]\), also \(\lambda_1(C^x)=f(x)\). Damit gilt
@ -35,7 +35,7 @@ Das heißt:
\[C:=\{(x,y)\in\mdr^2:x\in I, 0\leq y\leq f(x)\}\]
Aus Beispiel (2) und \ref{Satz 4.13} folgt \[\lambda_2(C)=\text{R-}\int_a^bf(x)\,dx \]
\item $X$ und $f$ seien wie in Beispiel (2). Setze \[G:=\{(x,f(x)):x\in X\}\]
$G$ ist kompakt, also ist \(G\in\fb_2\).
$G$ ist kompakt, also ist \(G\in\fb_2\).
Ist \(x\notin X\), so ist \(G^x=\emptyset\), also \(\lambda_1(G^x)=0\).
Ist \(x\in X\), so ist \(G^x=\{f(x)\}\), also \(\lambda_1(G^x)=0\).
Aus \ref{Satz 9.1} folgt \[\lambda_2(G)=\int_\mdr\lambda_1(G^x)\,dx=0\]
@ -58,7 +58,7 @@ Sei $(A_j)$ eine disjunkte Folge in $\fb_d$. Dann ist $(A_j^x)$ ebenfalls disjun
D.h. $\mu$ ist ein Maß auf $\fb_d$. Analog lässt sich zeigen, dass $\nu$ ein Maß auf $\fb_d$ ist.\\
Sei nun $I\in\ci_d$, dann existieren $I'\in\ci_k, I''\in\ci_l$ mit $I=I'\times I''$. Aus §\ref{Kapitel 8} folgt:
\begin{align*}
I^x=\begin{cases} I''&,x\in I'\\
I^x=\begin{cases} I''&,x\in I'\\
\emptyset &,x\not\in I'\end{cases}
\end{align*}
Also ist $\lambda_l(I^x)=\lambda_l(I'')\cdot\mathds{1}_{I'}(x)$ und damit:
@ -111,7 +111,7 @@ Also folgt aus \ref{Satz 3.4} die Messbarkeit von $\tilde f$.
\begin{beispiel}
\index{Rotationskörper}
\begin{enumerate}
\item Sei $r>0$ und
\item Sei $r>0$ und
\[K:=\{(x,y)\in\mdr^2\mid x^2+y^2<r^2\}\]
Dann ist $K$ offen, also $K\in\fb_2$ und es gilt:
\[\partial K=\overline{K}\setminus K=\{(x,y)\in\mdr^2\mid x^2+y^2=r^2\}\in\fb_2\]
@ -120,7 +120,7 @@ Damit enthält die Menge $(\partial K)_y$ für alle $x\in\mdr$ höchstens zwei E
Mit $\overline K=(\partial K) \dot\cup K$ folgt dann
\[\lambda_2(K)=\lambda_2(\partial K)+\lambda_2(\overline K)=\lambda_2(\overline K)=\pi r^2\]
Sei nun $A\in\fb_2$ mit $K\subseteq A\subseteq\overline K$, dann ist $\lambda_2(A)=\pi r^2$.
\item Sei $r>0$ und
\item Sei $r>0$ und
\[K:=\{(x,y,z)\in\mdr^3\mid x^2+y^2+z^2\le r^2\}\]
Dann ist $K$ abgeschlossen, also $K\in\fb_3$.\\
\textbf{Fall $|z|>r$:} Es ist $K_z=\emptyset$, also $\lambda_2(K_z)=0$.\\
@ -146,7 +146,7 @@ und damit $\lambda_2(V_z)=\pi f(z)^2$.\\
Aus \ref{Satz 9.1} folgt dann:
\begin{align*}
\lambda_3(V)&=\int_\mdr \lambda_2(V_z)\text{ d}z\\
&= \pi\int_a^b f(z)^2\text{ d}z
&= \pi\int_a^b f(z)^2\text{ d}z
\end{align*}
\item Sei $h>0$, $I=[0,h]$ und $f(z)=\frac rhz$. Definiere den Kegel
\[V:=\{(x,y,z)\in\mdr^3\mid x^2+y^2\le \frac{r^2}{h^2}z^2\}\]

View file

@ -15,7 +15,7 @@
\fancyhead{}
\renewcommand{\headrulewidth}{0pt}
}
\setlength{\headheight}{15pt} % fixes \headheight warning
\lhead{\Vorname{}~\Nachname{}, \Strasse{}, \PLZ{}~\Ort}
\rhead{Id-Nr. \Idnr}

View file

@ -5,7 +5,7 @@
\usepackage[ngerman, num]{isodate} % get DD.MM.YYYY dates
\usepackage{pdfpages} % Signatureinbingung und includepdf
\usepackage{myInformation}
% pdfinfo
\pdfinfo{
/Author (\Nachname, \Vorname)
@ -18,7 +18,7 @@
\signature{\Vorname~\Nachname}
\setkomavar{customer}[Steuernummer (Id-Nr.)]{\Idnr}
\backaddress{\Vorname~\Nachname, \Strasse~\Hausnummer, \PLZ~\Ort}
% Begin document %%%%%%%%%%%%%%%%%%%%%%%%%%%%%%%%%%%%%%%%%%%%%%%%%%%%
\begin{document}
\begin{letter}{\Empfaenger \\ \EStrasse \\ \EPLZ~\EOrt}
@ -26,7 +26,7 @@
\subject{Einnahmenüberschussrechnung \Year}
\opening{Sehr geehrte Damen und Herren,}
Im Anhang befindet sich die Überschussrechnung von \Year.
Im Anhang befindet sich die Überschussrechnung von \Year.
\closing{Mit freundlichen Grüßen,}
\end{letter}

View file

@ -1,5 +1,5 @@
* Einmalig müssen Parameter in der `myInformation.tex` bearbeitet werden.
* Dann nur noch jedes Jahr die vier `.csv`-Dateien und gegebenenfalls
* Dann nur noch jedes Jahr die vier `.csv`-Dateien und gegebenenfalls
unter `Einnahmenueberschussrechnung.tex` Anmerkungen machen
Tags: Steuer, Steuererklärung, LaTeX, Finanzen, EÜR

View file

@ -19,10 +19,10 @@
\newcommand\yourTutorial{10}
%%%%%%%%%%%%%%%%%%%%%%%%%%%%%%%%%%%%%%%%%%%%%%%%%%%%%%%%%%%%%%%%%%%%%
\hypersetup {
pdfauthor = {Martin Thoma},
pdfkeywords = {Feedback},
pdftitle = {Feedback}
\hypersetup {
pdfauthor = {Martin Thoma},
pdfkeywords = {Feedback},
pdftitle = {Feedback}
}
\pagestyle{fancy}% eigenen Seitestil aktivieren}
@ -74,7 +74,7 @@ Diese Themen sollten wiederholt werden (z.B. Arrays, for/while Schleifen, Ausdr
\hfill\vspace{3cm}
\end{framed}
\noindent \textbf{Andere Kommentare}: z.B. Was ist unklar?
\noindent \textbf{Andere Kommentare}: z.B. Was ist unklar?
Was war heute gut / schlecht?
\begin{framed}
\hfill\vspace{3cm}

View file

@ -18,7 +18,7 @@ Folgende Definition wurde dem Skript von Herrn Prof.~Dr.~Leuzinger für
Lineare Algebra entnommen:
\begin{definition}\xindex{Abbildung!affine}%
Es seien $V$ und $W$ $\mdk$-Vektorräume und $\mda(V)$ und $\mda(W)$ die
Es seien $V$ und $W$ $\mdk$-Vektorräume und $\mda(V)$ und $\mda(W)$ die
zugehörigen affinen Räume. Eine Abbildung $f:V \rightarrow W$ heißt \textbf{affin},
falls für alle $a, b \in V$ und alle $\lambda, \mu \in \mdk$ mit $\lambda + \mu = 1$ gilt:
\[f(\lambda a + \mu b) = \lambda f(a) + \mu f(b)\]
@ -36,13 +36,13 @@ Lineare Algebra entnommen:
\end{definition}
\begin{satz*}[Zwischenwertsatz]\xindex{Zwischenwertsatz}%
Sei $a<b$ und $f \in\ C[a, b]:=C([a, b])$, weiter sei $y_0 \in \mdr$ und
$f(a) < y_0 < f(b)$ oder $f(b) < y_0 < f(a)$. Dann existiert ein
Sei $a<b$ und $f \in\ C[a, b]:=C([a, b])$, weiter sei $y_0 \in \mdr$ und
$f(a) < y_0 < f(b)$ oder $f(b) < y_0 < f(a)$. Dann existiert ein
$x_0 \in [a, b]$ mit $f(x_0) = y_0$.
\end{satz*}
\begin{definition}\xindex{Eigenwert}\xindex{Eigenvektor}%
Sei $V$ ein Vektorraum über einem Körper $\mdk$ und $f: V \rightarrow V$ eine
Sei $V$ ein Vektorraum über einem Körper $\mdk$ und $f: V \rightarrow V$ eine
lineare Abbildung.
$v \in V \setminus \Set{0}$ heißt \textbf{Eigenvektor} $:\Leftrightarrow \exists \lambda \in \mdk: f(v) = \lambda v$.

View file

@ -39,10 +39,10 @@
\usepackage[left=10mm,right=10mm, top=2mm, bottom=10mm]{geometry}
\usepackage{../shortcuts}
\hypersetup{
pdfauthor = {Martin Thoma},
pdfkeywords = {Geometrie und Topologie},
pdftitle = {Fragen zu Definitionen}
\hypersetup{
pdfauthor = {Martin Thoma},
pdfkeywords = {Geometrie und Topologie},
pdftitle = {Fragen zu Definitionen}
}
\allowdisplaybreaks
@ -73,7 +73,7 @@ $f:|K| \rightarrow |L|$
mit $f(\Delta) \notin L$?}
\section*{18.) ÜB 1, Aufgabe 2}
\underline{Vor.:} Es sei $(X, d)$ ein metrischer Raum, $A \subseteq X$.
\underline{Vor.:} Es sei $(X, d)$ ein metrischer Raum, $A \subseteq X$.
Weiter bezeichne $\fT$ die von $d$ auf $X$ erzeugte Topologie $\fT'$, die von
der auf $A \times A$ eingeschränkten Metrik $d|_{A \times A}$ erzeugte Topologie.
@ -87,7 +87,7 @@ Sei $U \in \fT|_A = \Set{V \cap A | V \in \fT}$.\\
Dann ex. also $V \in \fT$ mit
$U = V \cap A$.\\
Sei $x \in U$.\\
Da $V \in \fT$, ex. nach Bemerkung~3 ein $r > 0$ mit
Da $V \in \fT$, ex. nach Bemerkung~3 ein $r > 0$ mit
\begin{align*}
\fB_r(x) := \Set{y \in X | d(x,y) < r} &\subseteq V\\
@ -123,7 +123,7 @@ Da $x \in U$ beliebig gewählt war gilt: $\fT|_A \subseteq \fT'$
\[m_g: X \rightarrow X, x \mapsto g \circ x\]
ein Homöomorphismus ist.
\item Ist $G$ eine topologische Gruppe, so heißt die Gruppenoperation $\circ$
\textbf{stetig}\xindex{Gruppenoperation!stetige}, wenn
\textbf{stetig}\xindex{Gruppenoperation!stetige}, wenn
$\circ: G \times X \rightarrow X$ stetig ist.
\end{defenum}
\end{definition}
@ -172,7 +172,7 @@ $\Rightarrow$ Widerspruch
Da $r > 0$ ist $H_1$ nicht leer, da $r \in \mdr$ ist $H_2$ nicht leer.
\underline{Zu zeigen:} $\forall A \in H_i$, $B \in H_j$ mit
$i,j \in \Set{1,2}$ gilt:
$i,j \in \Set{1,2}$ gilt:
$\overline{AB} \cap g \neq \emptyset \Leftrightarrow i \neq j$\\
\enquote{$\Leftarrow$}: Da $d_\mdh$ stetig ist, folgt diese Richtung
direkt. Alle Punkte in $H_1$ haben einen Abstand von $m$ der kleiner
@ -189,7 +189,7 @@ $\Rightarrow$ Widerspruch
\[\mdh = \underbrace{\Set{z \in \mdh | \Re(z) < x}}_{=: H_1 \text{ (Links)}} \dcup \underbrace{\Set{z \in \mdh | \Re(z) > x}}_{=: H_2 \text{ (Rechts)}}\]
\underline{Zu zeigen:} $\forall A \in H_i$, $B \in H_j$ mit
$i,j \in \Set{1,2}$ gilt:
$i,j \in \Set{1,2}$ gilt:
$\overline{AB} \cap g \neq \emptyset \Leftrightarrow i \neq j$\\
\enquote{$\Leftarrow$}: Wie zuvor mit dem Zwischenwertsatz.
@ -207,7 +207,7 @@ $\Rightarrow$ Widerspruch
\begin{enumerate}
\item Deformationsretrakt: Das hatten wir nicht in der Vorlesung, oder? Ich meine mich zwar an das Wort zu erinnern (aus einem Übungsblatt? Einem Tutorium?) Könntest du bitte nochmals erklären was das ist?
Das ist zwar auf Blatt 7 und 8 vorgekommen, aber sonst nie.
\item Damit verbunden: Was genau ist eine "Einbettung"?
\item Damit verbunden: Was genau ist eine "Einbettung"?
\item Was bedeutet der Pfeil: $f:S^1 \hookrightarrow \mdr^2\;\;\;$ Einbettung der Kreislinie in die Ebene
\item Was ist eine Inklusionsabbildung?
\item Was ist ein Homotopietyp? (Ist das eventuell die Anzahl der Homotopieklassen?)

View file

@ -11,7 +11,7 @@
\item Ist $I$ eine Menge und $U_i \in \fT$ für jedes $i \in I$,
so ist $\displaystyle \bigcup_{i \in I} U_i \in \fT$
\end{defenumprops}
Die Elemente von $\fT$ heißen \textbf{offene Teilmengen} von $X$.
Die Elemente von $\fT$ heißen \textbf{offene Teilmengen} von $X$.
$A \subseteq X$ heißt \textbf{abgeschlossen}, wenn $X \setminus A$ offen ist.
\end{definition}
@ -44,7 +44,7 @@ Auch gibt es Mengen, die sowohl abgeschlossen als auch offen sind.
\item Jeder metrische Raum $(X, d)$ ist auch ein topologischer Raum.
\item Für eine Menge $X$ heißt $\fT_{\ts{Diskret}} = \powerset{X}$ \textbf{diskrete Topologie}\xindex{Topologie!diskrete}.
\item $X :=\mdr, \fT_Z := \Set{U \subseteq \mdr | \mdr \setminus U \text{ endlich}} \cup \Set{\emptyset}$ heißt \textbf{Zariski-Topologie} \xindex{Topologie!Zariski}\\
Beobachtungen:
Beobachtungen:
\begin{itemize}
\item $U \in \fT_Z \gdw \exists f \in \mdr[X]$, sodass $\mdr \setminus U = V(f) = \Set{x \in \mdr | f(x) = 0}$
\item Es gibt keine disjunkten offenen Mengen in $\fT_Z$.
@ -77,10 +77,10 @@ Auch gibt es Mengen, die sowohl abgeschlossen als auch offen sind.
\begin{beispiel}
\begin{bspenum}
\item Sei $X = \mdr$ mit euklidischer Topologie und
$M = \mdq$. Dann gilt: $\overline{M} = \mdr$ und
\item Sei $X = \mdr$ mit euklidischer Topologie und
$M = \mdq$. Dann gilt: $\overline{M} = \mdr$ und
$M^\circ = \emptyset$
\item Sei $X = \mdr$ und $M=(a,b)$. Dann gilt:
\item Sei $X = \mdr$ und $M=(a,b)$. Dann gilt:
$\overline{M} = [a,b]$
\item Sei $X = \mdr, \fT = \fT_Z$ und $M = (a,b)$. Dann gilt:
$\overline{M} = \mdr$
@ -102,14 +102,14 @@ Auch gibt es Mengen, die sowohl abgeschlossen als auch offen sind.
\begin{beispiel}[Basis und Subbasis]
\begin{bspenum}
\item Jede Basis ist auch eine Subbasis, z.B.\\
$S=\Set{ (a,b) | a,b \in \mdr, a<b }$ ist für $\mdr$ mit der
$S=\Set{ (a,b) | a,b \in \mdr, a<b }$ ist für $\mdr$ mit der
Standardtopologie sowohl Basis als auch Subbasis.
\item Gegeben sei $X = \mdr^n$ mit euklidischer Topologie $\fT$. Dann ist
\[\fB = \Set{B_r(x) | r \in \mdq_{> 0}, x \in \mdq^n}\]
ist eine abzählbare Basis von $\fT$.
\item Sei $(X, \fT)$ ein topologischer Raum mit
\item Sei $(X, \fT)$ ein topologischer Raum mit
$X = \Set{0,1,2}$ und $\fT = \Set{\emptyset, \Set{0}, \Set{0,1}, \Set{0,2}, X}$.\\
Dann ist $\calS = \Set{\emptyset, \Set{0,1}, \Set{0,2}}$ eine Subbasis von
Dann ist $\calS = \Set{\emptyset, \Set{0,1}, \Set{0,2}}$ eine Subbasis von
$\fT$, da gilt:
\begin{itemize}
\item $\calS \subseteq \fT$
@ -132,11 +132,11 @@ Auch gibt es Mengen, die sowohl abgeschlossen als auch offen sind.
Sei $(X, \fT)$ ein topologischer Raum und $Y \subseteq X$.\\
$\fT_Y := \Set{U \cap Y | U \in \fT}$ ist eine Topologie auf $Y$.
$\fT_Y$ heißt \textbf{Teilraumtopologie} und $(Y, \fT_Y)$ heißt ein
$\fT_Y$ heißt \textbf{Teilraumtopologie} und $(Y, \fT_Y)$ heißt ein
\textbf{Teilraum} von $(X, \fT)$.
\end{definition}
Die Teilraumtopologie wird auch \textit{Spurtopologie} oder
Die Teilraumtopologie wird auch \textit{Spurtopologie} oder
\textit{Unterraumtopologie} genannt.
%%%%%%%%%%%%%%%%%%%%%%%%%%%%%%%%%%%%%%%%%%%%%%%%%%%%%%%%%%%%%%%%%%%%%
@ -190,14 +190,14 @@ Die Teilraumtopologie wird auch \textit{Spurtopologie} oder
\begin{beispiel}
$X = \mdr, a \sim b :\Leftrightarrow a-b \in \mdz$
\input{figures/number-ray-circle-topology}
$0 \sim 1$, d.~h. $[0] = [1]$
\end{beispiel}
\begin{beispiel}\xindex{Torus}%
Sei $X = \mdr^2$ und $(x_1, y_1) \sim (x_2, y_2) \gdw x_1 - x_2 \in \mdz$
Sei $X = \mdr^2$ und $(x_1, y_1) \sim (x_2, y_2) \gdw x_1 - x_2 \in \mdz$
und $y_1 - y_2 \in \mdz$. Dann ist $X /_\sim$ ein Torus.
\end{beispiel}
@ -234,7 +234,7 @@ Die Teilraumtopologie wird auch \textit{Spurtopologie} oder
\begin{definition}\xindex{Isometrie}\label{def:Isometrie}%
Seien $(X, d_X)$ und $(Y, d_Y)$ metrische Räume und $\varphi: X \rightarrow Y$
eine Abbildung mit
eine Abbildung mit
\[\forall x_1, x_2 \in X: d_X(x_1, x_2) = d_Y(\varphi(x_1), \varphi(x_2)) \]
Dann heißt $\varphi$ eine \textbf{Isometrie} von $X$ nach $Y$.
@ -252,7 +252,7 @@ Die Teilraumtopologie wird auch \textit{Spurtopologie} oder
0 & \text{falls } x=y\\
1 & \text{falls } x \neq y
\end{cases}\]
die \textbf{diskrete Metrik}. Die Metrik $d$ induziert die
die \textbf{diskrete Metrik}. Die Metrik $d$ induziert die
\textbf{diskrete Topologie}.
\end{beispiel}
\clearpage
@ -280,7 +280,7 @@ Die Teilraumtopologie wird auch \textit{Spurtopologie} oder
\end{beispiel}
\clearpage
\begin{beispiel}[SNCF-Metrik\footnotemark]\xindex{Metrik!SNCF}
$X = \mdr^2$
$X = \mdr^2$
\input{figures/sncf-metrik}
\end{beispiel}
@ -293,7 +293,7 @@ Die Teilraumtopologie wird auch \textit{Spurtopologie} oder
\end{definition}
\begin{bemerkung}[Trennungseigenschaft]\label{Trennungseigenschaft}
Metrische Räume sind hausdorffsch, wegen
Metrische Räume sind hausdorffsch, wegen
\[d(x, y) > 0 \Rightarrow \exists \varepsilon > 0: \fB_\varepsilon(x) \cap \fB_\varepsilon(y) = \emptyset\]
\end{bemerkung}
@ -337,7 +337,7 @@ Die Teilraumtopologie wird auch \textit{Spurtopologie} oder
Sei $(x_n)$ eine konvergierende Folge und $x$ und $y$ Grenzwerte der Folge.
Da $X$ hausdorffsch ist, gibt es Umgebungen $U_x$ von $x$ und $U_y$
von $y$ mit $U_x \cap U_y = \emptyset$ falls $x \neq y$. Da
von $y$ mit $U_x \cap U_y = \emptyset$ falls $x \neq y$. Da
$(x_n)$ gegen $x$ und $y$ konvergiert, existiert ein
$n_0$ mit $x_n \in U_x \cap U_y$ für alle $n \geq n_0$
$\Rightarrow x = y \qed$
@ -345,14 +345,14 @@ Die Teilraumtopologie wird auch \textit{Spurtopologie} oder
\section{Stetigkeit}\index{Stetigkeit|(}
\begin{definition}
Seien $(X, \fT_X), (Y, \fT_Y)$ topologische Räume und
Seien $(X, \fT_X), (Y, \fT_Y)$ topologische Räume und
$f:X \rightarrow Y$ eine Abbildung.
\begin{defenum}
\item \label{def:stetigkeit} $f$ heißt \textbf{stetig}\xindex{Abbildung!stetige}
$:\gdw \forall U \in \fT_Y: f^{-1} (U) \in \fT_X$.
\item \label{def:homoeomorphismus} $f$ heißt \textbf{Homöomorphismus}\xindex{Homöomorphismus}, wenn $f$ stetig ist
und es eine
und es eine
stetige Abbildung $g: Y \rightarrow X$ gibt, sodass
$g \circ f = \id_X$ und $f \circ g = \id_Y$.
\end{defenum}
@ -378,9 +378,9 @@ Die Teilraumtopologie wird auch \textit{Spurtopologie} oder
\enquote{$\Rightarrow$}: Sei $x \in X, \varepsilon > 0$ gegeben
und $U := \fB_\varepsilon(f(x))$.\\
Dann ist $U$ offen in $Y$.\\
$\xRightarrow{\crefabbr{def:stetigkeit}} f^{-1}(U)$ ist
$\xRightarrow{\crefabbr{def:stetigkeit}} f^{-1}(U)$ ist
offen in $X$. Dann ist $x \in f^{-1}(U)$.\\
$\Rightarrow \exists \delta > 0$, sodass
$\Rightarrow \exists \delta > 0$, sodass
$\fB_\delta(x) \subseteq f^{-1} (U)$\\
$\Rightarrow f(\fB_\delta(x)) \subseteq U$\\
$\Rightarrow \Set{y \in X | d_X(x,y) < \delta} \Rightarrow$ Beh.
@ -415,18 +415,18 @@ Die Teilraumtopologie wird auch \textit{Spurtopologie} oder
\begin{figure}[htp]
\centering
\input{figures/topology-continuous-mapping}
\caption{Beispiel einer stetigen Funktion $f$, deren
\caption{Beispiel einer stetigen Funktion $f$, deren
Umkehrabbildung $g$ nicht stetig ist.}
\label{fig:nicht-stetige-umkehrabbildung}
\end{figure}
Die Umkehrabbildung $g$ ist nicht stetig, da $g^{-1}(U)$
nicht offen ist (vgl. \cref{fig:nicht-stetige-umkehrabbildung}).
\end{bspenum}
\end{beispiel}
\begin{bemerkung}[Verkettungen stetiger Abbildungen sind stetig]
Seien $X, Y, Z$ topologische Räume, $f:X \rightarrow Y$ und
Seien $X, Y, Z$ topologische Räume, $f:X \rightarrow Y$ und
$g:Y \rightarrow Z$ stetige Abbildungen.
Dann ist $g \circ f: X \rightarrow Z$ stetig.
@ -449,10 +449,10 @@ Die Teilraumtopologie wird auch \textit{Spurtopologie} oder
\begin{bemerkung}
\begin{bemenum}
\item \xindex{Homöomorphismengruppe}Für jeden topologischen Raum $X$ ist
\item \xindex{Homöomorphismengruppe}Für jeden topologischen Raum $X$ ist
\[\Homoo(X) := \Set{f: X \rightarrow X | f \text{ ist Homöomorphismus}}\]
eine Gruppe.
\item \xindex{Isometrie}Jede Isometrie $f:X \rightarrow Y$ zwischen metrischen
\item \xindex{Isometrie}Jede Isometrie $f:X \rightarrow Y$ zwischen metrischen
Räumen ist ein Homöomorphismus.
\item \xindex{Isometriegruppe}$\Iso(X) := \Set{f:X \rightarrow X | f \text{ ist Isometrie}}$ ist
eine Untergruppe von $\Homoo(X)$ für jeden
@ -462,7 +462,7 @@ Die Teilraumtopologie wird auch \textit{Spurtopologie} oder
\begin{bemerkung}[Projektionen sind stetig]
Seien $X, Y$ topologische Räume. $\pi_X: X \times Y \rightarrow X$
und $\pi_Y: X \times Y \rightarrow Y$ die Projektionen
und $\pi_Y: X \times Y \rightarrow Y$ die Projektionen
\[\pi_X: (x,y) \mapsto x \text{ und } \pi_Y: (x,y) \mapsto y\]
Wird $X \times Y$ mit der Produkttopologie versehen, so sind $\pi_X$
und $\pi_Y$ stetig.
@ -482,8 +482,8 @@ Die Teilraumtopologie wird auch \textit{Spurtopologie} oder
\end{bemerkung}
\begin{beweis}
Nach Definition ist
$U \subseteq \overline{X}$ offen $\gdw \pi^{-1}(U) \subseteq X$
Nach Definition ist
$U \subseteq \overline{X}$ offen $\gdw \pi^{-1}(U) \subseteq X$
offen. $\qed$
\end{beweis}
@ -498,7 +498,7 @@ sodass $\pi$ stetig wird.
S^n &= \Set{x \in \mdr^{n+1} | \|x\| = 1}\\
&= \Set{x \in \mdr^{n+1} | \sum_{i=1}^{n+1} x_i^2 = 1}
\end{align*}
\Obda sei $N = \begin{pmatrix}0\\ \vdots\\ 0\\1\end{pmatrix}$. Die
Gerade durch $N$ und $P$ schneidet die Ebene $H$ in genau einem
Punkt $\hat{P}$. $P$ wird auf $\hat{P}$ abgebildet.
@ -523,7 +523,7 @@ sodass $\pi$ stetig wird.
schneiden sich $L_P$ und $H$ in genau einem Punkt $\hat{P}$.
Es gilt: $f$ ist bijektiv und die Umkehrabbildung ist ebenfalls
stetig.
stetig.
\end{beispiel}
\index{Stetigkeit|)}
%%%%%%%%%%%%%%%%%%%%%%%%%%%%%%%%%%%%%%%%%%%%%%%%%%%%%%%%%%%%%%%%%%%%%
@ -533,7 +533,7 @@ sodass $\pi$ stetig wird.
\begin{definition}\xindex{Raum!zusammenhaengender@zusammenhängender}\xindex{Menge!zusammenhaengende@zusammenhängende}%
\begin{defenum}
\item Ein Raum $X$ heißt \textbf{zusammenhängend}, wenn es keine offenen,
nichtleeren Teilmengen $U_1, U_2$ von $X$ gibt mit
nichtleeren Teilmengen $U_1, U_2$ von $X$ gibt mit
$U_1 \cap U_2 = \emptyset$ und $U_1 \cup U_2 = X$.
\item Eine Teilmenge $Y \subseteq X$ heißt zusammenhängend, wenn $Y$
als topologischer Raum mit der Teilraumtopologie zusammenhängend ist.
@ -542,7 +542,7 @@ sodass $\pi$ stetig wird.
\begin{bemerkung}
$X$ ist zusammenhängend $\gdw$ Es gibt keine abgeschlossenen,
nichtleeren Teilmengen $A_1, A_2$ mit $A_1 \cap A_2 = \emptyset$
nichtleeren Teilmengen $A_1, A_2$ mit $A_1 \cap A_2 = \emptyset$
und $A_1 \cup A_2 = X$.
\end{bemerkung}
@ -553,20 +553,20 @@ sodass $\pi$ stetig wird.
\underline{Annahme}: $\mdr^n = U_1 \dcup U_2$ mit $\emptyset \neq U_1, U_2 \in \fT_{\ts{Euklid}}$ existieren.
Sei $x \in U_1, y \in U_2$ und $[x,y]$ die Strecke zwischen $x$
und $y$. Sei $V = [x,y]$. Nun betrachten wir $V \subsetneq \mdr^n$ als
und $y$. Sei $V = [x,y]$. Nun betrachten wir $V \subsetneq \mdr^n$ als
(metrischen) Teilraum mit der Teilraumtopologie $\fT_V$.
Somit gilt $U_1 \cap [x,y] \in \fT_V$ wegen der Definition der
Somit gilt $U_1 \cap [x,y] \in \fT_V$ wegen der Definition der
Teilraumtopologie.
Dann gibt es $z \in [x,y]$ mit $z \in \partial (U_1 \cap [x,y])$,
aber $z \notin U_1 \Rightarrow z \in U_2$. In jeder Umgebung von
aber $z \notin U_1 \Rightarrow z \in U_2$. In jeder Umgebung von
$z$ liegt ein Punkt von $U_1 \Rightarrow$ Widerspruch zu $U_2$ offen.
\item $\mdr \setminus \Set{0}$ ist nicht zusammenhängend, denn
$\mdr \setminus \Set{0} = \mdr_{< 0} \cup \mdr_{> 0}$
\item $\mdr^2 \setminus \Set{0}$ ist zusammenhängend.
\item $\mdq \subsetneq \mdr$ ist nicht zusammenhängend, da
\item $\mdq \subsetneq \mdr$ ist nicht zusammenhängend, da
$(\mdq \cap \mdr_{< \sqrt{2}}) \cup (\mdq \cap \mdr_{> \sqrt{2}}) = \mdq$
\item $\Set{x}$ ist zusammenhängend für jedes $x \in X$,
\item $\Set{x}$ ist zusammenhängend für jedes $x \in X$,
wobei $X$ ein topologischer Raum ist.
\item $\mdr$ mit Zariski-Topologie ist zusammenhängend.\xindex{Topologie!Zariski}
\end{bspenum}
@ -590,7 +590,7 @@ sodass $\pi$ stetig wird.
$\Rightarrow \overline{A} \subseteq A_2$\\
$\Rightarrow A_1 = \emptyset$\\
$\Rightarrow$ Widerspruch zu $A_1 \neq \emptyset$\\
$\Rightarrow A \cap A_1 \neq \emptyset$ und analog
$\Rightarrow A \cap A_1 \neq \emptyset$ und analog
$A \cap A_2 \neq \emptyset$\\
$\Rightarrow$ Widerspruch zu $A$ ist zusammenhängend. $ \qed$
\end{beweis}
@ -614,7 +614,7 @@ sodass $\pi$ stetig wird.
\begin{definition}\xindex{Zusammenhangskomponente}%
Sei $X$ ein topologischer Raum.
Für $x \in X$ sei $Z(x) \subseteq X$ definiert durch
\[Z(x) := \bigcup_{\mathclap{\substack{A \subseteq X \text{zhgd.}\\ x \in A}}} A\]
@ -647,7 +647,7 @@ sodass $\pi$ stetig wird.
\begin{align*}
\Rightarrow Z(x) \cup Z(y) &\subseteq Z(x) \Rightarrow Z(y) \subseteq Z(x)\\
&\subseteq Z(y) \Rightarrow Z(x) \subseteq Z(y)
\end{align*}
\end{align*}
\end{enumerate}
$\qed$
@ -678,7 +678,7 @@ sodass $\pi$ stetig wird.
Ein topologischer Raum $X$ heißt \textbf{kompakt}, wenn jede
offene Überdeckung von $X$
\[\fU = \Set{U_i}_{i \in I} \text{ mit } U_i \text{ offen in } X\]
eine endliche Teilüberdeckung
eine endliche Teilüberdeckung
\[\bigcup_{\mathclap{i \in J \subseteq I}} U_i = X \text{ mit } |J| \in \mdn\]
besitzt.
\end{definition}
@ -688,24 +688,24 @@ sodass $\pi$ stetig wird.
%%%%%%%%%%%%%%%%%%%%%%%%%%%%%%%%%%%%%%%%%%%%%%%%%%%%%%%%%%%%%%%%%%%%%
\begin{bemerkung}\label{abgeschlossen01IstKompakt}
Das Einheitsintervall $I := [0,1]$ ist kompakt bezüglich der
Das Einheitsintervall $I := [0,1]$ ist kompakt bezüglich der
euklidischen Topologie.
\end{bemerkung}
\begin{beweis}
Sei $(U_i)_{i \in J}$ eine offene Überdeckung von $I$.
Es genügt zu zeigen, dass es ein $\delta > 0$ gibt, sodass jedes
Es genügt zu zeigen, dass es ein $\delta > 0$ gibt, sodass jedes
Teilintervall der Länge $\delta$ von $I$ in einem der $U_i$ enthalten ist.
Wenn es ein solches $\delta$ gibt, kann man $I$ in endlich viele
Wenn es ein solches $\delta$ gibt, kann man $I$ in endlich viele
Intervalle der Länge $\delta$ unterteilen und alle $U_i$ in die endliche
Überdeckung aufnehmen, die Teilintervalle enthalten.
Angenommen, es gibt kein solches $\delta$. Dann gibt es für jedes
Angenommen, es gibt kein solches $\delta$. Dann gibt es für jedes
$n \in \mdn$ ein Intervall $I_n \subseteq [0,1]$ der Länge $\nicefrac{1}{n}$
sodass $I_n \subsetneq U_i$ für alle $i \in J$.
Sei $x_n$ der Mittelpunkt von $I_n$. Die Folge $(x_n)$ hat einen
Sei $x_n$ der Mittelpunkt von $I_n$. Die Folge $(x_n)$ hat einen
Häufungspunkt $x \in [0,1]$. Dann gibt es $i \in J$ mit $x \in U_i$.
Da $U_i$ offen ist, gibt es ein $\varepsilon > 0$, sodass $(x - \varepsilon, x + \varepsilon) \subseteq U_i$.
Dann gibt es $n_0$, sodass gilt:
@ -713,7 +713,7 @@ $\nicefrac{1}{n_0} < \nicefrac{\varepsilon}{2}$ und für unendlich viele\footnot
$n\geq n_0: |x - x_n| < \nicefrac{\varepsilon}{2}$, also $I_n \subseteq (x - \varepsilon, x + \varepsilon) \subseteq U_i$
für mindestens ein $n \in \mdn$.\footnote{Sogar für unendlich viele.}
$\Rightarrow$ Widerspruch
$\Rightarrow$ Widerspruch
Dann überdecke $[0,1]$ mit endlich vielen Intervallen $I_1, \dots, I_d$
der Länge $\delta$. Jedes $I_j$ ist in $U_{ij}$ enthalten.
@ -727,7 +727,7 @@ $\qed$
\item $\mdr$ ist nicht kompakt.
\item $(0,1)$ ist nicht kompakt.\\
$U_n = (\nicefrac{1}{n}, 1-\nicefrac{1}{n}) \Rightarrow \bigcup_{n \in \mdn} U_n = (0,1)$
\item $\mdr$ mit der Zariski-Topologie ist kompakt und jede
\item $\mdr$ mit der Zariski-Topologie ist kompakt und jede
Teilmenge von $\mdr$ ist es auch.\xindex{Topologie!Zariski}
\end{bspenum}
\end{beispiel}
@ -770,12 +770,12 @@ $\qed$
Die offenen Mengen $U_{x_0, y} \times V_{x_0, y}$ für festes $x_0$
und alle $y \in Y$ überdecken $\Set{x_0} \times y$. Da $Y$ kompakt
ist, ist auch $\Set{x_0} \times Y$ kompakt. Also gibt es
$y_1, \dots, y_{m(x_0)}$ mit
ist, ist auch $\Set{x_0} \times Y$ kompakt. Also gibt es
$y_1, \dots, y_{m(x_0)}$ mit
$\bigcup_{i=1}^{m(x_0)} U_{x_0, y_i} \times V_{x_0, y_i} \supseteq \Set{x_0} \times Y$.
Sei ${\color{blue} U_{x_0}} := \bigcap_{i=1}^{m(x)} U_{x_0, y_i}$.
Da $X$ kompakt ist, gibt es $x_1, \dots, x_n \in X$ mit
Da $X$ kompakt ist, gibt es $x_1, \dots, x_n \in X$ mit
$\bigcup_{j=1}^n U_{x_j} = X$\\
$\Rightarrow \bigcup_{j=1}^k \bigcup_{i=1}^{m(x_j)} \underbrace{\left ( U_{x_j, y_i} \times V_{x_j, y_i} \right)}_{\mathclap{\text{Ein grün-oranges Kästchen}}} \supseteq X \times Y$\\
$\Rightarrow \bigcup_j \bigcup_i W_i (x_j, y_i) = X \times Y \qed$
@ -789,7 +789,7 @@ $\qed$
\begin{beweis}
\underline{z.~Z.:} Komplement ist offen
Ist $X = K$, so ist $K$ abgeschlossen in $X$. Andernfalls sei
Ist $X = K$, so ist $K$ abgeschlossen in $X$. Andernfalls sei
$y \in X \setminus K$. Für jedes $x \in K$ seien $U_x$ bzw. $V_y$
Umgebungen von $x$ bzw. von $y$, sodass $U_x \cap V_y = \emptyset$.
@ -819,10 +819,10 @@ $\qed$
\begin{beweis}
Sei $(V_i)_{i \in I}$ offene Überdeckung von $f(K)$\\
$\xRightarrow{f \text{ stetig}} (f^{-1}(V_i))_{i \in I}$ ist offene Überdeckung von $K$\\
$\xRightarrow{\text{Kompakt}}$ es gibt $i_1, \dots, i_n$,
$\xRightarrow{\text{Kompakt}}$ es gibt $i_1, \dots, i_n$,
sodass $f^{-1}(V_{i_1}), \dots, f^{-1}(V_{i_n})$ Überdeckung von
$K$ ist.\\
$\Rightarrow f(f^{-1}( V_{i_1})), \dots, f(f^{-1}(V_{i_n}))$
$\Rightarrow f(f^{-1}( V_{i_1})), \dots, f(f^{-1}(V_{i_n}))$
überdecken $f(K)$.
Es gilt: $f(f^{-1}(V)) = V \cap f(X) \qed$
@ -839,7 +839,7 @@ $\qed$
Da $\mdr^n$ und $\mdc^n$ hausdorffsch sind, ist $K$ nach
\cref{hausdorffraumKompakteTeilmengeAbgeschlossen} abgeschlossen.
Nach Voraussetzung kann $K$ mit endlich vielen offenen Kugeln von
Nach Voraussetzung kann $K$ mit endlich vielen offenen Kugeln von
Radien 1 überdeckt werden $\Rightarrow K$ ist beschränkt.
\enquote{$\Leftarrow$} Sei $A \subseteq \mdr^n$ (oder $\mdc^n$)
@ -852,7 +852,7 @@ $\qed$
Nach \cref{kompaktTimesKompaktIstKompakt} und
\cref{abgeschlossen01IstKompakt} ist $W$ kompakt, also ist $A$
nach \cref{abgeschlossenInKomaktIstKompakt} auch kompakt.
Genauso ist $Z$ kompakt, weil
Genauso ist $Z$ kompakt, weil
\[\Set{z \in \mdc | |z| \leq 1}\]
homöomorph zu
\[\Set{(x,y) \in \mdr^2 | \|(x,y)\| \leq 1}\]
@ -864,11 +864,11 @@ $\qed$
%%%%%%%%%%%%%%%%%%%%%%%%%%%%%%%%%%%%%%%%%%%%%%%%%%%%%%%%%%%%%%%%%%%%%
\section{Wege und Knoten}\index{Knoten|(}
\begin{definition}\xindex{Weg}\xindex{Weg!geschlossener}\xindex{Weg!einfacher}%
Sei $X$ ein topologischer Raum.
Sei $X$ ein topologischer Raum.
\begin{defenum}
\item Ein \textbf{Weg} in $X$ ist eine stetige Abbildung $\gamma:[0,1] \rightarrow X$.
\item $\gamma$ heißt \textbf{geschlossen}, wenn $\gamma(1) = \gamma(0)$ gilt.
\item $\gamma$ heißt \textbf{einfach}, wenn $\gamma|_{[0,1)}$
\item $\gamma$ heißt \textbf{einfach}, wenn $\gamma|_{[0,1)}$
injektiv ist.
\end{defenum}
\end{definition}
@ -901,11 +901,11 @@ $\qed$
$A_1 \cup A_2 = X$. Sei $x \in A_1, y \in A_2, \gamma:[0,1] \rightarrow X$
ein Weg von $x$ nach $y$.
Dann ist $C:= \gamma([0,1]) \subseteq X$ zusammenhängend, weil
Dann ist $C:= \gamma([0,1]) \subseteq X$ zusammenhängend, weil
$\gamma$ stetig ist.
\[C = \underbrace{(C \cap A_1)}_{\ni x} \cup \underbrace{(C \cap A_2)}_{\ni y}\]
ist Zerlegung in nichtleere, disjunkte, abgeschlossene Teilmengen
$\Rightarrow$ Widerspruch
$\Rightarrow$ Widerspruch
\item Sei $X = \Set{(x,y) \in \mdr^2| x^2 + y^2 = 1 \lor y = 1 +2\cdot e^{-\frac{1}{10} x}}$.
@ -943,7 +943,7 @@ $\qed$
\end{beweis}
\begin{beispiel}[Hilbert-Kurve]\xindex{Hilbert-Kurve}%
Es gibt stetige, surjektive Abbildungen
Es gibt stetige, surjektive Abbildungen
$[0,1] \rightarrow [0,1] \times [0,1]$. Ein Beispiel ist die
in \cref{fig:hilbert-curve} dargestellte Hilbert-Kurve.
@ -952,7 +952,7 @@ $\qed$
\begin{definition}\xindex{Jordankurve}\xindex{Jordankurve!geschlossene}%
Sei $X$ ein topologischer Raum. Eine
\textbf{Jordankurve} in $X$ ist ein Homöomorphismus
\textbf{Jordankurve} in $X$ ist ein Homöomorphismus
$\gamma: [0,1] \rightarrow C \subseteq X$ bzw.
$\gamma: S^1 \rightarrow C \subseteq X$, wobei $C := \Bild{\gamma}$.
\end{definition}
@ -967,7 +967,7 @@ Jede Jordankurve ist also ein einfacher Weg.
\begin{figure}[htp]
\centering
\input{figures/topology-jordan}
\input{figures/topology-jordan}
\label{fig:jordan-kurvensatz}
\caption{Die unbeschränkte Zusammenhangskomponente wird häufig inneres, die beschränkte äußeres genannt.}
\end{figure}
@ -993,15 +993,15 @@ Jede Jordankurve ist also ein einfacher Weg.
\label{fig:knot-unknot}
}%
\subfloat[Kleeblattknoten]{
\includegraphics[width=0.2\linewidth, keepaspectratio]{figures/blue-trefoil-knot.png}
\includegraphics[width=0.2\linewidth, keepaspectratio]{figures/blue-trefoil-knot.png}
\label{fig:knot-trefoil}
}%
\subfloat[Achterknoten]{
\includegraphics[width=0.2\linewidth, keepaspectratio]{figures/blue-eight-knot.png}
\includegraphics[width=0.2\linewidth, keepaspectratio]{figures/blue-eight-knot.png}
\label{fig:knot-eight-knot}
}%
\subfloat[$6_2$-Knoten]{
\includegraphics[width=0.2\linewidth, keepaspectratio]{figures/blue-6-2-knot.png}
\includegraphics[width=0.2\linewidth, keepaspectratio]{figures/blue-6-2-knot.png}
\label{fig:knot-6-2}
}
@ -1014,20 +1014,20 @@ Jede Jordankurve ist also ein einfacher Weg.
Zwei Knoten $\gamma_1, \gamma_2: S^1 \rightarrow \mdr^3$ heißen
\textbf{äquivalent}, wenn es eine stetige Abbildung
\[H: S^1 \times [0,1] \rightarrow \mdr^3\]
gibt mit
gibt mit
\begin{align*}
H(z,0) &= \gamma_1(z) \;\;\;\forall z \in S^1\\
H(z,1) &= \gamma_2(z) \;\;\;\forall z \in S^1
\end{align*}
und für jedes
feste $t \in [0,1]$ ist
feste $t \in [0,1]$ ist
\[H_z: S^1 \rightarrow \mdr^3, z \mapsto H(z,t)\]
ein Knoten. Die Abbildung $H$ heißt \textbf{Isotopie} zwischen
$\gamma_1$ und $\gamma_2$.
\end{definition}
\begin{definition}\xindex{Knotendiagramm}%
Sei $\gamma: [0,1] \rightarrow \mdr^3$ ein Knoten, $E$ eine Ebene und
Sei $\gamma: [0,1] \rightarrow \mdr^3$ ein Knoten, $E$ eine Ebene und
$\pi: \mdr^3 \rightarrow E$ eine Projektion auf $E$.
$\pi$ heißt \textbf{Knotendiagramm} von $\gamma$, wenn gilt:
@ -1047,16 +1047,16 @@ Jede Jordankurve ist also ein einfacher Weg.
\begin{figure}[htp]
\centering
\subfloat[$\Omega_1$]{
\includegraphics[height=0.2\linewidth, keepaspectratio]{figures/reidemeister-move-1.png}
\includegraphics[height=0.2\linewidth, keepaspectratio]{figures/reidemeister-move-1.png}
\label{fig:reidemeister-1}
}\qquad\qquad%
\subfloat[$\Omega_2$]{
\includegraphics[height=0.2\linewidth, keepaspectratio]{figures/reidemeister-move-2.png}
\includegraphics[height=0.2\linewidth, keepaspectratio]{figures/reidemeister-move-2.png}
\label{fig:reidemeister-2}
}
\subfloat[$\Omega_3$]{
\includegraphics[height=0.2\linewidth, keepaspectratio]{figures/reidemeister-move-3.png}
\includegraphics[height=0.2\linewidth, keepaspectratio]{figures/reidemeister-move-3.png}
\label{fig:reidemeister-3}
}
@ -1069,15 +1069,15 @@ Jede Jordankurve ist also ein einfacher Weg.
\end{beweis}
\begin{definition}\xindex{Färbbarkeit}%
Ein Knotendiagramm heißt \textbf{3-färbbar},
wenn jeder Bogen von $D$ so mit einer Farbe gefärbt werden kann,
dass an jeder Kreuzung eine oder 3 Farben auftreten und alle 3
Ein Knotendiagramm heißt \textbf{3-färbbar},
wenn jeder Bogen von $D$ so mit einer Farbe gefärbt werden kann,
dass an jeder Kreuzung eine oder 3 Farben auftreten und alle 3
Farben auftreten.
\end{definition}
\begin{figure}[htp]
\centering
\includegraphics[height=0.3\linewidth, keepaspectratio]{figures/tricoloring.png}
\includegraphics[height=0.3\linewidth, keepaspectratio]{figures/tricoloring.png}
\caption{Ein 3-gefärber Kleeblattknoten}
\label{fig:treefoil-knot-three-colors}

View file

@ -15,7 +15,7 @@
Familie $(U_i, \varphi_i)_{i \in I}$ von Karten auf $X$,
sodass $\bigcup_{i \in I} U_i = X$.
\item $X$ heißt (topologische) $n$-dimensionale \textbf{Mannigfaltigkeit}\xindex{Mannigfaltigkeit},
wenn $X$ hausdorffsch ist, eine abzählbare Basis der
wenn $X$ hausdorffsch ist, eine abzählbare Basis der
Topologie hat und einen $n$-dimensionalen Atlas besitzt.
\end{defenum}
\end{definition}
@ -27,13 +27,13 @@ Anschaulich ist also ein $n$-dimensionale Mannigfaltigkeit lokal dem $\mdr^n$ ä
\end{bemerkung}
\begin{beweis}
Sei $(X, \fT)$ ein topologischer Raum und $(U, \varphi)$ mit $U \in \fT$
Sei $(X, \fT)$ ein topologischer Raum und $(U, \varphi)$ mit $U \in \fT$
und $\varphi:U \rightarrow V \subseteq \mdr^n$,
wobei $V$ offen und $\varphi$ ein Homöomorphismus ist, eine Karte auf $X$.
Da jede offene Teilmenge des $\mdr^n$ genauso mächtig ist wie der $\mdr^n$,
$\varphi$ als Homöomorphismus insbesondere bijektiv ist und Mengen, zwischen
denen eine Bijektion existiert, gleich mächtig sind, ist $U$ genauso mächtig
$\varphi$ als Homöomorphismus insbesondere bijektiv ist und Mengen, zwischen
denen eine Bijektion existiert, gleich mächtig sind, ist $U$ genauso mächtig
wie der $\mdr^n$. Da jede Mannigfaltigkeit mindestens eine Karte hat, muss
jede Mannigfaltigkeit $X$ mindestens so mächtig sein wie der $\mdr^n$. $\qed$
\end{beweis}
@ -59,8 +59,8 @@ Mannigfaltigkeiten können beliebig viele Elemente haben.
\begin{beispiel}[Mannigfaltigkeiten]
\begin{bspenum}
\item Jede offene Teilmenge $U \subseteq \mdr^n$ ist eine
$n$-dimensionale Mannigfaltigkeit mit einem Atlas aus
\item Jede offene Teilmenge $U \subseteq \mdr^n$ ist eine
$n$-dimensionale Mannigfaltigkeit mit einem Atlas aus
einer Karte.
\item $\mdc^n$ ist eine $2n$-dimensionale Mannigfaltigkeit
mit einem Atlas aus einer Karte:
@ -106,7 +106,7 @@ Mannigfaltigkeiten können beliebig viele Elemente haben.
Es gibt keine Umgebung von $0$ in $[0,1]$, die homöomorph
zu einem offenem Intervall ist.
\item $V_1 = \Set{(x,y) \in \mdr^2 | x \cdot y = 0}$ ist
keine Mannigfaltigkeit.
keine Mannigfaltigkeit.
Das Problem ist $(0,0)$. Wenn man diesen Punkt entfernt,
zerfällt der Raum in 4 Zusammenhangskomponenten.
@ -116,7 +116,7 @@ Mannigfaltigkeiten können beliebig viele Elemente haben.
Mannigfaltigkeit.
\item $X = (\mdr \setminus \Set{0}) \cup (0_1, 0_2)$ \label{bsp:mannigfaltigkeit8}
\[U \subseteq X \text{ offen } \gdw
\[U \subseteq X \text{ offen } \gdw
\begin{cases}
U \text{ offen in } \mdr \setminus \Set{0}, &\text{falls } 0_1 \notin U, 0_2 \in U\\
\exists \varepsilon > 0: (-\varepsilon, \varepsilon) \subseteq U &\text{falls } 0_1 \in U, 0_2 \in U
@ -128,7 +128,7 @@ Mannigfaltigkeiten können beliebig viele Elemente haben.
\underline{Aber:} $X$ ist nicht hausdorffsch!
Denn es gibt keine disjunkten Umgebungen von $0_1$ und
$0_2$.
\item \label{bsp:gln-ist-mf}\xindex{Gruppe!allgemeine lineare}$\GL_n(\mdr)$ ist eine Mannigfaltigkeit der Dimension
\item \label{bsp:gln-ist-mf}\xindex{Gruppe!allgemeine lineare}$\GL_n(\mdr)$ ist eine Mannigfaltigkeit der Dimension
$n^2$, weil offene Teilmengen von $\mdr^{n^2}$ eine
Mannigfaltigkeit bilden.
\end{bspenum}
@ -141,12 +141,12 @@ Mannigfaltigkeiten können beliebig viele Elemente haben.
Seien $X, Y$ $n$-dimensionale Mannigfaltigkeiten, $U \subseteq X$
und $V \subseteq Y$ offen, $\Phi: U \rightarrow V$ ein Homöomorphismus
$Z = (X \dcup Y) /_\sim$ mit der von $u \sim \Phi(u)\;\forall{u \in U}$
erzeugten Äquivalenzrelation und der von $\sim$ induzierten
erzeugten Äquivalenzrelation und der von $\sim$ induzierten
Quotiententopologie.
$Z$ heißt \textbf{Verklebung} von $X$ und $Y$ längs $U$ und $V$.
$Z$ besitzt einen Atlas aus $n$-dimensionalen Karten.
Falls $Z$ hausdorffsch ist, ist $Z$ eine $n$-dimensionale
Falls $Z$ hausdorffsch ist, ist $Z$ eine $n$-dimensionale
Mannigfaltigkeit.
\end{definition}
@ -210,7 +210,7 @@ Mannigfaltigkeiten können beliebig viele Elemente haben.
$G: U \rightarrow \mdr^n, \; u \mapsto (g(u), u)$
eine stetige Abbildung auf eine offene Umgebung $V$ von
$x$ in $X$ ist.
\end{enumerate}
\end{enumerate}
$\qed$
\end{beweis}
@ -244,8 +244,8 @@ Mannigfaltigkeiten können beliebig viele Elemente haben.
Sei $X$ ein Hausdorffraum mit abzählbarer Basis der Topologie.
$X$ heißt $n$-dimensionale \textbf{Mannigfaltigkeit mit Rand},
wenn es einen Atlas $(U_i, \varphi_i)$ gibt, wobei $U_i \subseteq X_i$
offen und $\varphi_i$ ein Homöomorphismus auf eine offene
Teilmenge von
offen und $\varphi_i$ ein Homöomorphismus auf eine offene
Teilmenge von
\[\mdr_{+,0}^n := \Set{(x_1, \dots, x_n) \in \mdr^n | x_n \geq 0}\]
ist.
\end{definition}
@ -275,7 +275,7 @@ $\mdr_{+,0}^n$ ist ein \enquote{Halbraum}\xindex{Halbraum}.
\begin{definition}\xindex{Rand}%
Sei $X$ eine $n$-dimensionale Mannigfaltigkeit mit Rand und
Atlas $\atlas$. Dann heißt
Atlas $\atlas$. Dann heißt
\[\partial X := \bigcup_{(U, \varphi) \in \atlas} \Set{x \in U | \varphi (x) = 0}\]
\textbf{Rand} von $X$.
\end{definition}
@ -322,7 +322,7 @@ Differenzierbare Mannigfaltigkeiten der Klasse $C^\infty$ werden auch
\textit{glatt} genannt.
\begin{definition}%
Sei $X$ eine differenzierbare Mannigfaltigkeit der Klasse $C^k$
Sei $X$ eine differenzierbare Mannigfaltigkeit der Klasse $C^k$
($k \in \mdn \cup \Set{\infty}$) mit Atlas $\atlas = (U_i, \varphi_i)_{i \in I}$.
\begin{defenum}
@ -330,10 +330,10 @@ Differenzierbare Mannigfaltigkeiten der Klasse $C^\infty$ werden auch
mit $\atlas$, wenn alle Kartenwechsel $\varphi \circ \varphi_i^{-1}$
und $\varphi_i \circ \varphi^{-1}$ ($i \in I$ mit $U_i \cap U \neq \emptyset$)
differenzierbar von Klasse $C^k$ sind.
\item Die Menge aller mit $\atlas$ verträglichen Karten auf
\item Die Menge aller mit $\atlas$ verträglichen Karten auf
$X$ bildet einen maximalen Atlas der Klasse $C^k$. Er
heißt \textbf{$C^k$-Struktur}\xindex{Ck-Struktur@$C^k$-Struktur} auf $X$.
Eine $C^\infty$-Struktur heißt auch \textbf{differenzierbare Struktur}\xindex{Struktur!differenzierbare}
auf $X$.
\end{defenum}
@ -357,7 +357,7 @@ Differenzierbare Mannigfaltigkeiten der Klasse $C^\infty$ werden auch
gibt, sodass $\psi \circ f \circ \varphi^{-1}$ stetig
differenzierbar von Klasse $C^k$ in $\varphi(x)$ ist.
\item $f$ heißt \textbf{differenzierbar}
(von Klasse $C^k$), wenn $f$ in jedem $x \in X$
(von Klasse $C^k$), wenn $f$ in jedem $x \in X$
differenzierbar ist.
\item $f$ heißt \textbf{Diffeomorphismus}\xindex{Diffeomorphismus},
wenn $f$ differenzierbar von Klasse $C^\infty$ ist und
@ -375,7 +375,7 @@ Differenzierbare Mannigfaltigkeiten der Klasse $C^\infty$ werden auch
\begin{beweis}
Seien $(U', \varphi')$ und $(V', \psi')$ Karten von $X$ bzw. $Y$
um $x$ bzw. $f(x)$ mit $f(U') \subseteq V'$.
$\Rightarrow \psi' \circ f \circ (\varphi')^{-1}$\\
$= \psi' \circ ( \psi^{-1} \circ \psi) \circ f \circ (\varphi^{-1} \circ \varphi ) \circ (\varphi')^{-1}$
@ -397,8 +397,8 @@ Differenzierbare Mannigfaltigkeiten der Klasse $C^\infty$ werden auch
\begin{definition}\label{def:8.5}\xindex{Fläche!reguläre}\xindex{Parametrisierung!reguläre}%
$S \subseteq \mdr^3$ heißt \textbf{reguläre Fläche} $:\gdw$
$\forall s \in S\;\exists $ Umgebung $V(s) \subseteq \mdr^3$ $\exists U \subseteq \mdr^2$ offen:
$\exists \text{ differenzierbare Abbildung } F: U \rightarrow V \cap S$:
$\forall s \in S\;\exists $ Umgebung $V(s) \subseteq \mdr^3$ $\exists U \subseteq \mdr^2$ offen:
$\exists \text{ differenzierbare Abbildung } F: U \rightarrow V \cap S$:
$\text{Rg}(J_F(u)) = 2\;\;\;\forall u \in U$.
$F$ heißt (lokale) \textbf{reguläre Parametrisierung} von $S$.
@ -440,7 +440,7 @@ Differenzierbare Mannigfaltigkeiten der Klasse $C^\infty$ werden auch
%\caption{}
\end{figure}
\[J_F(u,v) =
\[J_F(u,v) =
\begin{pmatrix}
-r(v) \sin u & r'(v) \cos u\\
r(v) \cos u & r'(v) \sin u\\
@ -449,7 +449,7 @@ Differenzierbare Mannigfaltigkeiten der Klasse $C^\infty$ werden auch
hat Rang 2 für alle $(u,v) \in \mdr^2$.
\item Kugelkoordinaten: $F: \mdr^2 \rightarrow \mdr^3$,\\
$(u, v) \mapsto (R \cos v \cos u, R \cos v \sin u, R \sin v)$\\
Es gilt: $F(u,v) \in S_R^2$, denn
Es gilt: $F(u,v) \in S_R^2$, denn
\begin{align*}
& R^2 \cos^2(v) \cos^2(u) + R^2 \cos^2(v) \sin^2(u) + R^2 \sin^2(v)\\
=& R^2 (\cos^2(v) \cos^2(u) + \cos^2(v) \sin^2(u) + \sin^2(v))\\
@ -459,7 +459,7 @@ Differenzierbare Mannigfaltigkeiten der Klasse $C^\infty$ werden auch
\end{align*}
Die Jacobi-Matrix
\[J_F(u,v) =
\[J_F(u,v) =
\begin{pmatrix}
-R \cos v \sin u & -R \sin v \cos u\\
R \cos v \cos u & -R \sin v \sin u\\
@ -480,10 +480,10 @@ Differenzierbare Mannigfaltigkeiten der Klasse $C^\infty$ werden auch
\begin{beweis}\leavevmode
$S \subseteq \mdr^3$ ist als reguläre Fläche eine 2-dimensionale Mannigfaltigkeit.
$S \subseteq \mdr^3$ ist als reguläre Fläche eine 2-dimensionale Mannigfaltigkeit.
Aus der Definition von regulären Flächen folgt direkt, dass Karten $(U_i, F_i)$ und
$(U_j \subseteq \mdr^2, F_j:\mdr^2 \rightarrow \mdr^3)$ von $S$ mit
$U_i \cap U_j \neq \emptyset$ existieren, wobei $F_i$ und $F_j$ nach
$U_i \cap U_j \neq \emptyset$ existieren, wobei $F_i$ und $F_j$ nach
Definition differenzierbare Abbildungen sind.
\underline{z.Z.:} $F_j^{-1} \circ F_i$ ist ein Diffeomorphismus.
@ -494,15 +494,15 @@ Differenzierbare Mannigfaltigkeiten der Klasse $C^\infty$ werden auch
\caption{Reguläre Fläche $S$ zum Beweis von \cref{kor:regular-surface-mannigfaltigkeit}}
\label{fig:parametric-surface-mapping}
\end{figure}
\underline{Idee:} Finde differenzierbare Funktion $\widetilde{F_j^{-1}}$
in Umgebung $W$ von $s$, sodass $\widetilde{F_j^{-1}}|_{S \cap W} = F_j^{-1}$.
\underline{Ausführung:} Sei $u_0 \in U_i$, $v_0 \in U_j$ mit $F_i(u_0) = s = F_j(v_0)$.
Da $\rang(J_{F_j}(v_0)) = 2$ ist, ist \obda
\[\det
Da $\rang(J_{F_j}(v_0)) = 2$ ist, ist \obda
\[\det
\begin{pmatrix}
\frac{\partial x}{\partial u} & \frac{\partial x}{\partial v}\\
\frac{\partial y}{\partial u} & \frac{\partial y}{\partial v}
@ -513,10 +513,10 @@ Differenzierbare Mannigfaltigkeiten der Klasse $C^\infty$ werden auch
Definiere $\widetilde{F_j}: U_j \times \mdr \rightarrow \mdr^3$ durch
\[\widetilde{F_j} (u, v, t) := \left(x(u,v), y(u,v), z(u,v)+t \right )\]
Offensichtlich: $\widetilde{F_j} |_{U_j \times \Set{0}} = F_j$
\[J_{\widetilde{F_j}} =
\[J_{\widetilde{F_j}} =
\begin{pmatrix}
\frac{\partial x}{\partial u} & \frac{\partial x}{\partial v} & 0\\
\frac{\partial y}{\partial u} & \frac{\partial y}{\partial v} & 0\\
@ -553,7 +553,7 @@ Differenzierbare Mannigfaltigkeiten der Klasse $C^\infty$ werden auch
\begin{beispiel}[Lie-Gruppen]
\begin{bspenum}
\item Alle endlichen Gruppen sind 0-dimensionale Lie-Gruppen.
\item $\GL_n(\mdr)$
\item $\GL_n(\mdr)$
% ist eine Lie-Gruppe, da sie nach \cref{bsp:gln-ist-mf} eine Mannigfaltigkeit ist.
% $\det: \GL_n \rightarrow \mdr$ ist eine stetige Abbildung.
\item $(\mdr^\times, \cdot)$
@ -653,10 +653,10 @@ Differenzierbare Mannigfaltigkeiten der Klasse $C^\infty$ werden auch
\begin{enumerate}[label=(\roman*),ref=\theenumii.\roman*]
\item Für $\Delta \in K$ und $S \subseteq \Delta$ Teilsimplex
ist $S \in K$.
\item \label{def:simplizialkomplex.ii} Für $\Delta_1, \Delta_2 \in K$ ist
$\Delta_1 \cap \Delta_2$ leer oder ein
Teilsimplex von $\Delta_1$ und von
$\Delta_2$.
\item \label{def:simplizialkomplex.ii} Für $\Delta_1, \Delta_2 \in K$ ist
$\Delta_1 \cap \Delta_2$ leer oder ein
Teilsimplex von $\Delta_1$ und von
$\Delta_2$.
\end{enumerate}
\item $|K| := \bigcup_{\Delta \in K} \Delta$ (mit Teilraumtopologie)
heißt \textbf{geometrische Realisierung}\xindex{Realisierung!geometrische}
@ -723,7 +723,7 @@ Differenzierbare Mannigfaltigkeiten der Klasse $C^\infty$ werden auch
\input{figures/topology-linear-mapping.tex}
\item Folgende Abbildung $\varphi: \Delta^n \rightarrow \Delta^{n-1}$
\item Folgende Abbildung $\varphi: \Delta^n \rightarrow \Delta^{n-1}$
ist simplizial:
\input{figures/topology-triangle-to-line.tex}
@ -742,7 +742,7 @@ Differenzierbare Mannigfaltigkeiten der Klasse $C^\infty$ werden auch
Sei $K$ ein endlicher Simplizialkomplex. Für $n \geq 0$ sei
$a_n(K)$ die Anzahl der $n$-Simplizes in $K$.
Dann heißt
Dann heißt
\[\chi(K) := \sum_{n=0}^{\dim K} (-1)^n a_n(K)\]
\textbf{Eulerzahl} (oder Euler-Charakteristik\index{Euler-Charakteristik|see{Eulerzahl}})
von $K$.
@ -842,7 +842,7 @@ Differenzierbare Mannigfaltigkeiten der Klasse $C^\infty$ werden auch
\begin{bemerkung}\label{kor:simplex-unterteilung}
Sei $\Delta$ ein $n$-Simplex und $x \in \Delta^\circ \subseteq \mdr^n$.
Sei $K$ der Simplizialkomplex, der aus $\Delta$ durch
Sei $K$ der Simplizialkomplex, der aus $\Delta$ durch
\enquote{Unterteilung} in $x$ entsteht. Dann ist $\chi(K) = \chi(\Delta) = 1$.
\end{bemerkung}
@ -920,7 +920,7 @@ Differenzierbare Mannigfaltigkeiten der Klasse $C^\infty$ werden auch
$\partial P$ von $0$ aus auf $\partial \fB_1(0) = S^2$.
Erhalte Triangulierung von $S^2$.
\item Sind $P_1$ und $P_2$ konvexe Polygone und $T_1, T_2$
die zugehörigen Triangulierungen von $S^2$, so gibt es
die zugehörigen Triangulierungen von $S^2$, so gibt es
eine Triangulierung $T$, die sowohl um $T_1$ als
auch um $T_2$ Verfeinerung ist (vgl. \cref{fig:topology-3}).
@ -947,7 +947,7 @@ Differenzierbare Mannigfaltigkeiten der Klasse $C^\infty$ werden auch
und $C_n(K)$ der $\mdr$-Vektorraum mit Basis $A_n(K)$, d.~h.
\[C_n(K) = \Set{\sum_{\sigma \in A_n(K)} c_\sigma \cdot \sigma | c_\sigma \in \mdr}\]
Sei $\sigma = \Delta(x_0, \dots, x_n) \in A_n(K)$, sodass
Sei $\sigma = \Delta(x_0, \dots, x_n) \in A_n(K)$, sodass
$x_0 < x_1 < \dots < x_n$.
Für $i = 0, \dots, n$ sei $\partial_i \sigma := \Delta(x_0, \dots, \hat{x_i}, \dots, x_n)$
@ -1001,14 +1001,14 @@ Differenzierbare Mannigfaltigkeiten der Klasse $C^\infty$ werden auch
\end{beweis}
\begin{definition}%
Sei $K$ ein Simplizialkomplex,
$Z_n := \text{Kern}(d_n) \subseteq C_n$ und
Sei $K$ ein Simplizialkomplex,
$Z_n := \text{Kern}(d_n) \subseteq C_n$ und
$B_n := \text{Bild}(d_{n+1}) \subseteq C_n$.
\begin{defenum}
\item $H_n = H_n(K, \mdr) := Z_n / B_n$ heißt $n$-te
\item $H_n = H_n(K, \mdr) := Z_n / B_n$ heißt $n$-te
\textbf{Homologiegruppe}\xindex{Homologiegruppe} von $K$.
\item $b_n(K) := \dim_{\mdr} H_n$ heißt $n$-te
\item $b_n(K) := \dim_{\mdr} H_n$ heißt $n$-te
\textbf{Betti-Zahl}\xindex{Betti-Zahl} von $K$.
\end{defenum}
\end{definition}

View file

@ -6,7 +6,7 @@
\section{Homotopie von Wegen}
\begin{figure}[ht]
\centering
\subfloat[$\gamma_1$ und $\gamma_2$ sind homotop, da man sie
\subfloat[$\gamma_1$ und $\gamma_2$ sind homotop, da man sie
\enquote{zueinander verschieben} kann.]{
\input{figures/topology-homotop-paths.tex}
\label{fig:homotope-wege-anschaulich}
@ -20,7 +20,7 @@
\end{figure}
\begin{definition}%
Sei $X$ ein topologischer Raum, $a, b \in X$,
Sei $X$ ein topologischer Raum, $a, b \in X$,
$\gamma_1, \gamma_2: I \rightarrow X$ Wege von $a$ nach $b$,
d.~h. $\gamma_1(0) = \gamma_2(0) = a$, $\gamma_1(1) = \gamma_2(1) = b$
@ -38,7 +38,7 @@
\end{definition}
\begin{bemerkung}
Sei $X$ ein topologischer Raum, $a, b \in X$,
Sei $X$ ein topologischer Raum, $a, b \in X$,
$\gamma_1, \gamma_2: I \rightarrow X$ Wege von $a$ nach $b$
und $H$ eine Homotopie zwischen $\gamma_1$ und $\gamma_2$.
@ -70,7 +70,7 @@
H'(t, 2s) &\text{falls } 0 \leq s \leq \frac{1}{2}\\
H''(t, 2s-1) &\text{falls } \frac{1}{2} \leq s \leq 1\end{cases}$
$\Rightarrow$ $H$ ist stetig und Homotopie von $\gamma_1$ nach
$\Rightarrow$ $H$ ist stetig und Homotopie von $\gamma_1$ nach
$\gamma_3$.
\end{itemize}
$\qed$
@ -78,12 +78,12 @@
\begin{beispiel}
\begin{bspenum}
\item Sei $X = S^1$. $\gamma_1$ und $\gamma_2$ aus
\item Sei $X = S^1$. $\gamma_1$ und $\gamma_2$ aus
\cref{fig:circle-two-paths} nicht homotop.
\item Sei $X = T^2$. $\gamma_1, \gamma_2$ und $\gamma_3$
aus \cref{fig:torus-three-paths} sind paarweise
nicht homotop.
\item Sei $X = \mdr^2$ und $a=b=(0,0)$.
\item Sei $X = \mdr^2$ und $a=b=(0,0)$.
Je zwei Wege im $\mdr^2$ mit Anfangs- und Endpunkt $(0,0)$
sind homotop.
@ -99,7 +99,7 @@
$\gamma_0(t) = (0,0) \; \forall t \in I$. Sei
$\gamma(0) = \gamma(1) = (0,0)$.
$H(t,s) := (1-s) \gamma(t)$ ist stetig,
$H(t,s) := (1-s) \gamma(t)$ ist stetig,
$H(t,0) = \gamma(t)\; \forall t \in I$ und
$H(t,1) = (0,0) \; \forall t \in I$.
\end{bspenum}
@ -123,7 +123,7 @@
% Mitschrieb vom 05.12.2013 %
%%%%%%%%%%%%%%%%%%%%%%%%%%%%%%%%%%%%%%%%%%%%%%%%%%%%%%%%%%%%%%%%%%%%%
\begin{bemerkung}\label{kor:homotope-wege}
Sei $X$ ein topologischer Raum, $\gamma: I \rightarrow X$ ein
Sei $X$ ein topologischer Raum, $\gamma: I \rightarrow X$ ein
Weg und $\varphi: I \rightarrow I$ stetig mit $\varphi(0) = 0$,
$\varphi(1) = 1$. Dann sind $\gamma$ und $\gamma \circ \varphi$
homotop.
@ -139,7 +139,7 @@
\begin{definition}\xindex{Weg!zusammengesetzter}%
Seien $\gamma_1, \gamma_2$ Wege in $X$ mit $\gamma_1(1) = \gamma_2(0)$.
Dann ist
Dann ist
\[\gamma (t) = \begin{cases}
\gamma_1(2t) &\text{falls } 0 \leq t < \frac{1}{2}\\
\gamma_2(2t-1) &\text{falls } \frac{1}{2} \leq t \leq 1
@ -149,7 +149,7 @@
\end{definition}
\begin{bemerkung}\label{kor:assoziativitaet-von-zusammensetzen-von-wegen}
Das Zusammensetzen von Wegen ist nur bis auf
Das Zusammensetzen von Wegen ist nur bis auf
Homotopie assoziativ, d.~h.:
\begin{align*}
\gamma_1 * (\gamma_2 * \gamma_3) &\neq (\gamma_1 * \gamma_2) * \gamma_3\\
@ -203,13 +203,13 @@
Sei $H_i$ eine Homotopie zwischen $\gamma_i$ und $\gamma_i'$,
$i=1,2$.
Dann ist
Dann ist
\[H(t,s) := \begin{cases}
H_1(2t, s) &\text{falls } 0 \leq t \leq \frac{1}{2}\;\;\;\forall s \in I\\
H_2(2t-1,s) &\text{falls } \frac{1}{2} \leq t \leq 1
\end{cases}\]
eine Homotopie zwischen
eine Homotopie zwischen
$\gamma_1 * \gamma_2$ und $\gamma_1' * \gamma_2 '$.
\end{beweis}
@ -219,7 +219,7 @@ Eine spezielle Homotopieäquivalenz sind sog. Deformationsretraktionen:
und $\iota = (\id_X)|_A$.
\begin{defenum}
\item $\iota: A \rightarrow X$ mit $\iota(x) = x$ heißt die
\item $\iota: A \rightarrow X$ mit $\iota(x) = x$ heißt die
\textbf{Inklusionsabbildung}\xindex{Inklusionsabbildung} und
man schreibt: $\iota: A \hookrightarrow X$.
\item $r$ heißt \textbf{Retraktion}\xindex{Retraktion}, wenn $r|_A = \id_A$ ist.
@ -264,7 +264,7 @@ Für einen Weg $\gamma$ sei $[\gamma]$ seine \textbf{Homotopieklasse}\xindex{Hom
\item Assoziativität folgt aus \cref{kor:assoziativitaet-von-zusammensetzen-von-wegen}
\item Neutrales Element $e = [\gamma_0], \gamma_0(t) = x \;\;\; \forall t \in I$.
$e * [\gamma] = [\gamma] = [\gamma] * e$, da $\gamma_0 * \gamma \sim \gamma$
\item \xindex{Weg!inverser} Inverses Element $[\gamma]^{-1} = [\overline{\gamma}] = [\gamma(1-t)]$,
\item \xindex{Weg!inverser} Inverses Element $[\gamma]^{-1} = [\overline{\gamma}] = [\gamma(1-t)]$,
denn $\overline{\gamma} * \gamma \sim \gamma_0 \sim \gamma * \overline{\gamma}$
\end{enumerate}
\end{beweis}
@ -280,7 +280,7 @@ Für einen Weg $\gamma$ sei $[\gamma]$ seine \textbf{Homotopieklasse}\xindex{Hom
$[\gamma^k] \mapsto k$ ist ein Isomorphismus.
\item $\pi_1 (\mdr^2, 0) = \pi_1 (\mdr^2, x) = \Set{e}$ für jedes $x \in \mdr^2$
\item $\pi_1 (\mdr^n, x) = \Set{e}$ für jedes $x \in \mdr^n$
\item $G \subseteq \mdr^n$ heißt \textbf{sternförmig}\xindex{sternförmig} bzgl. $x \in G$,
\item $G \subseteq \mdr^n$ heißt \textbf{sternförmig}\xindex{sternförmig} bzgl. $x \in G$,
wenn für jedes $y \in G$ auch die Strecke $[x, y] \subseteq G$
ist.
@ -338,7 +338,7 @@ Für einen Weg $\gamma$ sei $[\gamma]$ seine \textbf{Homotopieklasse}\xindex{Hom
für ein $x \in X$.
\end{definition}
Wenn $\pi_1(X,x) = \Set{e}$ für ein $x \in X$ gilt, dann wegen
Wenn $\pi_1(X,x) = \Set{e}$ für ein $x \in X$ gilt, dann wegen
\cref{kor:gruppenisomorphismus-wege} sogar für alle $x \in X$.
\begin{bemerkung}\label{korr:11.5}
@ -359,7 +359,7 @@ Wenn $\pi_1(X,x) = \Set{e}$ für ein $x \in X$ gilt, dann wegen
\item $f_*$ ist wohldefiniert: Seien $\gamma_1, \gamma_2$ homotope
Wege von $x$. z.Z.: $f \circ \gamma_1 \sim f \circ \gamma_2$:
Nach Voraussetzung gibt es stetige Abbildungen $H:I\times I \rightarrow X$
mit
mit
\begin{align*}
H(t,0) &= \gamma_1(t),\\
H(t,1) &= \gamma_2(t),\\
@ -376,7 +376,7 @@ Wenn $\pi_1(X,x) = \Set{e}$ für ein $x \in X$ gilt, dann wegen
\begin{beispiel}
\begin{bspenum}
\item $f:S^1 \hookrightarrow \mdr^2$ ist injektiv, aber
\item $f:S^1 \hookrightarrow \mdr^2$ ist injektiv, aber
$f_*:\pi_1(S^1, 1) \cong \mdz \rightarrow \pi_1(\mdr^2, 1) = \Set{e}$
ist nicht injektiv.
\item $f: \mdr \rightarrow S^1, t \mapsto (\cos 2 \pi t, \sin 2 \pi t)$
@ -406,7 +406,7 @@ Wenn $\pi_1(X,x) = \Set{e}$ für ein $x \in X$ gilt, dann wegen
stetig mit $f(x_0) = y_0 = g(x_0)$.
$f$ und $g$ heißen \textbf{homotop} ($f \sim g$), wenn es eine stetige
Abbildung $H: X \times I \rightarrow Y$ mit
Abbildung $H: X \times I \rightarrow Y$ mit
\begin{align*}
H(x,0) &= f(x) \; \forall x \in X\\
H(x,1) &= g(x) \; \forall x \in X\\
@ -426,7 +426,7 @@ Wenn $\pi_1(X,x) = \Set{e}$ für ein $x \in X$ gilt, dann wegen
Z.~z.: $f \circ \gamma \sim g \circ \gamma$
Sei dazu $H_\gamma: I \times I \rightarrow Y, (t,s) \mapsto H(\gamma(t), s)$.
Dann gilt:
Dann gilt:
\begin{align*}
H_\gamma(t,0) &= H(\gamma(t), 0) = (f \circ \gamma)(t) \;\forall t \in I\\
H_\gamma(1,s) &= H(\gamma(1), s) = H(x_0, s) = y_0\;\forall s \in I\\
@ -450,7 +450,7 @@ Wenn $\pi_1(X,x) = \Set{e}$ für ein $x \in X$ gilt, dann wegen
\end{beispiel}
\begin{satz}[Satz von Seifert und van Kampen \enquote{light}]\label{thm:seifert-van-kampen}
Sei $X$ ein topologischer Raum, $U, V \subseteq X$ offen mit
Sei $X$ ein topologischer Raum, $U, V \subseteq X$ offen mit
$U \cup V = X$ und $U \cap V$ wegzusammenhängend.
Dann wird $\pi_1(X,x)$ für $x \in U \cap V$ erzeugt von geschlossenen
@ -460,14 +460,14 @@ Wenn $\pi_1(X,x) = \Set{e}$ für ein $x \in X$ gilt, dann wegen
\begin{beweis}
Sei $\gamma: I \rightarrow X$ ein geschlossener Weg um $x$.
Überdecke $I$ mit endlich vielen offenen Intervallen
$I_1, I_2, \dots, I_n$, die ganz in
$I_1, I_2, \dots, I_n$, die ganz in
$\gamma^{-1}(U)$ oder ganz in $\gamma^{-1}(V)$ liegen.
\Obda sei $\gamma(I_1) \subseteq U, \gamma(I_2) \subseteq V$, etc.
Wähle $t_i \in I_i \cap I_{i+1}$, also $\gamma(t_i) \in U \cap V$.
Sei $\sigma_i$ Weg in $U \cap V$ von $x_0$ nach $\gamma(t_i) \Rightarrow \gamma$
ist homotop zu
ist homotop zu
\[\underbrace{\gamma_1 * \overline{\sigma_1}}_{\text{in } U} * \underbrace{\sigma_1 * \gamma_2 * \overline{\sigma_2}}_{\text{in } V} * \dots * \sigma_{n-1} * \gamma_2 \text{ mit } \gamma_i := \gamma |_{I_i}\]
\end{beweis}
@ -547,17 +547,17 @@ Wenn $\pi_1(X,x) = \Set{e}$ für ein $x \in X$ gilt, dann wegen
\begin{beweis}
Sei $p: Y \rightarrow X$ eine Überlagerung und $x \in X$ beliebig.
Dann existiert eine offene Umgebung $U(x) \subseteq X$ und offene
Teilmengen $V_j \subseteq X$ mit
Teilmengen $V_j \subseteq X$ mit
$p^{-1}(U) = \Dcup V_j$ und
$p|_{V_j}: V_j \rightarrow U$ ist Homöomorphismus.
D.~h. es existiert ein $y \in V_j$, so dass $p|_{V_j}(y) = x$.
Da $x \in X$ beliebig war und ein $y \in Y$ existiert, mit
Da $x \in X$ beliebig war und ein $y \in Y$ existiert, mit
$p(y) = x$, ist $p$ surjektiv. $\qed$
\end{beweis}
\begin{definition}\xindex{Abbildung!offene}%
Seien $(X, \fT_X), (Y, \fT_Y)$ topologische Räume und $f:X \rightarrow Y$ eine
Seien $(X, \fT_X), (Y, \fT_Y)$ topologische Räume und $f:X \rightarrow Y$ eine
Abbildung.
$f$ heißt \textbf{offen} $:\gdw \forall U \in \fT_X: f(U) \in \fT_Y$.
@ -600,7 +600,7 @@ Wenn $\pi_1(X,x) = \Set{e}$ für ein $x \in X$ gilt, dann wegen
\begin{definition}\xindex{diskret}%
Sei $X$ ein topologischer Raum und $M \subseteq X$.
$M$ heißt \textbf{diskret} in $X$, wenn $M$ in $X$ keinen
$M$ heißt \textbf{diskret} in $X$, wenn $M$ in $X$ keinen
Häufungspunkt hat.
\end{definition}
@ -628,11 +628,11 @@ Wenn $\pi_1(X,x) = \Set{e}$ für ein $x \in X$ gilt, dann wegen
$\Rightarrow V_{j_1} \cap V_{j_2} = \emptyset$ nach Voraussetzung.
\underline{2. Fall}: $p(y_1) \neq p(y_2)$.
Dann seien $U_1$ und $U_2$ disjunkte Umgebungen von $p(y_1)$
und $p(y_2)$.
$\Rightarrow p^{-1}(U_1)$ und $p^{-1}(U_2)$ sind disjunkte
$\Rightarrow p^{-1}(U_1)$ und $p^{-1}(U_2)$ sind disjunkte
Umgebungen von $y_1$ und $y_2$.
\item Sei $x \in X$ beliebig, aber fest.
@ -717,7 +717,7 @@ Wenn $\pi_1(X,x) = \Set{e}$ für ein $x \in X$ gilt, dann wegen
Sei $q:U \rightarrow V$ die Umkehrabbildung zu $p|_V$.
Sei $W:= f^{-1}(U) \cap f_0^{-1}(V) \cap f_1^{-1}(V)$. $W$ ist
Sei $W:= f^{-1}(U) \cap f_0^{-1}(V) \cap f_1^{-1}(V)$. $W$ ist
offene Umgebung in $Z$ von $z$.
\underline{Behauptung:} $W \subseteq T$
@ -749,7 +749,7 @@ $p|_{V_j}: V_j \rightarrow U$ Homöomorphismus.
\begin{bemerkung}%Bemerkung 12.6 der Vorlesung
Wege in $X$ lassen sich zu Wegen in $Y$ liften.
Zu jedem $y \in p^{-1}(\gamma(0))$ gibt es genau einen Lift von
Zu jedem $y \in p^{-1}(\gamma(0))$ gibt es genau einen Lift von
$\gamma$.
\end{bemerkung}
@ -757,7 +757,7 @@ $p|_{V_j}: V_j \rightarrow U$ Homöomorphismus.
Seien $p: Y \rightarrow X$ eine Überlagerung, $a,b \in X$,
$\gamma_0, \gamma_1: I \rightarrow X$ homotope Wege von $a$ nach
$b$, $\tilde{a} \in p^{-1}(a), \tilde{\gamma_0}, \tilde{\gamma_1}$
Liftungen von $\gamma_0$ bzw. $\gamma_1$ mit
Liftungen von $\gamma_0$ bzw. $\gamma_1$ mit
$\tilde{\gamma_i}(0) = \tilde{a}$.
Dann ist $\tilde{\gamma_0}(1) = \tilde{\gamma_1}(1)$ und
@ -784,7 +784,7 @@ $p|_{V_j}: V_j \rightarrow U$ Homöomorphismus.
Da $p^{-1}(b)$ diskrete Teilmenge von $Y$ ist\\
$\Rightarrow \tilde{b_s} = \tilde{H}(1,s) = \tilde{H}(1,0) \;\forall s \in I$\\
$\Rightarrow \tilde{b_0} = \tilde{b_1}$ und $\tilde{H}$ ist Homotopie
$\Rightarrow \tilde{b_0} = \tilde{b_1}$ und $\tilde{H}$ ist Homotopie
zwischen $\tilde{\gamma_0}$ und $\tilde{\gamma_1}$. $\qed$
\end{beweis}
@ -801,7 +801,7 @@ $p|_{V_j}: V_j \rightarrow U$ Homöomorphismus.
\item Sei $\tilde{\gamma}$ ein Weg in $Y$ um $y_0$ und
$p_* ([\tilde{\gamma}]) = e$, also $p \circ \tilde{\gamma} \sim \gamma_{x_0}$
Nach \cref{proposition:12.7} ist dann
Nach \cref{proposition:12.7} ist dann
$\tilde{\gamma}$ homotop zum Lift des konstanten Wegs
$\gamma_{x_0}$ mit Anfangspunkt $y_0$, also zu
$\gamma_{y_0} \Rightarrow [\tilde{\gamma}] = e$
@ -823,9 +823,9 @@ $p|_{V_j}: V_j \rightarrow U$ Homöomorphismus.
Zu $i \in \Set{0, \dots, d-1}$ gibt es Weg $\delta_i$ in
$Y$ mit $\delta_i(0) = y_0$ und $\delta_i(1) = y_i$\\
$\Rightarrow p \cup \delta_i$ ist geschlossener Weg in
$\Rightarrow p \cup \delta_i$ ist geschlossener Weg in
$X$ um $x_0$.\\
$\Rightarrow$ Jedes $y_i$ mit $i=0, \dots, d-1$ ist
$\Rightarrow$ Jedes $y_i$ mit $i=0, \dots, d-1$ ist
$\tilde{\gamma}(1)$ für ein $[\gamma] \in \pi_1(X,x_0)$.
\end{enumerate}
\end{beweis}
@ -885,7 +885,7 @@ $p|_{V_j}: V_j \rightarrow U$ Homöomorphismus.
Offensichtlich ist $q(\tilde{p}(z)) = p(z)$.
\underline{Zu zeigen:} $\tilde{p}$ ist stetig in $z \in \tilde{X}$:
Sei $W \subseteq Y$ offene Umgebung von $\tilde{p}(z)$.
$\xRightarrow{q \text{ offen}} q(W)$ ist offene Umgebung von $p(z) \cdot d(\tilde{p}(z))$.
@ -915,8 +915,8 @@ $p|_{V_j}: V_j \rightarrow U$ Homöomorphismus.
\end{folgerung}
\begin{beweis}
Seien $x_0 \in X, \tilde{x_0} \in \tilde{X}$ mit
$p(\tilde{x_0}) = x_0$ und
Seien $x_0 \in X, \tilde{x_0} \in \tilde{X}$ mit
$p(\tilde{x_0}) = x_0$ und
$\tilde{y_0} \in q^{-1}(x_0) \subseteq \tilde{Y}$.
Nach \cref{thm:12.11} gibt es genau eine Überlagerung
@ -925,7 +925,7 @@ $p|_{V_j}: V_j \rightarrow U$ Homöomorphismus.
\[g: \tilde{Y} \rightarrow \tilde{X} \text{ mit } g(\tilde{y_0}) = \tilde{x_0} \text{ und } p \circ g = q\]
Damit gilt: $p \circ q \circ f = q \circ f = p$, $q \circ f \circ g = p \circ g = q$.
Also ist $g \circ f: \tilde{X} \rightarrow \tilde{X}$ Lift von
Also ist $g \circ f: \tilde{X} \rightarrow \tilde{X}$ Lift von
$p:\tilde{X} \rightarrow X$ mit $(g \circ f) (\tilde{x_0}) = \tilde{x_0}$.
Da auch $\id_{\tilde{x}}$ diese Eigenschaft hat, folgt mit
@ -961,7 +961,7 @@ der folgende Satz:
\[\tilde{U} = \tilde{U}(x, [\gamma]) := \Set{(y, [\gamma * \alpha]) | y \in U, \alpha \text{ Weg in } U \text{ von } x \text{ nach } y} \]
$p$ ist Überlagerung: $p|_{\tilde{U}} : \tilde{U} \rightarrow U$
bijektiv. $p$ ist stetig und damit $p|_{\tilde{U}}$ ein
bijektiv. $p$ ist stetig und damit $p|_{\tilde{U}}$ ein
Homöomorphismus.
Sind $\gamma_1, \gamma_2$ Wege von $x_0$ nach $x$ und $\gamma_1 \sim \gamma_2$,
@ -990,7 +990,7 @@ der folgende Satz:
\begin{defenum}
\item $f$ heißt \textbf{Decktransformation} von $p :\gdw p \circ f = p$.
\item Die Decktransformationen von $p: Y \rightarrow X$ bilden mit der Verkettung eine Gruppe,
\item Die Decktransformationen von $p: Y \rightarrow X$ bilden mit der Verkettung eine Gruppe,
die sog. \textbf{Decktransformationsgruppe}\xindex{Decktransformationsgruppe}.
Man schreibt:
$\Deck(p)$, $\Deck(Y/X)$ oder $\Deck(Y \rightarrow X)$.
@ -1023,10 +1023,10 @@ der folgende Satz:
\end{itemize}
\item Die Menge
\[\Fix(f) = \Set{y \in Y | f(y) = y}\]
ist abgeschlossen als Urbild der Diagonale
ist abgeschlossen als Urbild der Diagonale
$\Delta \subseteq Y \times Y$ unter der stetigen
Abbildung $y \mapsto (f(y),y)$. Außerdem ist $\Fix(f)$
offen, denn ist $y \in \Fix(f)$, so sei $U$ eine
offen, denn ist $y \in \Fix(f)$, so sei $U$ eine
Umgebung von $p(y) \in X$ wie in \cref{def:12.1}
und $U \subseteq p^{-1}(U)$ die Komponente, die $y$
enthält; also $p:V \rightarrow U$ ein Homöomorphismus.
@ -1041,7 +1041,7 @@ der folgende Satz:
\item Es sei $x_0 \in X$, $\deg(p) = d$ und $p^{-1}(x_0) = \Set{y_0, \dots, y_{d-1}}$.
Für $f \in \Deck(Y/X)$ ist $f(y_0)= \Set{y_0, \dots, y_{d-1}}$.
Zu $i \in \Set{0, \dots, d-1}$ gibt es höchstens ein
Zu $i \in \Set{0, \dots, d-1}$ gibt es höchstens ein
$f \in \Deck(Y/X)$ mit $f(y_0) = y_1$, denn ist
$f(y_0) = g(y_0)$, so ist $(g^{-1} \circ f)(y_0) = y_0$,
also nach \cref{kor:12.14c} $g^{-1} \circ f = \id_Y$.
@ -1061,7 +1061,7 @@ Nun werden wir eine Verbindung zwischen der Decktransformationsgruppe
und der Fundamentalgruppe herstellen:
\begin{satz}\label{thm:12.15}%In Vorlesung: Satz 12.15
Ist $p: \tilde{X} \rightarrow X$ eine universelle Überlagerung,
Ist $p: \tilde{X} \rightarrow X$ eine universelle Überlagerung,
so gilt:
\[\Deck(\tilde{X}/X) \cong \pi_1(X, x_0)\;\;\;\forall x_0 \in X\]
\end{satz}
@ -1074,11 +1074,11 @@ und der Fundamentalgruppe herstellen:
eindeutig bestimmt und damit auch $\rho$ wohldefiniert.
\begin{itemize}
\item \underline{$\rho$ ist Gruppenhomomorphismus}: Seien
\item \underline{$\rho$ ist Gruppenhomomorphismus}: Seien
$f, g \in \Deck(\tilde{X}/ X) \Rightarrow \gamma_{g \circ f} = \gamma_g * g(\gamma_f)$
$\Rightarrow p(\gamma_{g \circ f}) = p(\gamma_g) * \underbrace{(p \circ g)}_{=p} (\gamma_f) = \rho(g) \neq \rho(f)$
\item \underline{$\rho$ ist injektiv}: $\rho(f) = e \Rightarrow p (\gamma_f) \sim \gamma_{x_0}$
$\xRightarrow{\cref{thm:ueberlagerung-weg-satz-12.6}} \gamma_f \sim \gamma_{\tilde{x_0}}$
$\xRightarrow{\cref{thm:ueberlagerung-weg-satz-12.6}} \gamma_f \sim \gamma_{\tilde{x_0}}$
$\Rightarrow f(x_0) = \tilde{x_0} \xRightarrow{\crefabbr{kor:12.14c}} f = \id_{\tilde{x}}$.
\item \underline{$\rho$ ist surjektiv}: Sei $[\gamma] \in \pi_1(X, x_0)$,
$\tilde{\gamma}$ Lift von $\gamma$ nach $\tilde{x}$ mit
@ -1086,7 +1086,7 @@ und der Fundamentalgruppe herstellen:
sei $\tilde{x_1}$.
\underline{$p$ ist reguläre Überlagerung}: Seien
$\tilde{x_0}, \tilde{x_1} \in \tilde{X}$ mit
$\tilde{x_0}, \tilde{x_1} \in \tilde{X}$ mit
$p(\tilde{x_0}) = p(\tilde{x_1})$. Nach \cref{thm:12.11}
gibt es genau eine Überlagerung $\tilde{p}: \tilde{X} \rightarrow X$
mit $p=p \circ \tilde{p}$ und $\tilde{p}(\tilde{x_0}) = \tilde{x_1}$.
@ -1163,7 +1163,7 @@ und der Fundamentalgruppe herstellen:
\[m_g: X \rightarrow X, x \mapsto g \circ x\]
ein Homöomorphismus ist.
\item Ist $G$ eine topologische Gruppe, so heißt die Gruppenoperation $\circ$
\textbf{stetig}\xindex{Gruppenoperation!stetige}, wenn
\textbf{stetig}\xindex{Gruppenoperation!stetige}, wenn
\[\forall g \in G: m_g \text{ ist stetig}\]
gilt.
\end{defenum}
@ -1175,7 +1175,7 @@ und der Fundamentalgruppe herstellen:
\begin{beweis}\leavevmode
Nach Voraussetzung ist $m_g := \circ |_{\Set{g} \times X} : X \rightarrow X, x \mapsto g \circ x$ stetig.
Die Umkehrabbildung zu $m_g$ ist $m_{g^{-1}}$:
Die Umkehrabbildung zu $m_g$ ist $m_{g^{-1}}$:
\begin{align*}
(m_{g^{-1}} \circ m_g)(x) &= m_{g^{-1}} (m_g (x))\\
&= m_{g^{-1}} (g \circ x)\\
@ -1196,7 +1196,7 @@ und der Fundamentalgruppe herstellen:
\begin{bemenum}
\item Die Gruppenoperation von $G$ auf $X$ entsprechen bijektiv
den Gruppenhomomorphismen $\varrho: G \rightarrow \Perm(X) = \Sym(X) = \Set{f: X \rightarrow X | f \text{ ist bijektiv}}$
\item Ist $X$ ein topologischer Raum, so entsprechen dabei
\item Ist $X$ ein topologischer Raum, so entsprechen dabei
die Gruppenoperationen durch Homöomorphismus den Gruppenhomomorphismen
$G \rightarrow \Homoo(X)$
\end{bemenum}
@ -1213,7 +1213,7 @@ und der Fundamentalgruppe herstellen:
Umgekehrt: Sei $\varrho: G \rightarrow \Perm(X)$ Gruppenhomomorphismus. Definiere $\circ: G \times X \rightarrow X$ durch $g \circ x = \varrho (g)(x)$.
z.~z. \cref{def:gruppenoperation.2}:
z.~z. \cref{def:gruppenoperation.2}:
\begin{align*}
g_1 \circ (g_2 \circ x) &= \varrho (g_1) (g_2 \circ x)\\
&= \varrho(g_1) (\varrho(g_2)(x))\\
@ -1222,8 +1222,8 @@ und der Fundamentalgruppe herstellen:
&= (g_1 \cdot g_2) \circ x
\end{align*}
z.~z. \cref{def:gruppenoperation.1}:
$1_G \cdot x = \varrho(1_G)(x) = \id_X(x) = x$, weil $\varrho$ ein
z.~z. \cref{def:gruppenoperation.1}:
$1_G \cdot x = \varrho(1_G)(x) = \id_X(x) = x$, weil $\varrho$ ein
Homomorphismus ist.
\end{beweis}
@ -1253,7 +1253,7 @@ und der Fundamentalgruppe herstellen:
\begin{enumerate}[label=\roman*)]
\item $[e] \circ \tilde{x} = \rtilde{e * \delta} = \tilde{x}$
\item $\rtilde{\gamma_1 * \gamma_2 * \delta}(1) = [\gamma_1 * \gamma_2] \circ \tilde{x} = ([\gamma_1] * [\gamma_2]) \circ \tilde{x}$\\
$\gamma_1 * \gamma_2 * \delta(1) = [\gamma_1] \circ (\tilde{\gamma_2 * \delta})(1) = [\gamma_1] \circ ([\gamma_2] \circ \tilde{x})$
$\gamma_1 * \gamma_2 * \delta(1) = [\gamma_1] \circ (\tilde{\gamma_2 * \delta})(1) = [\gamma_1] \circ ([\gamma_2] \circ \tilde{x})$
\end{enumerate}
\end{beispiel}

View file

@ -18,7 +18,7 @@
\end{aufgabe}
\begin{aufgabe}\label{ub11:aufg3}
Sei $(X, d)$ eine absolute Ebene. Der \textit{Abstand}\xindex{Abstand} eines
Sei $(X, d)$ eine absolute Ebene. Der \textit{Abstand}\xindex{Abstand} eines
Punktes $P$ zu einer Menge $Y \subseteq X$ von Punkten ist
definiert durch $d(P, Y) := \inf{d(P, y) | y \in Y}$.
@ -27,13 +27,13 @@
\item \label{ub11:aufg3.a} Ist $\triangle ABC$ ein Dreieck, in dem die Seiten
$\overline{AB}$ und $\overline{AC}$ kongruent sind, so
sind die Winkel $\angle ABC$ und $\angle BCA$ gleich.
\item \label{ub11:aufg3.b} Ist $\triangle ABC$ ein beliebiges Dreieck, so liegt
\item \label{ub11:aufg3.b} Ist $\triangle ABC$ ein beliebiges Dreieck, so liegt
der längeren Seite der größere Winkel gegenüber und
umgekehrt.
\item \label{ub11:aufg3.c} Sind $g$ eine Gerade und $P \notin g$ ein Punkt, so gibt
es eine eindeutige Gerade $h$ mit $P \in h$ und die
$g$ im rechten Winkel schneidet. Diese Grade heißt
\textit{Lot}\xindex{Lot} von $P$ auf $g$ und der
$g$ im rechten Winkel schneidet. Diese Grade heißt
\textit{Lot}\xindex{Lot} von $P$ auf $g$ und der
Schnittpunkt des Lots mit $g$ heißt \textit{Lotfußpunkt}\xindex{Lotfußpunkt}.
\end{aufgabeenum}
\end{aufgabe}

View file

@ -12,25 +12,25 @@
\section{Axiome für die euklidische Ebene}
Axiome\xindex{Axiom} bilden die Grundbausteine jeder mathematischen Theorie. Eine
Sammlung aus Axiomen nennt man Axiomensystem\xindex{Axiomensystem}.
Da der Begriff des Axiomensystems so grundlegend ist, hat man auch
Da der Begriff des Axiomensystems so grundlegend ist, hat man auch
ein paar sehr grundlegende Forderungen an ihn: Axiomensysteme sollen
\textbf{widerspruchsfrei} sein, die Axiome sollen möglichst
\textbf{unabhängig} sein und \textbf{Vollständigkeit} wäre auch toll.
Mit Unabhängigkeit ist gemeint, dass kein Axiom sich aus einem anderem
herleiten lässt. Dies scheint auf den ersten Blick eine einfache
Eigenschaft zu sein. Auf den zweiten Blick muss man jedoch einsehen,
dass das Parallelenproblem, also die Frage ob das Parallelenaxiom
unabhängig von den restlichen Axiomen ist, über 2000 Jahre nicht
Eigenschaft zu sein. Auf den zweiten Blick muss man jedoch einsehen,
dass das Parallelenproblem, also die Frage ob das Parallelenaxiom
unabhängig von den restlichen Axiomen ist, über 2000 Jahre nicht
gelöst wurde. Ein ganz anderes Kaliber ist die Frage nach der
Vollständigkeit. Ein Axiomensystem gilt als Vollständig, wenn
jede Aussage innerhalb des Systems verifizierbar oder falsifizierbar
ist. Interessant ist hierbei der Gödelsche Unvollständigkeitssatz,
ist. Interessant ist hierbei der Gödelsche Unvollständigkeitssatz,
der z.~B. für die Arithmetik beweist, dass nicht alle Aussagen
formal bewiesen oder widerlegt werden können.
Kehren wir nun jedoch zurück zur Geometrie. Euklid hat in seiner
Kehren wir nun jedoch zurück zur Geometrie. Euklid hat in seiner
Abhandlung \enquote{Die Elemente} ein Axiomensystem für die Geometrie
aufgestellt.
aufgestellt.
\textbf{Euklids Axiome}
\begin{itemize}
@ -39,11 +39,11 @@ aufgestellt.
\item \textbf{Kreis} (um jeden Punkt mit jedem Radius)
\item Je zwei rechte Winkel sind gleich (Isometrie, Bewegung)
\item Parallelenaxiom von Euklid:\xindex{Parallelenaxiom}\\
Wird eine Gerade so von zwei Geraden geschnitten, dass die
Wird eine Gerade so von zwei Geraden geschnitten, dass die
Summe der Innenwinkel kleiner als zwei Rechte ist, dann schneiden sich
diese Geraden auf der Seite dieser Winkel.\\
\\
Man mache sich klar, dass das nur dann nicht der Fall ist,
Man mache sich klar, dass das nur dann nicht der Fall ist,
wenn beide Geraden parallel sind und senkrecht auf die erste stehen.
\end{itemize}
@ -60,7 +60,7 @@ aufgestellt.
\end{enumerate}
\item \textbf{Abstandsaxiom}\xindex{Abstandsaxiom}: Zu $P, Q, R \in X$ gibt es \label{axiom:2}
genau dann ein $g \in G$ mit $\Set{P, Q, R} \subseteq g$,
wenn gilt:
wenn gilt:
\begin{itemize}[]
\item $d(P, R) = d(P, Q) + d(Q, R)$ oder
\item $d(P, Q) = d(P, R) + d(R, Q)$ oder
@ -72,7 +72,7 @@ aufgestellt.
\begin{definition}
Sei $(X, d, G)$ eine Geometrie und seien $P, Q, R \in X$.
\begin{defenum}
\item $P, Q, R$ liegen \textbf{kollinear}\xindex{kollinear},
\item $P, Q, R$ liegen \textbf{kollinear}\xindex{kollinear},
wenn es $g \in G$ gibt mit $\Set{P, Q, R} \subseteq g$.
\item $Q$ \textbf{liegt zwischen}\xindex{liegt zwischen} $P$
und $R$, wenn $d(P, R) = d(P, Q) + d(Q, R)$
@ -103,10 +103,10 @@ aufgestellt.
\begin{beweis}\leavevmode
\begin{enumerate}[label=\alph*)]
\item \enquote{$\subseteq$} folgt direkt aus der Definition von $PR^+$ und $PR^-$\\
\enquote{$\supseteq$}: Sei $Q \in PR \Rightarrow P, Q, R$
\enquote{$\supseteq$}: Sei $Q \in PR \Rightarrow P, Q, R$
sind kollinear.\\
$\overset{\ref{axiom:2}}{\Rightarrow}
\begin{cases}
\begin{cases}
Q \text{ liegt zwischen } P \text{ und } R \Rightarrow Q \in PR\\
R \text{ liegt zwischen } P \text{ und } Q \Rightarrow Q \in PR\\
P \text{ liegt zwischen } Q \text{ und } R \Rightarrow Q \in PR
@ -133,21 +133,21 @@ aufgestellt.
\begin{enumerate}[label=§\arabic*),ref=§\arabic*,start=3]
\item \label{axiom:3}\textbf{Anordnungsaxiome}\xindex{Anordnungsaxiome}
\begin{enumerate}[label=(\roman*),ref=\theenumi{} (\roman*)]
\item \label{axiom:3.1} Zu jeder
Halbgerade $H$ mit Anfangspunkt $P \in X$ und jedem
$r \in \mdr_{\geq 0}$ gibt es genau ein
\item \label{axiom:3.1} Zu jeder
Halbgerade $H$ mit Anfangspunkt $P \in X$ und jedem
$r \in \mdr_{\geq 0}$ gibt es genau ein
$Q \in H$ mit $d(P,Q) = r$.
\item \label{axiom:3.2} Jede Gerade zerlegt
$X \setminus g = H_1 \dcup H_2$ in zwei
\item \label{axiom:3.2} Jede Gerade zerlegt
$X \setminus g = H_1 \dcup H_2$ in zwei
nichtleere Teilmengen $H_1, H_2$,
sodass für alle $A \in H_i$, $B \in H_j$ mit
$i,j \in \Set{1,2}$ gilt:
$i,j \in \Set{1,2}$ gilt:
$\overline{AB} \cap g \neq \emptyset \Leftrightarrow i \neq j$.\\
Diese Teilmengen $H_i$ heißen
\textbf{Halbebenen}\xindex{Halbebene} bzgl.
Diese Teilmengen $H_i$ heißen
\textbf{Halbebenen}\xindex{Halbebene} bzgl.
$g$.
\end{enumerate}
\item \label{axiom:4}\textbf{Bewegungsaxiom}\xindex{Bewegungsaxiom}:
\item \label{axiom:4}\textbf{Bewegungsaxiom}\xindex{Bewegungsaxiom}:
Zu $P, Q, P', Q' \in X$
mit $d(P,Q) = d(P', Q')$ gibt es mindestens 2 Isometrien $\varphi_1, \varphi_2$
mit $\varphi_i (P) = P'$ und $\varphi_i(Q) = Q'$ mit $i=1,2$.\footnote{Die \enquote{Verschiebung} von $P'Q'$ nach $PQ$ und die Isometrie, die zusätzlich an der Gerade durch $P$ und $Q$ spiegelt.}
@ -163,14 +163,14 @@ aufgestellt.
%%%%%%%%%%%%%%%%%%%%%%%%%%%%%%%%%%%%%%%%%%%%%%%%%%%%%%%%%%%%%%%%%%%%%
\begin{satz}[Satz von Pasch]\label{satz:pasch} %In Vorlesung: Bemerkung 14.5
Seien $P$, $Q$, $R$ nicht kollinear, $g \in G$ mit $g \cap \Set{P, Q, R} = \emptyset$
und $g \cap \overline{PQ} \neq \emptyset$.
und $g \cap \overline{PQ} \neq \emptyset$.
Dann ist entweder $g \cap \overline{PR} \neq \emptyset$ oder
Dann ist entweder $g \cap \overline{PR} \neq \emptyset$ oder
$g \cap \overline{QR} \neq \emptyset$.
\end{satz}
Dieser Satz besagt, dass Geraden, die eine Seite eines Dreiecks
(also nicht nur eine Ecke) schneiden, auch eine weitere Seite
Dieser Satz besagt, dass Geraden, die eine Seite eines Dreiecks
(also nicht nur eine Ecke) schneiden, auch eine weitere Seite
schneiden.
\begin{beweis}
@ -182,7 +182,7 @@ schneiden.
\end{beweis}
\begin{bemerkung}\label{kor:beh3}
Sei $P, Q \in X$ mit $P \neq Q$ sowie $A, B \in X \setminus PQ$
Sei $P, Q \in X$ mit $P \neq Q$ sowie $A, B \in X \setminus PQ$
mit $A \neq B$.
Außerdem seien $A$ und $B$ in der selben Halbebene bzgl. $PQ$ sowie
$Q$ und $B$ in der selben Halbebene bzgl. $PA$.
@ -197,8 +197,8 @@ schneiden.
\label{fig:geometry-5}
\end{figure}
Auch \cref{kor:beh3} lässt sich umgangssprachlich sehr viel
einfacher ausdrücken: Die Diagonalen eines konvexen Vierecks
Auch \cref{kor:beh3} lässt sich umgangssprachlich sehr viel
einfacher ausdrücken: Die Diagonalen eines konvexen Vierecks
schneiden sich.
\begin{beweis}%In Vorlesung: Behauptung 3
@ -217,7 +217,7 @@ schneiden sich.
$\overline{AP'}$ liegt in der anderen Halbebene
bzgl. $PA \Rightarrow C \notin \overline{P'A} \Rightarrow C \in \overline{AQ}$
\end{enumerate}
Da $C \in PB^+$ und $C \in \overline{AQ}$ folgt nun direkt:
Da $C \in PB^+$ und $C \in \overline{AQ}$ folgt nun direkt:
$\emptyset \neq \Set{C} \subseteq PB^+ \cap \overline{AQ} \qed$
\end{beweis}
@ -286,7 +286,7 @@ schneiden sich.
\begin{bemerkung}\label{kor:beh2'}
Sei $(X, d, G)$ eine Geometrie, die \ref{axiom:1}~-~\ref{axiom:3}
erfüllt, $P, Q \in X$ mit $P \neq Q$ und $\varphi$ eine Isometrie mit
erfüllt, $P, Q \in X$ mit $P \neq Q$ und $\varphi$ eine Isometrie mit
$\varphi(P) = P$ und $\varphi(Q) = Q$.
Dann gilt $\varphi(S) = S\;\;\;\forall S \in PQ$.
@ -302,7 +302,7 @@ schneiden sich.
&\overset{\mathclap{\ref{axiom:3.1}}}{\Rightarrow} \varphi(S) = S
\end{align*}
$\qed$
$\qed$
\end{beweis}
\begin{proposition}\label{satz:14.4}%In Vorlesung: Satz 14.4
@ -310,7 +310,7 @@ schneiden sich.
gibt es zu $P, P', Q, Q'$ mit $d(P, Q) = d(P', Q')$ höchstens
zwei Isometrien mit $\varphi(P) = P'$ und $\varphi(Q) = Q'$
Aus den Axiomen folgt, dass es in
Aus den Axiomen folgt, dass es in
der Situation von \ref{axiom:4} höchstens zwei Isometrien mit
$\varphi_i(P) = P'$ und $\varphi_i(Q) = Q'$ gibt.
\end{proposition}
@ -332,11 +332,11 @@ schneiden sich.
Nun zu den Beweisen der Teilaussagen:
\begin{enumerate}[label=(Teil \roman*),ref=(Teil \roman*)]
\item Sei $R \in X \setminus PQ$. Von den drei Punkten
\item Sei $R \in X \setminus PQ$. Von den drei Punkten
$\varphi_1(R), \varphi_2(R), \varphi_3(R)$ liegen zwei
in der selben Halbebene bzgl. $P'Q' = \varphi_i(PQ)$.
\Obda seien $\varphi_1(R)$ und $\varphi_2(R)$ in der
\Obda seien $\varphi_1(R)$ und $\varphi_2(R)$ in der
selben Halbebene.
Es gilt: $\begin{aligned}[t]
@ -392,7 +392,7 @@ schneiden sich.
Also gilt insbesondere $\varphi(\triangle A'B'C') = \triangle ABC$. $\qed$
\end{beweis}
\begin{bemerkung}[WSW-Kongruenzsatz]\xindex{Kongruenzsatz!WSW}%
Sei $(X, d, G)$ eine Geometrie, die \ref{axiom:1}~-~\ref{axiom:4} erfüllt.
Seien außerdem $\triangle ABC$ und $\triangle A'B'C'$ Dreiecke, für die gilt:
@ -424,15 +424,15 @@ schneiden sich.
\begin{definition}\label{def:14.8}%In Vorlesung: 14.8
\begin{defenum}
\item \label{def:14.8a} Ein \textbf{Winkel}\xindex{Winkel} ist ein Punkt $P \in X$
\item \label{def:14.8a} Ein \textbf{Winkel}\xindex{Winkel} ist ein Punkt $P \in X$
zusammen mit $2$ Halbgeraden mit Anfangspunkt $P$.\\
Man schreibt: $\angle R_1 P R_2$ bzw. $\angle R_2 P R_1$\footnote{Für dieses Skript gilt: $\angle R_1 P R_2 = \angle R_2 P R_1$. Also sind insbesondere alle Winkel $ \leq 180^\circ$.}
\item Zwei Winkel sind \textbf{gleich}, wenn es eine Isometrie gibt,
\item Zwei Winkel sind \textbf{gleich}, wenn es eine Isometrie gibt,
die den einen Winkel auf den anderen abbildet.
\item \label{def:14.8c} $\angle R_1' P' R_2'$ heißt \textbf{kleiner} als
$\angle R_1 P R_2$, wenn es eine Isometrie $\varphi$
gibt, mit $\varphi(P') = P$, $\varphi(P'R'^{+}_{1}) = PR_{1}^{+}$
und $\varphi(R_2')$ liegt in der gleichen Halbebene
und $\varphi(R_2')$ liegt in der gleichen Halbebene
bzgl. $PR_1$ wie $R_2$ und in der gleichen Halbebene
bzgl. $PR_2$ wie $R_1$
\item \label{def:14.8d} Im Dreieck $\triangle PQR$ gibt es \textbf{Innenwinkel}\xindex{Innenwinkel} und
@ -455,7 +455,7 @@ schneiden sich.
\end{figure}
\begin{bemerkung}\label{bem:14.9}%In Vorlesung: Bemerkung 14.9
In einem Dreieck ist jeder Innenwinkel kleiner als jeder nicht
In einem Dreieck ist jeder Innenwinkel kleiner als jeder nicht
anliegende Außenwinkel.
\end{bemerkung}
@ -481,7 +481,7 @@ schneiden sich.
\caption{Situation aus \cref{bem:14.9}}
\end{figure}
Es gilt: $d(Q,M) = d(M,R)$ und $d(P,M) = d(M,A)$ sowie
Es gilt: $d(Q,M) = d(M,R)$ und $d(P,M) = d(M,A)$ sowie
$\angle PMR = \angle AMQ \Rightarrow \triangle MRQ$ ist
kongruent zu $\triangle AMQ$, denn eine der beiden Isometrien, die
$\angle PMR$ auf $\angle AMQ$ abbildet, bildet $R$ auf $Q$ und
@ -496,7 +496,7 @@ schneiden sich.
\begin{proposition}[Existenz der Parallelen]\label{prop:14.7}%In Vorlesung: Proposition 14.7
Sei $(X, d, G)$ eine Geometrie mit den Axiomen \ref{axiom:1}~-~\ref{axiom:4}.
Dann gibt es zu jeder Geraden $g \in G$ und jedem Punkt $P \in X \setminus g$
Dann gibt es zu jeder Geraden $g \in G$ und jedem Punkt $P \in X \setminus g$
mindestens eine Parallele $h \in G$ mit $P \in h$ und $g \cap h = \emptyset$.
\end{proposition}
@ -535,7 +535,7 @@ Halbebene bzgl. $PQ$ liegt wie $R$.
\begin{figure}[htp]
\centering
\includegraphics[width=0.4\linewidth, keepaspectratio]{figures/Spherical_triangle_3d_opti.png}
\includegraphics[width=0.4\linewidth, keepaspectratio]{figures/Spherical_triangle_3d_opti.png}
\caption{In der sphärischen Geometrie gibt es, im Gegensatz zur euklidischen Geometrie, Dreiecke mit drei $90^\circ$-Winkeln.}
\label{fig:spherical-triangle}
\end{figure}
@ -567,7 +567,7 @@ Sei im Folgenden \enquote{$\IWS$} die \enquote{Innenwinkelsumme}.
Sei $\alpha$ ein Innenwinkel von $\triangle$.
\begin{behauptung}
Es gibt ein Dreieck $\triangle'$ mit
Es gibt ein Dreieck $\triangle'$ mit
$\IWS(\triangle') = \IWS(\triangle)$ und einem Innenwinkel
$\alpha' \leq \frac{\alpha}{2}$.
@ -579,8 +579,8 @@ Sei im Folgenden \enquote{$\IWS$} die \enquote{Innenwinkelsumme}.
\end{behauptung}
\begin{beweis}
Es seien $A, B, C \in X$ und $\triangle $ das Dreieck mit den
Eckpunkten $A, B, C$ und $\alpha$ sei der Innenwinkel bei $A$,
Es seien $A, B, C \in X$ und $\triangle $ das Dreieck mit den
Eckpunkten $A, B, C$ und $\alpha$ sei der Innenwinkel bei $A$,
$\beta$ der Innenwinkel bei $B$ und $\gamma$ der Innenwinkel bei $C$.
Sei $M$ der Mittelpunkt der Strecke $\overline{BC}$. Sei außerdem
@ -613,7 +613,7 @@ Sei im Folgenden \enquote{$\IWS$} die \enquote{Innenwinkelsumme}.
\end{figure}
\begin{beweis}
Sei $g$ eine Parallele von $AB$ durch $C$.
Sei $g$ eine Parallele von $AB$ durch $C$.
\begin{itemize}
\item Es gilt $\alpha' = \alpha$ wegen \cref{prop:14.7}.
@ -639,7 +639,7 @@ WSW.\xindex{Kongruenzsatz!SWW}
\label{fig:hyperbolische-geometrie-2}
\end{figure}
Der Beweis wird hier nicht geführt. Für Beweisvorschläge wäre ich
Der Beweis wird hier nicht geführt. Für Beweisvorschläge wäre ich
dankbar.
\begin{figure}[htp]
@ -730,10 +730,10 @@ $\xRightarrow{\text{Strahlensatz}} \frac{a}{h_c} = \frac{c}{h_a} \rightarrow a \
\item $(\mdr^2, d_\text{Euklid})$ ist offensichtlich eine euklidische Ebene.
\item Sei $(X,d)$ eine euklidische Ebene und $g_1, g_2$ Geraden
in $X$, die sich in einem Punkt $0$ im rechten Winkel
schneiden.
schneiden.
Sei $P \in X \setminus (g_1 \cup g_2)$ ein Punkt und $P_X$ der
Fußpunkt des Lots von $P$ auf $g_1$ (vgl. \cref{ub11:aufg3.c})
Sei $P \in X \setminus (g_1 \cup g_2)$ ein Punkt und $P_X$ der
Fußpunkt des Lots von $P$ auf $g_1$ (vgl. \cref{ub11:aufg3.c})
und $P_Y$ der Fußpunkt des Lots von $P$ auf $g_2$.
Sei $x_P := d(P_X, 0)$ und $y_P := d(P_Y, 0)$.
@ -754,11 +754,11 @@ $\xRightarrow{\text{Strahlensatz}} \frac{a}{h_c} = \frac{c}{h_a} \rightarrow a \
\label{fig:14.13.0.1}
\end{figure}
Sei $h:X \rightarrow \mdr^2$ eine Abbildung mit
$h(P) := (x_P, y_P)$
Dadurch wird $h$ auf dem Quadranten
definiert, in dem $P$ liegt, d.~h.
definiert, in dem $P$ liegt, d.~h.
\[\forall Q \in X \text{ mit } \overline{PQ} \cap g_1 = \emptyset = \overline{PQ} \cap g_2\]
Fortsetzung auf ganz $X$ durch konsistente Vorzeichenwahl.
@ -833,7 +833,7 @@ $\xRightarrow{\text{Strahlensatz}} \frac{a}{h_c} = \frac{c}{h_a} \rightarrow a \
Betrachte nun $z_1$ und $z_2$ als Punkte in der
euklidischen Ebene. Die Mittelsenkrechte zu diesen
Punkten schneidet die $x$-Achse. Alle Punkte auf
der Mittelsenkrechten zu $z_1$ und $z_2$ sind gleich
der Mittelsenkrechten zu $z_1$ und $z_2$ sind gleich
weit von $z_1$ und $z_2$ entfernt. Daher ist
der Schnittpunkt mit der $x$-Achse der Mittelpunkt
eines Kreises durch $z_1$ und $z_2$ (vgl. \cref{fig:hyperbolische-geometrie-axiom-1-2})
@ -867,7 +867,7 @@ $\xRightarrow{\text{Strahlensatz}} \frac{a}{h_c} = \frac{c}{h_a} \rightarrow a \
\underline{Zu zeigen:}
$\forall A \in H_i$, $B \in H_j$ mit
$i,j \in \Set{1,2}$ gilt:
$i,j \in \Set{1,2}$ gilt:
$\overline{AB} \cap g \neq \emptyset \Leftrightarrow i \neq j$\\
\enquote{$\Leftarrow$}: $A \in H_1, B \in H_2: \overline{AB} \cap g \neq \emptyset$
@ -882,10 +882,10 @@ $\xRightarrow{\text{Strahlensatz}} \frac{a}{h_c} = \frac{c}{h_a} \rightarrow a \
Sei $h$ die Gerade, die durch $A$ und $B$ geht.
Da $A,B \notin g$, aber $A, B \in h$ gilt, haben $g$ und $h$
Da $A,B \notin g$, aber $A, B \in h$ gilt, haben $g$ und $h$
insbesondere
mindestens einen unterschiedlichen Punkt. Aus \ref{axiom:1.1} folgt, dass sich
$g$ und $h$ in höchstens einen Punkt schneiden. Sei $C$ dieser
$g$ und $h$ in höchstens einen Punkt schneiden. Sei $C$ dieser
Punkt.
Aus $A,B \notin g$ folgt: $C \neq A$ und $C \neq B$. Also liegt
@ -903,8 +903,8 @@ $\xRightarrow{\text{Strahlensatz}} \frac{a}{h_c} = \frac{c}{h_a} \rightarrow a \
\end{beweis}
\begin{definition}\xindex{Möbiustransformation}%
Es seien $a,b,c,d \in \mdr$ mit $ad - bc \neq 0$ und
$\sigma: \mdc \rightarrow \mdc$ eine Abbildung definiert durch
Es seien $a,b,c,d \in \mdr$ mit $ad - bc \neq 0$ und
$\sigma: \mdc \rightarrow \mdc$ eine Abbildung definiert durch
\[\sigma(z) := \frac{az + b}{cz+d}\]
$\sigma$ heißt \textbf{Möbiustransformation}.
@ -953,7 +953,7 @@ $\xRightarrow{\text{Strahlensatz}} \frac{a}{h_c} = \frac{c}{h_a} \rightarrow a \
&= \frac{a(a'z+b')+b(c'z+d')}{c(a'z+b') + d(c'z+d')}\\
&= \frac{(aa'+bc')z + ab' + bd'}{(ca'+db')z + cb' + dd'}\\
&= \begin{pmatrix}aa'+bc'&ab'+bd'\\ca'+db'&cb'+dd'\end{pmatrix} \circ z\\
&= \left ( \begin{pmatrix}a&b\\c&d\end{pmatrix} \cdot \begin{pmatrix}a'&b'\\c'&d'\end{pmatrix} \right ) \circ z
&= \left ( \begin{pmatrix}a&b\\c&d\end{pmatrix} \cdot \begin{pmatrix}a'&b'\\c'&d'\end{pmatrix} \right ) \circ z
\end{align*}
\item Es gilt $\sigma(z) = (-\sigma)(z)$ für alle $\sigma \in \SL_2(\mdr)$
und $z \in \mdh$.
@ -975,7 +975,7 @@ $\xRightarrow{\text{Strahlensatz}} \frac{a}{h_c} = \frac{c}{h_a} \rightarrow a \
Daher genügt es zu zeigen, dass man mit $A_{\lambda}$, $B_t$ und $C$ alle Matrizen
aus $\SL_2(\mdr)$ erzeugen kann, genügt es also von einer beliebigen
Matrix durch Multiplikation mit Matrizen der Form $A_{\lambda}$,
Matrix durch Multiplikation mit Matrizen der Form $A_{\lambda}$,
$B_t$ und $C$ die Einheitsmatrix zu generieren.
Sei also
@ -1056,9 +1056,9 @@ $\xRightarrow{\text{Strahlensatz}} \frac{a}{h_c} = \frac{c}{h_a} \rightarrow a \
\begin{definition}\xindex{Doppelverhältnis}%In Vorlesung: Def+Prop 15.4
Seien $z_1, z_2, z_3, z_4 \in \mdc$ paarweise verschieden.
Dann heißt
Dann heißt
\[\DV(z_1, z_2, z_3, z_4) := \frac{\frac{z_1 - z_4}{z_1 - z_2}}{\frac{z_3 - z_4}{z_3 - z_2}} = \frac{(z_1 - z_4) \cdot (z_3 - z_2)}{(z_1 - z_2) \cdot (z_3 - z_4)}\]
\textbf{Doppelverhältnis} von
\textbf{Doppelverhältnis} von
$z_1, \dots, z_4$.
\end{definition}
@ -1071,7 +1071,7 @@ $\xRightarrow{\text{Strahlensatz}} \frac{a}{h_c} = \frac{c}{h_a} \rightarrow a \
oder wenn zwei der $z_i$ gleich sind.
\item $\DV(0, 1, \infty, z_4) = z_4$ (Der Fall $z_4 \in \Set{0, 1, \infty}$ ist zugelassen).
\item \label{bem:15.4d} Für $\sigma \in \PSL_2(\mdc)$ und $z_1, \dots, z_4 \in \mdc \cup \Set{\infty}$
ist
ist
\[\DV(\sigma(z_1), \sigma(z_2), \sigma(z_3), \sigma(z_4)) = \DV(z_1, z_2, z_3, z_4)\]
und für $\sigma(z) = \frac{1}{\overline{z}}$ gilt
\[\DV(\sigma(z_1), \sigma(z_2), \sigma(z_3), \sigma(z_4)) = \overline{\DV(z_1, z_2, z_3, z_4)}\]
@ -1110,7 +1110,7 @@ $\xRightarrow{\text{Strahlensatz}} \frac{a}{h_c} = \frac{c}{h_a} \rightarrow a \
Durch Einsetzen ergibt sich $\DV(z_1, \dots, z_4)=1$.
\end{itemize}
Im Fall, dass ein $z_i = \infty$ ist, ist
Im Fall, dass ein $z_i = \infty$ ist, ist
entweder $\DV(0, 1, \infty, z_4) = 0$ oder $\DV(0, 1, \infty, z_4) \pm \infty$
\item $\DV(0, 1, \infty, z_4) = \frac{(0- z_4) \cdot (\infty - 1)}{(0 -1) \cdot (\infty - z_4)} = \frac{z_4 \cdot (\infty - 1)}{\infty - z_4} = z_4$
\item Wenn jemand diesen Beweis führt, bitte an info@martin-thoma.de schicken.%TODO

View file

@ -12,16 +12,16 @@
\section{Krümmung von Kurven}\label{sec:Kurvenkrümmung}
\begin{definition}%In Vorlesung: Def.+Bem. 16.1
Sei $\gamma: I = [a, b] \rightarrow \mdr^n$ eine Kurve.
\begin{defenum}
\item Die Kurve $\gamma$ heißt
\item Die Kurve $\gamma$ heißt
\textbf{durch Bogenlänge parametrisiert}\xindex{parametrisiert!durch Bogenlänge},
wenn gilt:
\[\|\gamma'(t)\|_2 = 1 \;\;\; \forall t \in I\]
Dabei ist $\gamma'(t) = \left (\gamma_1'(t), \gamma_2'(t), \dots, \gamma_n'(t) \right)$.
\item $l(\gamma) = \int_a^b \|\gamma'(t)\| \mathrm{d} t$ heißt
\textbf{Länge von $\gamma$}\xindex{Kurve!Länge einer}.
\end{defenum}
\end{defenum}
\end{definition}
\begin{bemerkung}[Eigenschaften von Kurven I]%In Vorlesung: Def.+Bem. 16.1
@ -29,7 +29,7 @@
\begin{bemenum}
\item Ist $\gamma$ durch Bogenlänge parametrisiert, so ist $l(\gamma) = b-a$.
\item \label{bem:16.1d} Ist $\gamma$ durch Bogenlänge parametrisiert, so ist
\item \label{bem:16.1d} Ist $\gamma$ durch Bogenlänge parametrisiert, so ist
$\gamma'(t)$ orthogonal zu $\gamma''(t)$ für alle $t \in I$.
\end{bemenum}
\end{bemerkung}
@ -85,8 +85,8 @@ Da $n(t)$ und $\gamma''(t)$ nach \cref{bem:16.1d} linear
da gilt:
\begin{align*}
\langle n(t), \gamma'(t) \rangle &=
\left \langle
\langle n(t), \gamma'(t) \rangle &=
\left \langle
\begin{pmatrix}- \cos \frac{t}{r}\\ - \sin \frac{t}{r}\end{pmatrix},
\begin{pmatrix}- \sin \frac{t}{r}\\ \cos \frac{t}{r}\end{pmatrix}
\right \rangle\\
@ -128,7 +128,7 @@ Da $n(t)$ und $\gamma''(t)$ nach \cref{bem:16.1d} linear
Also gilt:
\[\det(\gamma'(t), n(t), b(t)) = 1\]
$b(t)$ heißt \textbf{Binormalenvektor}\xindex{Binormalenvektor},
die Orthonormalbasis
die Orthonormalbasis
\[\Set{\gamma'(t), n(t), b(t)}\]
heißt \textbf{begleitendes Dreibein}\xindex{Dreibein!begleitendes}.
\end{defenum}
@ -185,16 +185,16 @@ für eine $C^\infty$-Funktion $f: \mdr^3 \rightarrow \mdr$.
\begin{beweis}\leavevmode
\begin{enumerate}[label=\alph*)]
\item \label{bew:tangentialebene.a} $J_F$ ist eine $3 \times 2$-Matrix, die mit einem $2 \times 1$-Vektor
multipliziert wird. Das ist eine lineare Abbildung und aus der
multipliziert wird. Das ist eine lineare Abbildung und aus der
linearen Algebra ist bekannt, das das Bild ein Vektorraum ist.
Da $\rang(J_F) = 2$, ist auch $\dim (T_s S) = 2$.
\item Hier kann man wie in \cref{bew:tangentialebene.a} argumentieren
\item $T_s S = \{x \in \mdr^3 | \exists \text{parametrisierte Kurve }
\gamma:[- \varepsilon, + \varepsilon] \rightarrow S
\text{ für ein } \varepsilon > 0
\item $T_s S = \{x \in \mdr^3 | \exists \text{parametrisierte Kurve }
\gamma:[- \varepsilon, + \varepsilon] \rightarrow S
\text{ für ein } \varepsilon > 0
\text{ mit } \gamma(0) = s \text{ und } \gamma'(0) = x
\}$\\
Wenn jemand diesen Beweis führt, bitte an info@martin-thoma.de
Wenn jemand diesen Beweis führt, bitte an info@martin-thoma.de
schicken.%TODO
\item Sei $x \in T_s S, \gamma:[-\varepsilon, +\varepsilon] \rightarrow S$
eine parametrisierte Kurve mit $\varepsilon > 0$ und $\gamma'(0) = s$,
@ -231,7 +231,7 @@ Im Folgenden werden diese Begriffe jedoch synonym benutzt.
von $s$ und eine lokale Parametrisierung $F: U \rightarrow V$
von $S$ um $s$, sodass auf $F(U) = V \cap S$
ein stetiges Normalenfeld existiert.
\item $S$ ist genau dann orientierbar, wenn es einen
\item $S$ ist genau dann orientierbar, wenn es einen
differenzierbaren Atlas von $S$ aus lokalen Parametrisierungen
$F_i: U_i \rightarrow V_i,\;i \in I$ gibt, sodass
für alle $i, j \in F$ und alle $s \in V_i \cap V_j \cap S$
@ -256,7 +256,7 @@ Im Folgenden werden diese Begriffe jedoch synonym benutzt.
\begin{figure}[htp]\xindex{Möbiusband}
\centering
\includegraphics[width=0.5\linewidth, keepaspectratio]{figures/moebius-strip.pdf}
\includegraphics[width=0.5\linewidth, keepaspectratio]{figures/moebius-strip.pdf}
\caption{Möbiusband}
\label{fig:moebius-strip}
\end{figure}
@ -266,7 +266,7 @@ Im Folgenden werden diese Begriffe jedoch synonym benutzt.
Sei $S$ eine reguläre Fläche, $s \in S$, $n(s)$ ist ein Normalenvektor
in $s$, $x \in T_s S$, $\|x\| = 1$.
Sei $E$ der von $x$ und $n(s)$ aufgespannte 2-dimensionale
Sei $E$ der von $x$ und $n(s)$ aufgespannte 2-dimensionale
Untervektorraum von $\mdr^3$.
Dann gibt es eine Umgebung $V \subseteq \mdr^3$ von $s$, sodass
@ -277,7 +277,7 @@ Im Folgenden werden diese Begriffe jedoch synonym benutzt.
\end{bemerkung}
\begin{beweis}
\enquote{Satz über implizite Funktionen}\footnote{Siehe z.~B.
\enquote{Satz über implizite Funktionen}\footnote{Siehe z.~B.
\url{https://github.com/MartinThoma/LaTeX-examples/tree/master/documents/Analysis\%20II}}
\end{beweis}
@ -362,18 +362,18 @@ Im Folgenden werden diese Begriffe jedoch synonym benutzt.
\end{beweis}
\begin{bemerkung}%In Vorlesung: Bem.+Def. 18.6
Sei $S$ eine reguläre Fläche und $n=n(s)$ ein Normalenvektor an
Sei $S$ eine reguläre Fläche und $n=n(s)$ ein Normalenvektor an
$S$ in $s$.
Sei $T_{s}^{1} S = \Set{x \in T_s S | \|x\| = 1} \cong S^1$.
Dann ist
Dann ist
\[ \kappanor^n(s): T^1_s S \rightarrow \mdr, \;\;\; x \mapsto \kappanor(s,x)\]
eine glatte Funktion und
eine glatte Funktion und
$\Bild \kappanor^n(s)$ ist ein abgeschlossenes Intervall.
\end{bemerkung}
\begin{definition}\xindex{Hauptkrümmung}\xindex{Gauß-Krümmung}%In Vorlesung: Bem.+Def. 18.6
Sei $S$ eine reguläre Fläche und $n=n(s)$ ein Normalenvektor an
Sei $S$ eine reguläre Fläche und $n=n(s)$ ein Normalenvektor an
$S$ in $s$.
\begin{defenum}
@ -388,7 +388,7 @@ Im Folgenden werden diese Begriffe jedoch synonym benutzt.
\end{definition}
\begin{bemerkung}%In Vorlesung: Bem.+Def. 18.6
Ersetzt man $n$ durch $-n$, so gilt:
Ersetzt man $n$ durch $-n$, so gilt:
\begin{align*}
\kappanor^{-n}(s, x) &= - \kappanor^n(x)\; \forall x \in T_s^1 S\\
@ -457,7 +457,7 @@ $s$. Weiter sei $p := F^{-1}(s)$.
\item \label{bem:19.1a} Die Einschränkung des Standardskalarproduktes des $\mdr^3$ auf
$T_s S$ macht $T_s S$ zu einem euklidischen Vektorraum.
\item $\Set{D_p F(e_1), D_p F(e_2)}$ ist eine Basis von $T_s S$.
\item Bzgl. der Basis $\Set{D_p F(e_1), D_p F(e_2)}$ hat das
\item Bzgl. der Basis $\Set{D_p F(e_1), D_p F(e_2)}$ hat das
Standardskalarprodukt aus \cref{bem:19.1a} die Darstellungsmatrix
$I_S$.
\item $g_{i,j}(s)$ ist eine differenzierbare Funktion von $s$.
@ -465,7 +465,7 @@ $s$. Weiter sei $p := F^{-1}(s)$.
\end{bemerkung}
\begin{bemerkung}
\[\det(I_S) = \left \| \frac{\partial F}{\partial u_1}(p) \times \frac{\partial F}{\partial u_2}(p) \right \|^2\]
\[\det(I_S) = \left \| \frac{\partial F}{\partial u_1}(p) \times \frac{\partial F}{\partial u_2}(p) \right \|^2\]
\end{bemerkung}
\begin{beweis}\leavevmode
@ -495,7 +495,7 @@ $s$. Weiter sei $p := F^{-1}(s)$.
\begin{defenum}
\item Das Differential $\mathrm{d} A = \sqrt{\det (I)} \mathrm{d} u_1 \mathrm{d} u_2$
heißt \textbf{Flächenelement} von $S$ bzgl. der Parametrisierung $F$.
\item \label{def:berechenbares-integral}Für eine Funktion $f: V \rightarrow \mdr$ heißt
\item \label{def:berechenbares-integral}Für eine Funktion $f: V \rightarrow \mdr$ heißt
\[\int_V f \mathrm{d} A := \int_U f(\underbrace{F(u_1, u_2)}_{=: s}) \sqrt{\det I(s)} \mathrm{d} u_1 \mathrm{d} u_2\]
der \textbf{Wert des Integrals} von $f$ über $V$, falls das Integral rechts
existiert.
@ -535,7 +535,7 @@ $s$. Weiter sei $p := F^{-1}(s)$.
\begin{propenum}
\item \label{prop:5.1a} $n$ induziert für jedes $s \in S$ eine lineare Abbildung $d_s n: T_s S \rightarrow T_{n(s)} S^2$
durch
durch
\[d_s n(x) = \frac{\mathrm{d}}{\mathrm{d} t} n (\underbrace{s \text{\enquote{+}} tx}_{\mathclap{\text{Soll auf Fläche $S$ bleiben}}}) \Bigr |_{t=0}\]
Die Abbildung $d_s n$ heißt \textbf{Weingarten-Abbildung}
\item $T_{n(s)} S^2 = T_s S$.
@ -560,7 +560,7 @@ $s$. Weiter sei $p := F^{-1}(s)$.
Sei $x_i = D_p F(e_i) = \frac{\partial F}{\partial u_i} (p)\;\;\; i = 1,2$
\underline{Beh.:}
\underline{Beh.:}
$\langle x_i, d_s n(x_j) \rangle = \langle \frac{\partial^2 F}{\partial u_i \partial u_j} (p), d_s n (x_i) \rangle$
$\Rightarrow \langle \frac{\partial^2 F}{\partial u_i \partial u_j} (p), d_s n (x_i) \rangle = \langle x_j, d_s n (x_i) \rangle$
@ -597,9 +597,9 @@ $s$. Weiter sei $p := F^{-1}(s)$.
\begin{beweis}
Nach \cref{def:18.4} ist $\kappanor(s, \gamma) = \langle \gamma''(0), n(s) \rangle$.
Nach Voraussetzung gilt
Nach Voraussetzung gilt
\[n(\gamma(t)) \perp \gamma'(t) \Leftrightarrow \langle \gamma''(0), n(s) \rangle = 0\]
Die Ableitung nach $t$ ergibt
Die Ableitung nach $t$ ergibt
\begin{align*}
0 &= \frac{\mathrm{d}}{\mathrm{d}t}(\langle n (\gamma(t)), \gamma'(t))\\
&= \left \langle \frac{\mathrm{d}}{\mathrm{d}t} n(\gamma(t)) \Bigr |_{t=0}, \gamma'(0) \right \rangle + \langle n(s), \gamma''(0) \rangle\\
@ -629,7 +629,7 @@ $s$. Weiter sei $p := F^{-1}(s)$.
Eigenvektoren $y_1, y_2$ von $II_s$. Ist $x \in T_s S$, $\|x\| = 1$,
so gibt es $\varphi \in [0,2\pi)$ mit $x = \cos \varphi \cdot y_1 + \sin \varphi \cdot y_2$.
Seien $\lambda_1, \lambda_2$ die Eigenwerte von $II_s$, also
Seien $\lambda_1, \lambda_2$ die Eigenwerte von $II_s$, also
$II_s(y_i, y_i) = \lambda_i$. Dann gilt:
\begin{align*}
II_s (x,x) &= \cos^2 \varphi \lambda_1 + \sin^2 \varphi \lambda_2\\

View file

@ -20,13 +20,13 @@
$(X, \fT_X)$ ist also nicht hausdorffsch.
\textbf{Teilaufgabe c)} Nach Bemerkung \ref{Trennungseigenschaft}
sind metrische Räume hausdorffsch. Da $(X, \fT_X)$ nach (b) nicht
sind metrische Räume hausdorffsch. Da $(X, \fT_X)$ nach (b) nicht
hausdorffsch ist, liefert die Kontraposition der Trennungseigenschaft,
dass $(X, \fT_X)$ kein metrischer Raum sein kann.
\end{solution}
\begin{solution}[\ref{ub1:aufg4}]
\textbf{Teilaufgabe a)}
\textbf{Teilaufgabe a)}
\textbf{Beh.:} $\forall a \in \mdz: \Set{a}$ ist abgeschlossen.
@ -35,7 +35,7 @@
Wenn jemand diese Aufgabe gemacht hat, bitte die Lösung an info@martin-thoma.de
schicken.%TODO
\textbf{Teilaufgabe b)}
\textbf{Teilaufgabe b)}
\textbf{Beh.:} $\Set{-1, 1}$ ist nicht offen
@ -50,7 +50,7 @@
in dieser Topologie offen sein $\Rightarrow \Set{-1,1}$ ist
nicht offen. $\qed$
\textbf{Teilaufgabe c)}
\textbf{Teilaufgabe c)}
\textbf{Beh.:} Es gibt unendlich viele Primzahlen.
@ -58,7 +58,7 @@
Annahme: Es gibt nur endlich viele Primzahlen $p \in \mdp$
Dann ist
Dann ist
\[\mdz \setminus \Set{-1, +1} \overset{\text{FS d. Arithmetik}}= \bigcup_{p \in \mdp} U_{0,p}\]
endlich. Das ist ein Widerspruch zu $|\mdz|$ ist unendlich und
$|\Set{-1,1}|$ ist endlich. $\qed$
@ -67,7 +67,7 @@
\begin{solution}[\ref{ub2:aufg4}]
\begin{enumerate}[label=(\alph*)]
\item \textbf{Beh.:} Die offenen Mengen von $P$ sind
Vereinigungen von Mengen der Form
Vereinigungen von Mengen der Form
\[\prod_{j \in J} U_j \times \prod_{i \in \mdn, i \neq j} P_i\]
wobei $J \subseteq \mdn$ endlich und $U_j \subseteq P_j$
offen ist.
@ -79,7 +79,7 @@
$\forall{j \in J}$
eine Basis der Topologie.
Damit sind die offenen
Damit sind die offenen
Mengen von $P$ Vereinigungen von Mengen der obigen
Form. $\qed$
\end{beweis}
@ -95,10 +95,10 @@
für alle $i \in \mdn$ gilt entweder $p_i(Z) \subseteq \Set{0}$
oder $p_i(Z) \subseteq \Set{1}$. Es sei $z_i \in \Set{0,1}$
so, dass $p_i(Z) \subseteq \Set{z_i}$ für alle $i \in \mdn$.
Dann gilt also:
Dann gilt also:
\[\underbrace{p_i(x)}_{= x_i} = z_i = \underbrace{p_i(y)}_{= y_i} \forall i \in \mdn\]
Somit folgt: $x = y \qed$
\end{beweis}
\end{enumerate}
\end{solution}
@ -107,9 +107,9 @@
\begin{enumerate}[label=(\alph*)]
\item \textbf{Beh.:} $\GL_n(\mdr)$ ist nicht kompakt.\\
\textbf{Bew.:} $\det: \GL_n(\mdr) \rightarrow \mdr \setminus \Set{0}$
ist stetig. Außerdem ist
$\det(\GL_n(\mdr)) = \mdr \setminus \Set{0}$ nicht
kompakt. $\overset{\ref{kor:5.6}}{\Rightarrow}$
ist stetig. Außerdem ist
$\det(\GL_n(\mdr)) = \mdr \setminus \Set{0}$ nicht
kompakt. $\overset{\ref{kor:5.6}}{\Rightarrow}$
$\GL_n(\mdr)$ ist nicht kompakt. $\qed$
\item \textbf{Beh.:} $\SL_1(\mdr)$ ist nicht kompakt, für $n > 1$ ist $\SL_n(\mdr)$ kompakt.\\
\textbf{Bew.:} Für $\SL_1(\mdr)$ gilt:
@ -119,7 +119,7 @@
$\SL_n(\mdr) \subseteq \GL_n(\mdr)$ lässt sich mit einer
Teilmenge des $\mdr^{n^2}$ identifizieren. Nach \cref{satz:heine-borel}
sind diese genau dann kompakt, wenn sie beschränkt und
sind diese genau dann kompakt, wenn sie beschränkt und
abgeschlossen sind. Definiere nun für für $n \in \mdn_{\geq 2}, m \in \mdn$:
\[A_m = \text{diag}_n(m, \frac{1}{m}, \dots, 1)\]
Dann gilt: $\det A_m = 1$, d.~h. $A_m \in \SL_n(\mdr)$,
@ -138,7 +138,7 @@
nachgelesen werden.
\begin{definition}\xindex{Homomorphismus}%
Seien $(G, *)$ und $(H, \circ)$ Gruppen und
Seien $(G, *)$ und $(H, \circ)$ Gruppen und
$\varphi:G \rightarrow H$ eine Abbildung.
$\varphi$ heißt \textbf{Homomorphismus}, wenn
@ -161,16 +161,16 @@
\begin{solution}[\ref{ub3:meinsExtra2}]
Die Definition einer Isotopie kann auf \cpageref{def:Isotopie} nachgelesen
werden, die einer Isometrie auf \cpageref{def:Isometrie}.
\begin{definition}\xindex{Isomorphismus}%
Seien $(G, *)$ und $(H, \circ)$ Gruppen und
Seien $(G, *)$ und $(H, \circ)$ Gruppen und
$\varphi:G \rightarrow H$ eine Abbildung.
$\varphi$ heißt \textbf{Isomorphismus}, wenn $\varphi$ ein bijektiver
Homomorphismus ist.
\end{definition}
Eine Isotopie ist also für Knoten definiert, Isometrien machen nur in
Eine Isotopie ist also für Knoten definiert, Isometrien machen nur in
metrischen Räumen Sinn und ein Isomorphismus benötigt eine Gruppenstruktur.
\end{solution}
@ -180,7 +180,7 @@
\textbf{Beh.:} $M$ ist wegzusammehängend $\gdw M$ ist zusammenhängend
\begin{beweis}
\enquote{$\Rightarrow$}: Da $M$ insbesondere ein
topologischer Raum ist folgt diese Richtung direkt
topologischer Raum ist folgt diese Richtung direkt
aus \cref{kor:wegzusammehang-impliziert-zusammenhang}.
\enquote{$\Leftarrow$}: Seien $x,y \in M$ und
@ -192,7 +192,7 @@
\item $Z^C := \Set{\tilde{z} \in M | \nexists \text{Weg von } x \text{ nach } \tilde{z}}$ ist offen
Da $M$ eine Mannigfaltigkeit ist, existiert zu jedem
$\tilde{z} \in Z^C$ eine offene und wegzusammenhängende Umgebung
$\tilde{z} \in Z^C$ eine offene und wegzusammenhängende Umgebung
$U_{\tilde{z}} \subseteq M$.
Es gilt sogar $U_{\tilde{z}} \subseteq Z^C$, denn
@ -226,7 +226,7 @@
Da $(\mdr \setminus \Set{0}) \cup \Set{0_1}$ homöomorph
zu $\mdr$ ist, exisitert ein Weg $\gamma_1$ von $0_1$
zu einem beliebigen Punkt $a \in \mdr \setminus \Set{0}$.
Da $(\mdr \setminus \Set{0}) \cup \Set{0_2}$ ebenfalls
homöomorph zu $\mdr$ ist, existiert außerdem ein Weg
$\gamma_2$ von $a$ nach $0_2$. Damit existiert ein
@ -241,18 +241,18 @@
% \textbf{Beh.:} $H_k = \begin{cases}\mdr &\text{für } k\in \Set{0,1}\\
% 0 &\text{für } k \geq 2$
% \newcommand{\triangleSimplizialkomplex}{\mathord{\includegraphics[height=5ex]{figures/triangleSimplizialkomplex.pdf}}}
% \textbf{Bew.:} $S^1$ ist homöomorph zum Simplizialkomplex
% \textbf{Bew.:} $S^1$ ist homöomorph zum Simplizialkomplex
% $X = \triangleSimplizialkomplex$, d.~h. dem Rand
% von $\Delta^2$. Es gilt:
% \[X = \Set{\underbrace{v_0, v_1, v_2}_{A_0(X)}, \underbrace{\Delta (v_1, v_2)}_{=: a_0}, \underbrace{\underbrace{\Delta (v_0, v_2)}_{=: a_1}, \underbrace{\Delta(v_0, v_1)}_{=: a_2}}_{A_1(X)}}\]
% Damit folgt:
% Damit folgt:
% \begin{enumerate}
% \item Für $k \geq 2$ ist $C_k(X) \cong 0$, da es in diesen
% \item Für $k \geq 2$ ist $C_k(X) \cong 0$, da es in diesen
% Dimensionen keine Simplizes gibt, d.~h. $A_k(X) = \emptyset$ gilt.\\
% Also: $H_k(X) \cong 0 \; \forall k \geq 2$
% \item $C_0(X) = \Set{\sum_{i=0}^2 c_i v_i | c_i \in \mdr}$, da
% \item $C_0(X) = \Set{\sum_{i=0}^2 c_i v_i | c_i \in \mdr}$, da
% $A_0(x)$ Basis von $C_0(X)$ ist;\\
% $C_1(X) = \Set{\sum_{i=0}^2 c_i a_i | c_i \in \mdr}$, da
% $C_1(X) = \Set{\sum_{i=0}^2 c_i a_i | c_i \in \mdr}$, da
% $A_1(X)$ Basis von $C_1(X)$ ist.
% \item Für die Randabbildungen $d_i: C_i(X) \rightarrow C_{i-1}(X)$ gilt:
% $d_0 \equiv 0$, $d_1: C_1(X) \rightarrow C_0(X)$ ist definiert durch
@ -287,7 +287,7 @@
$\angle BCA \xRightarrow{\crefabbr{bem:14.9}} \measuredangle BC' A > \measuredangle BCA$\\
$\Rightarrow \measuredangle BCA < \measuredangle BC' A = \measuredangle ABC' < \measuredangle ABC $
Sei umgekehrt $\measuredangle ABC > \measuredangle BCA$,
kann wegen 1. Teil von \cref{ub11:aufg3.b} nicht
kann wegen 1. Teil von \cref{ub11:aufg3.b} nicht
$d(A,B) > d(A,C)$ gelten.\\
Wegen \cref{ub11:aufg3.a} kann nicht $d(A,B) = d(A,C)$
gelten.\\
@ -343,7 +343,7 @@
d(B, C) &= d(B', C')
\end{align*}
Sei $\varphi$ die Isometrie mit $\varphi(A) = A'$, $\varphi(B) = B'$ und
Sei $\varphi$ die Isometrie mit $\varphi(A) = A'$, $\varphi(B) = B'$ und
$\varphi(C')$ liegt in der selben Halbebene bzgl. $AB$ wie $C$. Diese
Isometrie existiert wegen \ref{axiom:4}.

View file

@ -5,23 +5,23 @@ wird, die die Inhalte noch lernen, sind sehr wahrscheinlich einige
Fehler im Skript. Das können Übertragungsfehler, Tippfehler oder
Verständnisprobleme sein.
Verbesserungsvorschläge (auch wenn es nur einzelne Textsetzungsprobleme oder
Verbesserungsvorschläge (auch wenn es nur einzelne Textsetzungsprobleme oder
Rechtschreibfehler sind) bitte immer direkt melden oder verbessern!
Den Verbesserungsvorschlag kann man
* entweder direkt selbst umsetzen und einen pull request machen oder
* mir per Email (info@martin-thoma.de) schicken.
Den Verbesserungsvorschlag kann man
* entweder direkt selbst umsetzen und einen pull request machen oder
* mir per Email (info@martin-thoma.de) schicken.
Ich werde dann versuchen die Verbesserungsvorschläge zeitnah einzuarbeiten.
Zeichnungen
===========
Das erstellen der Zeichnungen ist sehr zeitaufwendig. Das ist der
Das erstellen der Zeichnungen ist sehr zeitaufwendig. Das ist der
Grund, warum manchmal nur ein "TODO" im Dokument steht.
Ihr könnt mir gerne Zeichnungen schicken (entweder schön auf Papier
Zeichnen und abfotographieren / einscannen oder schon mit Inscape /
Gimp / ... oder sogar mit TikZ erstellen).
Gimp / ... oder sogar mit TikZ erstellen).
Akzeptable Formate sind: .jpg, .pdf, .svg, .png, .gif, .tex, .sketch
Alles andere kann ich vermutlich nicht einbinden.
@ -37,8 +37,8 @@ Rechtliches
===========
Die Autoren kann man über Git ermitteln. Ich schreibe meist nur den
Tafelanschrieb der Vorlesung ab; eventuell noch mit ein paar
Notizen meinerseits. Wenn mir Verbesserungsvorschläge per Email
geschickt werden, ist der Autor sowie das Datum der Email in der
Notizen meinerseits. Wenn mir Verbesserungsvorschläge per Email
geschickt werden, ist der Autor sowie das Datum der Email in der
Commit-Nachricht von Git zu sehen.
Bilder habe ich entweder selbst erstellt oder von tex.stackexchange.com.

View file

@ -6,19 +6,19 @@ der Vorlesung von Prof.~Dr.~Herrlich sowie die Mitschriften einiger
Übungen und Tutorien.
Das Skript ist kostenlos über \href{http://martin-thoma.com/geotopo/}{martin-thoma.com/geotopo}
verfügbar. Wer es gerne in A5 (Schwarz-Weiß, Ringbindung) für 10 Euro hätte,
verfügbar. Wer es gerne in A5 (Schwarz-Weiß, Ringbindung) für 10 Euro hätte,
kann mir eine E-Mail schicken (info@martin-thoma.de).
\section*{Danksagungen}
An dieser Stelle möchte ich Herrn~Prof.~Dr.~Herrlich für einige
Korrekturvorschläge und einen gut strukturierten Tafelanschrieb
An dieser Stelle möchte ich Herrn~Prof.~Dr.~Herrlich für einige
Korrekturvorschläge und einen gut strukturierten Tafelanschrieb
danken, der als Vorlage für dieses Skript diente. Tatsächlich basiert
die Struktur dieses Skripts auf der Vorlesung von Herrn~Prof.~Dr.~Herrlich
und ganze Abschnitte konnten direkt mit \LaTeX{} umgesetzt werden.
Vielen Dank für die Erlaubnis, Ihre Inhalte in diesem Skript einbauen
zu dürfen!
Vielen Dank auch an Frau Lenz und Frau Randecker, die es mir erlaubt
Vielen Dank auch an Frau Lenz und Frau Randecker, die es mir erlaubt
haben, ihre Übungsaufgaben und Lösungen zu benutzen.
Jérôme Urhausen hat durch viele Verbesserungsvorschläge und Beweise zu einer erheblichen
@ -70,11 +70,11 @@ der Umgang mit komplexen Zahlen $\mdc$, deren Betrag, Folgen und
Häufungspunkten nicht weiter schwer fallen.
Diese Vorkenntnisse werden vor allem in \enquote{Analysis I} vermittelt.
Außerdem wird vorausgesetzt, dass (affine) Vektorräume, Faktorräume,
Außerdem wird vorausgesetzt, dass (affine) Vektorräume, Faktorräume,
lineare Unabhängigkeit, der Spektralsatz und der projektive Raum $\praum(\mdr)$ aus
\enquote{Lineare Algebra I} bekannt sind. In \enquote{Lineare Algebra II}
wird der Begriff der Orthonormalbasis eingeführt.
Obwohl es nicht vorausgesetzt wird, könnte es von Vorteil sein
\enquote{Einführung in die Algebra und Zahlentheorie} gehört zu
\enquote{Einführung in die Algebra und Zahlentheorie} gehört zu
haben.

View file

@ -46,10 +46,10 @@
%%
\usepackage{../shortcuts}
\hypersetup{
pdfauthor = {Martin Thoma},
pdfkeywords = {Geometrie und Topologie},
pdftitle = {Geometrie und Topologie - Definitionen}
\hypersetup{
pdfauthor = {Martin Thoma},
pdfkeywords = {Geometrie und Topologie},
pdftitle = {Geometrie und Topologie - Definitionen}
}
\allowdisplaybreaks

View file

@ -38,10 +38,10 @@
\usepackage[left=10mm,right=10mm, top=2mm, bottom=10mm]{geometry}
\usepackage{../shortcuts}
\hypersetup{
pdfauthor = {Martin Thoma},
pdfkeywords = {Geometrie und Topologie},
pdftitle = {Geometrie und Topologie - Definitionen}
\hypersetup{
pdfauthor = {Martin Thoma},
pdfkeywords = {Geometrie und Topologie},
pdftitle = {Geometrie und Topologie - Definitionen}
}
\allowdisplaybreaks

View file

@ -18,15 +18,15 @@
tick align=outside,
%minor tick num=-3,
enlargelimits=true]
\addplot[domain=0:12, red, thick,samples=500] {1/3*x^1.5};
\addplot[domain=0:12, dotted, orange, thick,samples=500] {1*x^1.5};
\addplot[domain=0:12, dashed, blue, thick,samples=500] {2*x^1.5};
\addplot[domain=0:12, red, thick,samples=500] {1/3*x^1.5};
\addplot[domain=0:12, dotted, orange, thick,samples=500] {1*x^1.5};
\addplot[domain=0:12, dashed, blue, thick,samples=500] {2*x^1.5};
\addplot[domain=0:12, red, thick,samples=500] {-1/3*x^1.5};
\addplot[domain=0:12, dotted, orange, thick,samples=500] {-1*x^1.5};
\addplot[domain=0:12, dashed, blue, thick,samples=500] {-2*x^1.5};
\addplot[domain=0:12, red, thick,samples=500] {-1/3*x^1.5};
\addplot[domain=0:12, dotted, orange, thick,samples=500] {-1*x^1.5};
\addplot[domain=0:12, dashed, blue, thick,samples=500] {-2*x^1.5};
\addlegendentry{$a=\frac{1}{3}$}
\addlegendentry{$a=1$}
\addlegendentry{$a=2$}
\end{axis}
\end{axis}
\end{tikzpicture}

View file

@ -27,5 +27,5 @@
}
]
\addplot3[surf] {y*y-x*x*x};
\end{axis}
\end{axis}
\end{tikzpicture}

View file

@ -14,13 +14,13 @@
\tkzDefLine[orthogonal=through P,/tikz/overlay](O,X) \tkzGetPoint{helper}
\tkzInterLL(O,X)(P,helper) \tkzGetPoint{xp}
\draw [decorate,decoration={brace,amplitude=4pt,mirror}]
(O) -- (xp) node [black,midway,xshift=0cm, yshift=-0.3cm]
(O) -- (xp) node [black,midway,xshift=0cm, yshift=-0.3cm]
{\footnotesize $x_P$};
\tkzDefLine[orthogonal=through P,/tikz/overlay](O,Y) \tkzGetPoint{helper}
\tkzInterLL(O,Y)(P,helper) \tkzGetPoint{yp}
\draw [decorate,decoration={brace,amplitude=4pt}]
(O) -- (yp) node [black,midway,xshift=-0.4cm]
(O) -- (yp) node [black,midway,xshift=-0.4cm]
{\footnotesize $y_P$};
\tkzDrawPolygon(O,xp,P,yp)

View file

@ -10,13 +10,13 @@
\tkzDefLine[orthogonal=through P,/tikz/overlay](O,X) \tkzGetPoint{helper}
\tkzInterLL(O,X)(P,helper) \tkzGetPoint{xp}
\draw [decorate,decoration={brace,amplitude=4pt,mirror}]
(O) -- (xp) node [black,midway,xshift=0cm, yshift=-0.3cm]
(O) -- (xp) node [black,midway,xshift=0cm, yshift=-0.3cm]
{\footnotesize $x_P$};
\tkzDefLine[orthogonal=through P,/tikz/overlay](O,Y) \tkzGetPoint{helper}
\tkzInterLL(O,Y)(P,helper) \tkzGetPoint{yp}
\draw [decorate,decoration={brace,amplitude=4pt}]
(O) -- (yp) node [black,midway,xshift=-0.4cm]
(O) -- (yp) node [black,midway,xshift=-0.4cm]
{\footnotesize $y_P$};
\tkzDrawPolygon(O,xp,P,yp)

View file

@ -18,9 +18,9 @@
\path[name path=trpath] (tl) -- (fr);
\path[name path=tlpath] (tr) -- (fl);
\draw[name intersections={of=brpath and rbpath}] (intersection-1)coordinate (br){};
\draw[name intersections={of=blpath and lbpath}] (intersection-1)coordinate (bl){};
\draw[name intersections={of=trpath and tlpath}] (intersection-1)coordinate (tb){};
\draw[name intersections={of=brpath and rbpath}] (intersection-1)coordinate (br){};
\draw[name intersections={of=blpath and lbpath}] (intersection-1)coordinate (bl){};
\draw[name intersections={of=trpath and tlpath}] (intersection-1)coordinate (tb){};
\shade[right color=gray!10, left color=black!50, shading angle=105] (tf) -- (bf) -- (bl) -- (tl) -- cycle;
\shade[left color=gray!10, right color=black!50, shading angle=75] (tf) -- (bf) -- (br) -- (tr) -- cycle;

View file

@ -63,5 +63,5 @@
\node at (axis cs:0,3) [anchor=east] {$y$};
\node at (axis cs:2,0) [anchor=north] {$x$};
\end{axis}
\end{axis}
\end{tikzpicture}

View file

@ -43,5 +43,5 @@
\node[blue] at (axis cs:0,3) [anchor=east] {$x_2$};
\node[blue] at (axis cs:2,0) [anchor=north] {$x_1$};
\end{axis}
\end{axis}
\end{tikzpicture}

View file

@ -9,7 +9,7 @@
},
}
\begin{tikzpicture}
\draw[->] (-1.5,0) -- (5.5,0) node [below] {$\mathbb{R}$};
\foreach \x in {-1,...,5}

View file

@ -23,5 +23,5 @@
% Draw axis text
\node at (axis cs:-1,0.5) [anchor=east] {$\mathfrak{B}_r(0) = $};
\end{axis}
\end{axis}
\end{tikzpicture}

View file

@ -12,10 +12,10 @@
\draw[grid] (\x,-0.5) -- (\x,2.5);
\draw[grid] (-0.5,\y) -- (2.5,\y);
}
%draw the axes
\draw[axis] (-1,0,0) -- (3,0,0) node[anchor=west]{$y$};
\draw[axis] (0,-1,0) -- (0,3,0) node[anchor=west]{$x$};
\end{tikzpicture}

View file

@ -1,7 +1,7 @@
\begin{tikzpicture}[scale=.5, z={(.707,.3)}]
\draw (2,3,2) -- (0,0,0) -- (4,0,0) -- (4,0,4) -- (2,3,2)
\draw (2,3,2) -- (0,0,0) -- (4,0,0) -- (4,0,4) -- (2,3,2)
-- (4,0,0);
\draw[color=gray, style=dashed] (2,3,2) -- (0,0,4)
\draw[color=gray, style=dashed] (2,3,2) -- (0,0,4)
-- (0,0,0);
\draw[color=gray, style=dashed] (0,0,4) -- (4,0,4);
\end{tikzpicture}

View file

@ -28,14 +28,14 @@
enlargelimits=true,
tension=0.08]
% plot the stirling-formulae
\addplot[domain=-4:8, red, thick,samples=500] {0.5*x};
\addplot[domain=-2:2, red, thick,samples=500] {2*x};
\addplot[domain=-4:4, red, thick,samples=500] {x};
\addplot[domain=-4:8, red, thick,samples=500] {-0.5*x};
\addplot[domain=-4:8, red, thick,samples=500] {0.5*x};
\addplot[domain=-2:2, red, thick,samples=500] {2*x};
\addplot[domain=-4:4, red, thick,samples=500] {x};
\addplot[domain=-4:8, red, thick,samples=500] {-0.5*x};
\addplot[color=red,only marks,mark=o]
plot coordinates {
(1.5,3)
(1.5,1.5)
};
\end{axis}
\end{axis}
\end{tikzpicture}

View file

@ -6,7 +6,7 @@
\node (Rright) at (6,0) {};
\draw[dashed,very thick] (Pleft) -- (P);
\draw[dotted,very thick] (P) -- (R) -- (Rright);
\draw [thick,decoration={brace,mirror,raise=0.2cm},decorate] (Pleft) -- (P) node [pos=0.5,anchor=north,yshift=-0.25cm] {$PR^-$};
\draw [thick,decoration={brace,mirror,raise=0.2cm},decorate] (P) -- (R) node [pos=0.5,anchor=north,yshift=-0.25cm] {$\overline{PR}$};
\draw [thick,decoration={brace,mirror,raise=0.8cm},decorate] (P) -- (Rright) node [pos=0.5,anchor=north,yshift=-0.85cm] {$PR^+$};
\draw [thick,decoration={brace,mirror,raise=0.2cm},decorate] (Pleft) -- (P) node [pos=0.5,anchor=north,yshift=-0.25cm] {$PR^-$};
\draw [thick,decoration={brace,mirror,raise=0.2cm},decorate] (P) -- (R) node [pos=0.5,anchor=north,yshift=-0.25cm] {$\overline{PR}$};
\draw [thick,decoration={brace,mirror,raise=0.8cm},decorate] (P) -- (Rright) node [pos=0.5,anchor=north,yshift=-0.85cm] {$PR^+$};
\end{tikzpicture}

View file

@ -29,5 +29,5 @@
\node at (axis cs:0.8,1.2) [anchor=-90] {$y$};
\draw (axis cs:0.8,1.2) circle[radius=0.6];
\addplot[mark=*] coordinates {(0.8,1.2)};
\end{axis}
\end{axis}
\end{tikzpicture}

View file

@ -9,7 +9,7 @@
},
}
\begin{tikzpicture}
\draw[->] (-0.5,0) -- (1.5,0) node [below] {$\mathbb{R}$};
\foreach \x in {0,...,1}

View file

@ -63,7 +63,7 @@
\addplot[hatchcolor=red,mark=none, pattern=custom north west lines, draw=none] coordinates {(4.5, 0) (4.5,5) (5.5,5) (5.5,0) };
\addplot[red,mark=none, thick] coordinates {(4.5, 0) (4.5,5)};
\addplot[red,mark=none, thick] coordinates {(5.5, 0) (5.5,5)};
\addplot[mark=none, dashed] coordinates {(1, 0) (1,3)};
\addplot[mark=none, dashed] coordinates {(5, 0) (5,3)};
@ -78,5 +78,5 @@
\node[red] at (axis cs:1,-0.3) [anchor=north] {$U_1 \times X_2$};
\node[red] at (axis cs:5,-0.3) [anchor=north] {$U_2 \times X_2$};
\end{axis}
\end{axis}
\end{tikzpicture}

View file

@ -23,5 +23,5 @@
(0,0) (-1,1) (-2,2) (-1,3) (0, 3) (1, 4)};
\addplot[mark=none, blue, smooth cycle, thick, tension=3] coordinates {%
(0,0) (-1,1) (-2,2) (-1,3) (0, 3) (1, 4)};
\end{axis}
\end{axis}
\end{tikzpicture}

View file

@ -21,5 +21,5 @@
\addplot[domain=0:2.5, red, thick,samples=20] {-x+2.5};
\node[point,label={[label distance=0cm]45:$e_0$}] at (axis cs:2.5,0) {};
\node[point,label={[label distance=0cm]0:$e_1$}] at (axis cs:0,2.5) {};
\end{axis}
\end{axis}
\end{tikzpicture}

View file

@ -20,5 +20,5 @@
\node (b)[point,label={[label distance=0cm]5:$e_1$}] at (axis cs:0,2.5) {};
\node (c)[point,label={[label distance=0cm]0:$e_2$}] at (axis cs:2,2) {};
\draw[thick,fill=orange!50] (a.center) -- (b.center) -- (c.center) -- cycle;
\end{axis}
\end{axis}
\end{tikzpicture}

View file

@ -21,6 +21,6 @@
\addplot[domain=0.0105:0.011, blue, thick,samples=20] {10};
\addlegendentry{$\{(x, \sin(\frac{1}{x})) \in X \times Y\}$}
\addlegendentry{$(-1,1) \subseteq Y$}
\end{axis}
\end{axis}
\draw[ultra thick,blue] (0,0.5) -- (0,4);
\end{tikzpicture}

View file

@ -9,7 +9,7 @@
\node (e)[point] at (0,2) {};
\node (f)[point] at (4,2) {};
\end{scope}
\node (p)[point,label={[label distance=0cm]5:$P$}] at (1.5,0.5) {};
\draw[pattern=north east lines] (a.center) -- (b.center) -- (c.center) -- cycle;

View file

@ -9,7 +9,7 @@
\node (e)[point] at (0,2) {};
\node (f)[point] at (4,2) {};
\end{scope}
\node (p)[point,label={[label distance=0cm]5:$P$}] at (1.5,0.5) {};
\draw[pattern=north east lines] (a.center) -- (p.center) -- (b.center) -- cycle;

View file

@ -14,7 +14,7 @@
\begin{scope}[decoration={
markings,
mark=at position 0.6 with {\arrow{>}}}
]
]
\draw[postaction={decorate}] (a.center) -- (b.center);
\draw[postaction={decorate}] (d.center) -- (c.center);
\end{scope}
@ -22,7 +22,7 @@
\begin{scope}[decoration={
markings,
mark=at position 0.55 with {\arrow{>>}}}
]
]
\draw[postaction={decorate}] (b.center) -- (c.center);
\draw[postaction={decorate}] (a.center) -- (d.center);
\end{scope}

View file

@ -21,7 +21,7 @@
\begin{scope}[decoration={
markings,
mark=at position 0.6 with {\arrow{>}}}
]
]
\draw[postaction={decorate}] (a.center) -- (b.center);
\draw[postaction={decorate}] (d.center) -- (c.center);
\end{scope}
@ -29,7 +29,7 @@
\begin{scope}[decoration={
markings,
mark=at position 0.55 with {\arrow{>>}}}
]
]
\draw[postaction={decorate}] (b.center) -- (c.center);
\draw[postaction={decorate}] (a.center) -- (d.center);
\end{scope}

View file

@ -38,7 +38,7 @@
\begin{scope}[decoration={
markings,
mark=at position 0.6 with {\arrow{>}}}
]
]
\draw[postaction={decorate}] (a.center) -- (b.center);
\draw[postaction={decorate}] (d.center) -- (c.center);
\end{scope}
@ -46,7 +46,7 @@
\begin{scope}[decoration={
markings,
mark=at position 0.55 with {\arrow{>>}}}
]
]
\draw[postaction={decorate}] (b.center) -- (c.center);
\draw[postaction={decorate}] (a.center) -- (d.center);
\end{scope}

View file

@ -36,7 +36,7 @@
\begin{scope}[decoration={
markings,
mark=at position 0.6 with {\arrow{>}}}
]
]
\draw[postaction={decorate}] (a.center) -- (b.center);
\draw[postaction={decorate}] (d.center) -- (c.center);
\end{scope}
@ -44,7 +44,7 @@
\begin{scope}[decoration={
markings,
mark=at position 0.55 with {\arrow{>>}}}
]
]
\draw[postaction={decorate}] (b.center) -- (c.center);
\draw[postaction={decorate}] (a.center) -- (d.center);
\end{scope}

View file

@ -13,8 +13,8 @@
\node at ($(C')+(0,0.4)$) {$C'$};
\node at ($(B)+(0.2,-0.2)$) {$B$};
\node at ($(C)+(0.28,0.5)$) {$C$};
\tkzDrawPolygon[ultra thick,color=blue,fill=blue!20](A,B',C')
\tkzDrawPolygon[line width=0.3pt,color=red,fill=red!20](A,B,C)
\tkzDrawPolygon[ultra thick,color=blue,fill=blue!20](A,B',C')
\tkzDrawPolygon[line width=0.3pt,color=red,fill=red!20](A,B,C)
\tkzDrawPoints(A,B',C',B,C)
\tkzLabelSegment[below,red](A,B){$c$}
\tkzLabelSegment[left,red](A,C){$b$}

View file

@ -18,7 +18,7 @@
\tkzDrawLines(A,B)
\tkzDrawLine[dashed,color=orange,add=0.5 and 0.2](F,P)
\tkzDrawLine[dashed,color=blue,add=0.5 and 0.2](G,P)
%
%
\tkzLabelPoint[below left](A){$A$}
\tkzLabelPoint[below left](G){$G$}
\tkzLabelPoint[above left](P){$P$}

View file

@ -18,8 +18,8 @@
enlargelimits=true,
tension=0.08]
% plot the stirling-formulae
\addplot[domain=-4:8, red, thick,samples=500] {0.5*x};
\addplot[domain=-2:2, red, thick,samples=500] {2*x};
\addplot[domain=-4:8, red, thick,samples=500] {-0.5*x};
\end{axis}
\addplot[domain=-4:8, red, thick,samples=500] {0.5*x};
\addplot[domain=-2:2, red, thick,samples=500] {2*x};
\addplot[domain=-4:8, red, thick,samples=500] {-0.5*x};
\end{axis}
\end{tikzpicture}

View file

@ -29,5 +29,5 @@
\addplot[mark=none] coordinates {(\i,-0.2) (\i,5.2)};
}
\addplot[mark=none] coordinates {(0,2) (5,2)};
\end{axis}
\end{axis}
\end{tikzpicture}

View file

@ -68,7 +68,7 @@ in dem Erstellen dieses Skripts steckt:
|07.02.2014 | 11:15 - 11:20 | 5 | Definitionen vereinfacht
|07.02.2014 | 11:35 - 11:45 | 10 | Definition "operiert durch Homöomorphismen" korrigiert
|07.02.2014 | 15:00 - 15:30 | 30 | Verbesserungsvorschläge von Jérôme Urhausen, Email vom 08.02.2014, umgesetzt.
|07.02.2014 | 15:30 - 15:45 | 15 | Verbesserungen
|07.02.2014 | 15:30 - 15:45 | 15 | Verbesserungen
|07.02.2014 | 19:30 - 21:20 | 110 | Textsetzung, kleine Fehler und Verbesserung eines Bildes
|10.02.2014 | 10:30 - 11:05 | 35 | Formulierung in Definitionen vereinfacht; Textsetzung
|10.02.2014 | 11:05 - 11:20 | 15 | Verbesserungsvorschläge von Marco, Email 1 vom 10.02.2014, umgesetzt.
@ -92,6 +92,6 @@ in dem Erstellen dieses Skripts steckt:
|20.02.2014 | 12:00 - 13:00 | 60 | Verbesserungsvorschläge von Jonathan (Facebook, 20.02.2014) eingearbeitet.
|20.02.2014 | 13:00 - 13:45 | 45 | Verbesserungsvorschläge von Jérôme Urhausen, Email 1 vom 20.02.2014, umgesetzt.
|20.02.2014 | 19:30 - 20:15 | 45 | Verbesserungsvorschläge von Jérôme Urhausen, Email 2 vom 20.02.2014, umgesetzt.
| Zwischenstand | --- | --- | 6081 Minuten => Über 100 Stunden!
| Zwischenstand | --- | --- | 6081 Minuten => Über 100 Stunden!
|17.03.2014 | 16:00 - 18:00 | 120 | Textsetzung
|19.03.2014 | 08:00 - 10:00 | 120 | Verbesserung des Symbolverzeichnisses

View file

@ -1,4 +1,4 @@
Zu diesem Skript haben einige Leute beigetragen. Die Personen, die am
Zu diesem Skript haben einige Leute beigetragen. Die Personen, die am
meisten beigetragen haben, stehen direkt im Skript unter "Danksagungen".
Hier ist eine (hoffentlich bald) vollständige Liste der Mitwirkenden (alphabetisch geordnet) mit

View file

@ -4,7 +4,7 @@ bitte ich um eine Email.
Konventionen
============
* `\mathbb{N}` sollte vermieden werden. Stattdessen wird
* `\mathbb{N}` sollte vermieden werden. Stattdessen wird
`\mathbb{N}_0` und `\mathbb{N}_+` verwendet.
* `\subset` sollte vermieden werden. Stattdessen wird
`\subseteq` bzw. `\subsetneq` verwendet.

View file

@ -2,7 +2,7 @@
\subsection*{Teilaufgabe a}
\textbf{Gegeben:}
\[A =
\[A =
\begin{pmatrix}
3 & 15 & 13 \\
6 & 6 & 6 \\
@ -11,7 +11,7 @@
\textbf{Aufgabe:} LR-Zerlegung von $A$ mit Spaltenpivotwahl
\textbf{Lösung:}
\textbf{Lösung:}
\begin{align*}
&
@ -85,7 +85,7 @@ Nun gilt: $P A = L R = A^{(1)}$ (Kontrolle mit \href{http://www.wolframalpha.com
\textbf{Gegeben:}
\[A =
\[A =
\begin{pmatrix}
9 & 4 & 12 \\
4 & 1 & 4 \\
@ -111,7 +111,7 @@ Falls $A$ symmetrisch ist, gilt:
\begin{align}
\det(A_1) &= 9 > 0\\
\det(A_2) &=
\det(A_2) &=
\begin{vmatrix}
9 & 4 \\
4 & 1 \\

View file

@ -7,7 +7,7 @@ wobei $L$ eine invertierbare, untere Dreiecksmatrix ist.
Geben Sie die Formel zur Berechnung von $y_i$ an.
\textbf{Lösung:}
\textbf{Lösung:}
\[y_i = \frac{b_i - \sum_{k=1}^{i-1} l_{ik} \cdot y_k}{l_{ii}}\]
\begin{algorithm}[H]

View file

@ -33,7 +33,7 @@ mit Hilfe der LR Zerlegung nach $\Delta x$ auf.
\overbrace{\begin{pmatrix}
1 & 0\\
\frac{1}{9} & 1
\end{pmatrix}}^{=: L} \cdot
\end{pmatrix}}^{=: L} \cdot
\overbrace{\begin{pmatrix}
3 & 1\\
0 & \frac{8}{9}
@ -81,7 +81,7 @@ Anschließend berechnen wir
\begin{pmatrix}
x_1\\
y_1
\end{pmatrix} &=
\end{pmatrix} &=
\begin{pmatrix}
x_0\\
y_0
@ -89,7 +89,7 @@ Anschließend berechnen wir
\Leftrightarrow\begin{pmatrix}
x_1\\
y_1
\end{pmatrix} &=
\end{pmatrix} &=
\begin{pmatrix}
-\frac{1}{3}\\
0
@ -102,7 +102,7 @@ Anschließend berechnen wir
\Leftrightarrow\begin{pmatrix}
x_1\\
y_1
\end{pmatrix} &=
\end{pmatrix} &=
\begin{pmatrix}
-\nicefrac{13}{18}\\
-\nicefrac{15}{18}
@ -124,7 +124,7 @@ also ausführlich:
\overbrace{\begin{pmatrix}
1 & 0\\
l_{12} & 1
\end{pmatrix}}^L \cdot
\end{pmatrix}}^L \cdot
\overbrace{\begin{pmatrix}
r_{11} & r_{12}\\
0 & r_{22}
@ -139,7 +139,7 @@ also ausführlich:
\begin{pmatrix}
1 & 0\\
l_{12} & 1
\end{pmatrix} \cdot
\end{pmatrix} \cdot
\begin{pmatrix}
3 & \cos y\\
0 & r_{22}
@ -156,7 +156,7 @@ also ausführlich:
\begin{pmatrix}
1 & 0\\
x^2 & 1
\end{pmatrix} \cdot
\end{pmatrix} \cdot
\begin{pmatrix}
3 & \cos y\\
0 & -x^2 \cdot \cos y + e^y

View file

@ -1,5 +1,5 @@
\section*{Aufgabe 4}
\textbf{Aufgabe}:
\textbf{Aufgabe}:
\[I(f) = \int_a^b f(x) \mathrm{d}x \]

View file

@ -15,7 +15,7 @@ maximaler Ordnung zu erhalten. Wie hoch ist die Ordnung?
\paragraph{Lösung}
Nach VL kann bei Vorgabe von $s$ Knoten auch die Ordnung $s$ durch
Nach VL kann bei Vorgabe von $s$ Knoten auch die Ordnung $s$ durch
geschickte Wahl der Gewichte erreicht werden. Nach Satz 27 ist diese
Wahl eindeutig.
Also berechnen wir die Gewichte, um die Ordnung $p=2$ zu sichern.
@ -56,7 +56,7 @@ LGS lösen können:
\begin{align}
\begin{pmatrix}
c_1^0 & c_2^0\\
c_1^1 & c_2^1
c_1^1 & c_2^1
\end{pmatrix}
\cdot x
=

View file

@ -26,10 +26,10 @@
\title{Numerik Klausur 1 - Musterlösung}
\makeatletter
\AtBeginDocument{
\hypersetup{
\hypersetup{
pdfauthor = {Martin Thoma, Peter, Felix},
pdfkeywords = {Numerik, KIT, Klausur},
pdftitle = {\@title}
pdfkeywords = {Numerik, KIT, Klausur},
pdftitle = {\@title}
}
\pagestyle{fancy}
\lhead{\@title}

View file

@ -83,5 +83,5 @@ Das Verfahren ist also:
Alternativ kann man auch in einer angepassten LR-Zerlegung direkt die
Anzahl an Zeilenvertauschungen zählen. Dann benötigt man $P$ nicht mehr.
Ist die Anzahl der Zeilenvertauschungen ungerade, muss das Produkt
Ist die Anzahl der Zeilenvertauschungen ungerade, muss das Produkt
der $r_ii$ negiert werden.

View file

@ -1,6 +1,6 @@
\section*{Aufgabe 2}
\paragraph{Voraussetzung:}
\paragraph{Voraussetzung:}
Gegeben sei eine Funktion $F$:
\begin{align*}
F: \mathbb{R} &\rightarrow [-1, 1]\\
@ -32,23 +32,23 @@ Sei $0 \leq x < y \leq 1$. Dann folgt:\marginnote{Teil 2: $F$ ist auf $[0,1]$ ei
\end{align}
Da $F|_{[0,1]}$ eine Selbstabbildung und eine Kontraktion ist und
offensichtlich $[0,1]$ abgeschlossen ist, greift der
offensichtlich $[0,1]$ abgeschlossen ist, greift der
Banachsche Fixpunktsatz. Es folgt direkt, dass auch für alle $x \in [0,1]$
die Folge $(x)_k$ gegen den einzigen Fixpunkt $x^*$ konvergiert.
\end{proof}
\subsection*{Anmerkung}
Um zu zeigen, dass es genau einen Fixpunkt $x^*$ in $(0,1)$ gibt,
braucht man den Banachschen Fixpunktsatz nicht. Nur um zu zeigen,
Um zu zeigen, dass es genau einen Fixpunkt $x^*$ in $(0,1)$ gibt,
braucht man den Banachschen Fixpunktsatz nicht. Nur um zu zeigen,
dass die Fixpunktiteration auf für jedes $x \in \mathbb{R}$ gegen
diesen Fixpunkt $x^*$ konvergiert, braucht man ihn.
So kann man die existenz eines Fixpunktes zeigen:
Offensichtlich ist $F(0) \neq 0$ und $F(1) \neq 1$, also ist der
Offensichtlich ist $F(0) \neq 0$ und $F(1) \neq 1$, also ist der
Fixpunkt - falls vorhanden - in $(0,1)$. $F$ ist in $(0,1)$ stetig
und streng monoton fallend. Da auch $-x$ in $(0,1)$ streng monoton
fallend ist, folgt, dass $\cos(x) - x$ in $(0,1)$ streng monoton
fallend ist, folgt, dass $\cos(x) - x$ in $(0,1)$ streng monoton
fallend ist.
$x=0 \Rightarrow \cos(x) - x = \cos(0) - 0 = 1$

View file

@ -36,11 +36,11 @@ ist. Wenn diese nun auf $N$ Intervalle aufgepflittet wird gilt folgendes:
\end{align}
$\sum_{i=1}^{N-1} f(a + i \cdot h)$ steht für die Grenzknoten
(deshalb werden sie doppelt gezählt). Von den Grenzknoten gibt es
insgesamt $N-2$ Stück, da die tatsächlichen Integralgrenzen $a$ und $b$
(deshalb werden sie doppelt gezählt). Von den Grenzknoten gibt es
insgesamt $N-2$ Stück, da die tatsächlichen Integralgrenzen $a$ und $b$
nur einmal in die Berechnung mit einfließen.
$\sum_{l=0}^{N-1} f(a + \frac{1}{2} \cdot h + l \cdot h)$ sind die jeweiligen
$\sum_{l=0}^{N-1} f(a + \frac{1}{2} \cdot h + l \cdot h)$ sind die jeweiligen
mittleren Knoten der Intervalle. Davon gibt es $N$ Stück.
\subsection*{Teilaufgabe c)}

View file

@ -18,12 +18,12 @@ $c_1 = 0$ nicht die Gauss-Quadratur sein kann (Satz 31), kommt nur Ordnung $p=4$
und $p=5$ in Frage.
In dieser Aufgabe sind nur die symmetrischen QF, also die von Ordnung
$p=4$ explizit anzugeben. Für die QF von Ordnung $p=5$ hätte man nur
$p=4$ explizit anzugeben. Für die QF von Ordnung $p=5$ hätte man nur
die Gewichte in Abhängigkeit der Knoten darstellen müssen und
eine Bedinung nur an die Knoten herleiten müssen.
\subsection*{Ordnung 4}
Es gilt $g(x) = c$ für eine Konstante $c$, da $\text{Grad}(g(x))=0$ ist.
Es gilt $g(x) = c$ für eine Konstante $c$, da $\text{Grad}(g(x))=0$ ist.
Also ist \ref{a3} gleichbedeutend mit:
\begin{align}
\int_0^1 M(x) \cdot c \mathrm{d}x &= 0 \\
@ -56,7 +56,7 @@ Aus $c_1 = 0 $ folgt, dass $c_3 = 1$ ist. Außerdem muss $c_2 = \frac{1}{2} $ se
Es handelt sich um die aus der Vorlesung bekannte Simpsonregel.
\subsection*{Ordnung 5}
Es gilt $g(x) = ax+c$ für Konstanten $a \neq 0, c$, da $\text{Grad}(g(x))=1$ ist.
Es gilt $g(x) = ax+c$ für Konstanten $a \neq 0, c$, da $\text{Grad}(g(x))=1$ ist.
Also ist \ref{a3} gleichbedeutend mit:
\begin{align}
\int_0^1 M(x) \cdot (ax+c) \mathrm{d}x &= 0 \\

View file

@ -28,10 +28,10 @@
\title{Numerik Klausur 2{} - Musterlösung}
\makeatletter
\AtBeginDocument{
\hypersetup{
\hypersetup{
pdfauthor = {Felix Benz-Baldas, Martin Thoma, Peter Merkert},
pdfkeywords = {Numerik, KIT, Klausur},
pdftitle = {\@title}
pdfkeywords = {Numerik, KIT, Klausur},
pdftitle = {\@title}
}
\pagestyle{fancy}
\lhead{\@title}

View file

@ -15,7 +15,7 @@
\end{align}
\item Abgeschlossenheit: $D$ ist offentsichtlich abgeschlossen.
\item Kontraktion: \\ %TODO:
\item Kontraktion: \\ %TODO:
%\textbf{Behauptung:} $F(x)$ ist auf $A$ eine Kontraktion.
%\textbf{Beweis:}
%z.Z.: $\exists L \in [0, 1): \forall x,y \in A: || F(x) - F(y) || \leq L \cdot || x - y||$

View file

@ -14,7 +14,7 @@ Für die ersten 3. Ordnungsbedingungen gilt:
\end{align*}
\subsection*{Teilaufgabe c}
Wähle die Simpson-Regel, also $c_1=0, c_2 = \frac{1}{2}, c_3 = 1$ und
Wähle die Simpson-Regel, also $c_1=0, c_2 = \frac{1}{2}, c_3 = 1$ und
$b_1 = b_3 = \frac{1}{6}$ und $b_2 = \frac{4}{6}$.
Überprüfe nun Ordnungsbedingungen 1-4 $\Rightarrow$ Simpson-Regel hat Ordnung 4

View file

@ -25,10 +25,10 @@
\title{Numerik Klausur 3 - Musterlösung}
\makeatletter
\AtBeginDocument{
\hypersetup{
\hypersetup{
pdfauthor = {Felix Benz-Baldas, Martin Thoma, Peter Merkert},
pdfkeywords = {Numerik, KIT, Klausur},
pdftitle = {\@title}
pdfkeywords = {Numerik, KIT, Klausur},
pdftitle = {\@title}
}
\pagestyle{fancy}
\lhead{\@title}

View file

@ -1,7 +1,7 @@
\section*{Aufgabe 1}
\textbf{Gegeben:}
\[A =
\[A =
\begin{pmatrix}
6 & -6 & 0 \\
-3 & 7 & 2 \\
@ -15,7 +15,7 @@ b =\begin{pmatrix}
\textbf{Aufgabe:} $Ax = b$ mit Gaußelimination und Spaltenpivotwahl lösen
\textbf{Lösung:}
\textbf{Lösung:}
\begin{align}
\begin{gmatrix}[p]

View file

@ -25,10 +25,10 @@
\title{Numerik Klausur 4 - Musterlösung}
\makeatletter
\AtBeginDocument{
\hypersetup{
\hypersetup{
pdfauthor = {Martin Thoma, Peter, Felix},
pdfkeywords = {Numerik, KIT, Klausur},
pdftitle = {\@title}
pdfkeywords = {Numerik, KIT, Klausur},
pdftitle = {\@title}
}
\pagestyle{fancy}
\lhead{\@title}

View file

@ -25,7 +25,7 @@ Für alle tridiagonalen Matrizen gilt:
Offensichtlich ändert diese Operation nur Zeile 2. $a_{21}$ wird zu 0,
$a_{22}$ ändert sich irgendwie, alles andere bleibt unverändert.
Die gesammte Matrix ist keine tridiagonale Matrix mehr, aber die
Die gesammte Matrix ist keine tridiagonale Matrix mehr, aber die
um Submatrix in $R^{(n-1) \times (n-1)}$ ist noch eine.
Muss man zuvor Zeile 1 und 2 tauschen (andere Zeilen kommen nicht in
@ -35,7 +35,7 @@ an der tridiagonalen Struktur der Submatrix.
\paragraph{Teil 2: (ii) für $A \in \mathbb{R}^{2 \times 2}$}
Sei $\begin{pmatrix}a_{11} & a_{12}\\a_{21} & a_{22} \end{pmatrix} \in \mathbb{R}^{2 \times 2}$
beliebig.
beliebig.
O.B.d.A sei die Spaltenpivotwahl bereits durchgeführt, also $|a_{11}| \geq |a_{21}|$.
@ -55,7 +55,7 @@ Nun folgt:
\end{gmatrix}
\end{align}
Wegen $|a_{11}| \geq |a_{21}|$ gilt:
Wegen $|a_{11}| \geq |a_{21}|$ gilt:
\begin{align}
\|\frac{a_{21}}{a_{11}}\| \leq 1
\end{align}
@ -71,7 +71,7 @@ Damit ist Aussage (ii) für $A \in \mathbb{R}^{2 \times 2}$ gezeigt.
\paragraph{Teil 3: (ii) für allgemeinen Fall}
Aus Teil 2 folgt die Aussage auch direkt für größere Matrizen.
Der worst case ist, wenn man beim Addieren einer Zeile auf eine
Der worst case ist, wenn man beim Addieren einer Zeile auf eine
andere mit $\max_{i,j}|a_{ij}|$ multiplizieren muss um das erste nicht-0-Element
der Zeile zu entfernen und das zweite auch $\max_{i,j}|a_{ij}|$ ist.
Dann muss man aber im nächsten schritt mit einem Faktor $\leq \frac{1}{2}$

View file

@ -4,14 +4,14 @@ Bestimme alle Quadraturformeln mit $s=3$ und Knoten
$0 = c_1 < c_2, c_3$ und Ordnung $p \geq 4$.
Schreiben Sie ein Programm in Pseudocode, welches zu vorgegebenem
$c_2$ den Knoten $c_3$ und die Gewichte $b_i$ möglichst effizient
$c_2$ den Knoten $c_3$ und die Gewichte $b_i$ möglichst effizient
berechnet.
Wie viele symmetrische Quadraturformeln gibt es mit diesen Eigneschaften?
\subsection*{Lösung}
Da $c_1 = 0$ kann es keine Gauß-Quadraturformel sein. Daher kann
die Ordnung nicht $2 \cdot s = 6$ sein. Interessant sind also
Da $c_1 = 0$ kann es keine Gauß-Quadraturformel sein. Daher kann
die Ordnung nicht $2 \cdot s = 6$ sein. Interessant sind also
\begin{itemize}
\item[(A)] Symmetrische Quadraturformeln der Ordnung 4
\item[(B)] Unsymmetrische Quadraturformeln der Ordnung 4

View file

@ -26,10 +26,10 @@
\title{Numerik Klausur 5 - Musterlösung}
\makeatletter
\AtBeginDocument{
\hypersetup{
\hypersetup{
pdfauthor = {Martin Thoma, Peter, Felix},
pdfkeywords = {Numerik, KIT, Klausur},
pdftitle = {\@title}
pdfkeywords = {Numerik, KIT, Klausur},
pdftitle = {\@title}
}
\pagestyle{fancy}
\lhead{\@title}

View file

@ -34,10 +34,10 @@ und diese liegt in $[0,1]$.
\Leftrightarrow x &= - \ln(2x) = F_2(x)\label{a2iif2}
\end{align}
Gleichung \ref{a2iif1} zeigt, dass der Fixpunkt von $F_1$ mit der
Gleichung \ref{a2iif1} zeigt, dass der Fixpunkt von $F_1$ mit der
Nullstelle von $f$ übereinstimmt.
Gleichung \ref{a2iif2} zeigt, dass der Fixpunkt von $F_1$ mit der
Gleichung \ref{a2iif2} zeigt, dass der Fixpunkt von $F_1$ mit der
Nullstelle von $f$ übereinstimmt. Da es nur in $[0,1]$ eine Nullstelle
gibt (vgl. Teilaufgabe i), ist die Einschränkung von $x$ auf $\mathbb{R}^+$
irrelevant.
@ -49,7 +49,7 @@ Rechenungenauigkeit)
$F_1$ ist auf $[0,1]$ eine Kontraktion mit Kontraktionszahl $\theta = \frac{1}{2}$:
Nach dem Mittelwertsatz der Differentialrechnung ex. ein $\xi \in (a,b)$ mit $ 0 \leq a < b \leq 1$, sodass
Nach dem Mittelwertsatz der Differentialrechnung ex. ein $\xi \in (a,b)$ mit $ 0 \leq a < b \leq 1$, sodass
gilt:

View file

@ -28,10 +28,10 @@
\title{Numerik Klausur 6 - Musterlösung}
\makeatletter
\AtBeginDocument{
\hypersetup{
\hypersetup{
pdfauthor = {Martin Thoma, Peter, Felix},
pdfkeywords = {Numerik, KIT, Klausur},
pdftitle = {\@title}
pdfkeywords = {Numerik, KIT, Klausur},
pdftitle = {\@title}
}
\pagestyle{fancy}
\lhead{\@title}

View file

@ -1,7 +1,7 @@
Diese Lösungen sind noch im Aufbau.
Wenn du einen Fehler findest (auch Textsetzungs- und Rechtschreibfehler
oder missverständliche Stellen)
oder missverständliche Stellen)
oder selbst eine Lösung geschrieben hast, kannst du mir eine Email
schreiben (info@martin-thoma.de). Oder du machst direkt einen Pull-Request.

View file

@ -53,7 +53,7 @@ sie auf jeden Fall für $a=1, b=0$ sowie für $a=1, b=1$ gelten. Aber:
\frac{2\cdot1+5\cdot0}{5\cdot1+10\cdot0} = \frac{3}{5} &\neq \frac{8}{15} = \frac{3\cdot1+5\cdot1}{5\cdot1+10\cdot1}
\end{align}
Offensichtlich gibt also es kein $c_2$, dass diese Bedingung für jedes $a,b \in \mathbb{R}$
Offensichtlich gibt also es kein $c_2$, dass diese Bedingung für jedes $a,b \in \mathbb{R}$
erfüllt. Daher kann es keine Quadraturformel der Ordnung $5$ mit den Knoten
$0$ und $1$ geben.
@ -83,7 +83,7 @@ Und damit:
\end{align}
Nun könnte man das ganze in die 4. Ordnungsbedinung einsetzen \dots aber ich
glaube nicht, dass das schön wird. Mache das, wer will.
glaube nicht, dass das schön wird. Mache das, wer will.
\subsubsection*{Ordnung 4}
Die Simpson-Regel erfüllt offensichtlich alle Bedinungen und hat
@ -97,4 +97,4 @@ Ordnung 4:
\end{align}
Dass die Simpson-Regel Ordnung 4 hat, lässt sich schnell über
die Ordnungsbedingungen zeigen.
die Ordnungsbedingungen zeigen.

Some files were not shown because too many files have changed in this diff Show more